BOC Quiz Questions

अब Quizwiz के साथ अपने होमवर्क और परीक्षाओं को एस करें!

What MINIMUM level of a particular factor will cause the aPTT test to become prolonged? a. Less than 40% b. Less than 50% c. Less than 60% d. Less than 70%

A. A prolonged aPTT test result will occur only if one or more factor levels that are measured by this test are less than 40% of the normal level(s).

Which of the following is the recommended method for preparing a cerebrospinal fluid (CSF) sample for a differential count? a. Cytocentrifugation b. Concentration of cells by traditional centrifugation c. Manual smear methods d. Use of hemocytometer

A. A slide for microscopic examination of CSF should be made using a cytocentrifugation technique (cytospin). Alternate methods of slide preparation, such as concentration of cells by traditional centrifugation, manual smear methods, and counting chambers are not recommended as morphologies may not be maintained and sufficient cells may not be present on the slide for accurate evaluation of the sample.

The fibrinolytic system may be activated by which one of the following? a. Streptokinase b. Streptolysin c. Fibrin d. EDTA

A. Streptokinase is an exogenous activator produced by some bacterial species that binds and activates plasminogen, a component of the fibrinolytic system. Streptolysin is a streptococcal hemolytic exotoxin that can cause hemolysis in humans. Fibrin is the end product of the coagulation cascade in which the fibrinolytic system acts upon. EDTA is an anticoagulant used in specimen collection tubes, which does not activate the fibrinolytic system.

How should the production of a blue or purple color on the test pad of an Ictotest® be interpreted? a. Positive for bilirubin b. Positive for ketones c. Negative for bilirubin d. Negative for ketones

A. The production of a blue or purple color on the test pad of an Ictotest® be interpreted as positive for bilirubin.

Which Plasmodium morphologic form is considered most immature? a. Trophozoite b. Schizont c. Microgametocyte d. Macrogametocyte

A. The trophozoite (ring form) is the most immature for the Plasmodium species maturation in the red blood cells. The stages of maturation are: early trophozoite, late trophozoite, schizont, microgametocyte, and macrogametocyte.

Which of the following needle gauges has the smallest lumen or bore? a. 18 b. 20 c. 21 d. 22

D. There is an inverse proportional between the gauge number and the bore/lumen of the needle. The larger the gauge number, the smaller the bore/lumen. In our case, 22 gauge needle would have the smallest bore/lumen among the given choices.

When blood flow goes down, what happens to the blood lactate levels?

Increases

Which of the following types of molecules is most likely to be antigenic/immunogenic? a. Nucleic acids b. Carbohydrates c. Lipids d. Proteins

Proteins are complex molecules with a high molecular weight and are more antigenic than the other types of molecules. Carbohydrates are the second most antigenic molecules. Lipids are the third antigenic molecules from this list. Nucleic acids are not generally antigenic.

An immunosuppressed patient with a CSF shunt is exhibiting symptoms of a shunt infection. When performing a Gram stain, the organism observed in the image to the right is seen. What is the most likely identity of this organism? a. Candida sp. b. Staphylococcus sp. c. E. coli d. Enterococcus sp.

The correct answer is A. Candida sp. The structures observed in the image to the right are budding yeast cells. The most common CSF shunt infection observed in immunosuppressed patients is caused by Candida sp. Staphylococcus sp. are the most common cause of CSF shunt infections, but would exhibit gram positive cocci in clusters, which is not observed in this image. E. coli is a Gram negative rod, which is not observed in this image, although it can cause CSF shunt infections. Enterococcus sp. are not a common cause of CSF shunt infections and would exhibit Gram positive cocci in pairs and chains, which is not observed in this image.

The trophozoites indicated by arrows and the 10 µm oval cyst at the middle left in the photomicrograph were observed in a stool specimen from a person with mild intermittent diarrhea. What is the presumptive identification of this organism? a. Iodamoeba butschlii b. Chilomastix mesnili c. Entamoeba hartmanni d. Endolimax nana

a. Iodamoeba butschlii trophozoites are relatively small, with "ball in socket" nuclei adjacent to the outer membrane. Observed also is a cyst with a single nucleus with an irregular karyosome surrounded by a vacuolated cytoplasm. An outer chromatin ring is not observed. Of interest, this cyst would appear yellow-brown when stained with iodine, from which the genus name is derived. Chilomastix mesneli trophozoites are small, pear-shaped with a single large nucleus placed immediately beneath the outer membrane and an adjacent cytosome. The cysts are lemon-shaped with a thin smooth cell wall, ranging from 6 - 10 µm, with a distinct anterior knob. A single nucleus with a small karyosome is devoid of an outer chromatin ring. A cytostome in the form of a "shepherd's crook" is adjacent to the nucleus. Entamoeba hartmanni trophozoites have an eccentric karyosome or uneven chromatin similar to E. histolytica. The ova may present with from one up to four nuclei, each distinctive for having a central large karyosome surrounded by a blotchy chromatin ring. A small chromatin bar with rounded edges may also be seen. Endolimax nana trophozoites also have a large karyosome with no peripheral chromatin. Differentiation is made by observing the cysts that are small, ranging from 6 - 8 µm, and are characterized by the presence of four nuclei, each with a minute central karyosome surrounded by a clear space.

Identify the nucleated blood cell: (Picture not included. Same size as a RBC) a. Lymphocyte b. Neutrophil c. Basophil d. Monocyte

a. Lymphocyte is the correct answer. Lymphocytes are non-granular white cells with a large dark staining nucleus and very little cytoplasm which stains light blue. They are the smallest white cells by size and typically second most numerous after neutrophils. Neutrophils are the most numerous granulocytic white blood cells. The neutrophils' cytoplasm stains pink, with bluish/lavender granules, and they have multi-lobulated nuclei which stains dark blue/purple. Basophils are the least numerous granulocytic white blood cells. They can be identified by their large dark blue/purple staining granules and S shaped nucleus. Monocytes are non-granular and the largest white blood cell by size. They have a folded nucleus and an irregular cellular shape. The cytoplasm stains gray-blue and often contains vacuoles.

Which of the following reagent strip tests is based on the Ehrlich-aldehyde reaction? a. Urobilinogen b. Specific gravity c. Glucose d. pH

a. The urobilinogen urine chemical reagent strip test is based on the Ehrlich-aldehyde reaction in which urobilinogen reacts with p-dimethylaminobezaldehyde (Ehrlich reagent). In this test, a pink-red color forms in proportion to the amount of urobilinogen present.

Using an automated cell counter analyzer, an increased Red Cell Distribution Width (RDW) should correlate with which of the following? a. Leukocytosis b. Anisocytosis c. Spherocytosis d. Macrocytosis

b. Anisocytosis Anisocytosis, or the increased variation in red cell size, would be expected if the red cell distribution width (RDW) was elevated. Leukocytosis defines an increase in white blood cell counts. Spherocytosis could cause a lowered MCV, but would not affect the RDW as much as anisocytosis would. Finally, if macrocytosis is present, the MCV would be increased, but unless there is anisocytosis present also (meaning microcytes and normocytes) the RDW would not be greatly affected.

Which of the following organisms has been known to cause epidemic tinea capitis in children but almost never affects adults? a. Microsporum canis b. Microsporum audouinii c. Trichophyton verrucosum d. Epidermophyton floccosum

b. Microsporum audouinii Tinea capitis is a disease caused by superficial fungal infection of the scalp. Tinea capitis is the most common pediatric dermatophyte infection in the world. Microsporum audouinii is a causative agent of tinea capitis in children, but very rarely in adults. The organism rarely produces distorted, spindle-shaped macroconidia or microconidia. Microsporum canis is a zoonotic dermatophyte that causes tinea infections of the skin and hair. Children and adults acquire the infection through contact with infected animals. The organism produces large, spindle-shaped, echinate macroconidia with a few microconidia. Trichophyton verrucosumis a dermatophyte that causes tinea infections of the skin, hair, and nails. It is most commonly associated with skin and hair infections of the beard, neck, and wrists of farmers that work with infected cattle. The organism produces pencil-shaped macroconidia and numerous microconidia. Epidermophyton floccosum is a dermatophytes that causes tinea infections of the skin and nails. It commonly causes tinea pedis and tinea cruris. The macroconidia are described as multi-celled and club-shaped. Microconidia are not produced.

Why is Rh immune globulin (RhIg) administered within 72 hours of delivery to an Rh negative mother if the newborn is found to be D-positive or weak-D positive? a. Prevent future children from producing antibodies b. Prevent fetal cells from initially sensitizing the mother c. Prevent antibody response in a previously sensitized mother d. Neutralize any natural maternal antibodies present

b. The production of IgG antibodies (particularly anti-D) can have threatening consequences for the fetus. RhIg is available and prevents alloimmunization in D negative mothers exposed to D positive red cells. RhIg suppresses the immune response after exposure to D positive fetal cells and prevents the mother from producing anti-D. All Rh negative women should receive at least one full dose of RhIg at 28 weeks gestation and another full dose after birth (within 72 hours of delivery, assuming that the newborn is found to be D-positive or weak D-positive). This situation develops from the fetal cells having the D antigen and not from fetal antibodies. Once a woman is alloimmunized and produces antibodies, the condition cannot be reversed. Anti-D is not considered a natural antibody.

Parvovirus B19 is the only human pathogen in the family Parvoviridae. It is most known for its association with which of the following diseases? Please select the single best answer a. Hepatitis b. Kaposi sarcoma c. Fifth disease d. Measles

c. Fifth disease Parvovirus B19 is the known causative agent of Fifth disease, so named because it is the fifth of the childhood exanthems. Replication of the virus is largely restricted to erythroid progenitor cells, bone marrow, and fetal liver cells. Generally, fifth disease only demonstrates a characteristic facial rash; however, in patients with anemia, infection with parvovirus B19 may lead to severe outcomes. Hepatitis is caused by hepatitis B virus (HBV) of the family Hepadnaviridae. This virus has long been known to be the cause of significant liver damage associated with morbidity and mortality. Despite the creation of a successful vaccine, an estimated 50 million new cases of hepatitis are seen annually worldwide. Kaposi sarcoma, a tumor of the connective tissue, is caused by human herpes virus 8 (HHV-8). It is a member of the family Herpesviridae, along with other viruses such as herpes simplex virus types 1 and 2 (HSV-1, HSV-2), varicella-zoster virus (VZV), Epstein-Barr virus (EBV), and cytomegalovirus (CMV). The measles virus, causative agent of measles, is a member of the Paramixoviridae family along with the mumps virus, respiratory syncytial virus (RSV), and metapneumovirus. The measles virus is spread through contact with respiratory secretions of infected individuals. Despite an effective vaccine and regular childhood vaccination in most populations, outbreaks of measles continue to be seen.

Sodium thiosulfate is often added to selective culture media to a. Detect hydrogen sulfide b. Inhibit the growth of gram positive bacteria c. Provide sulfur atoms for H2S positive isolates d. Serve as a pH buffer

c. Sodium thiosulfate is added to selective culture media such as Hektoen Enteric (HE) agar as a rich source of sulfur atoms for those bacteria capable of producing hydrogen sulfide. The hydrogen sulfide is detected by ferric ammonium sulfate or other detectors. Bile salts are added to prevent the growth of gram positive bacteria and phosphates are most commonly used as pH buffers.

What abnormaility is present in the white blood cell indicated by the arrow? (Can't include picture-Neutrophil with darkly pigmented spots) a. Döhle bodies b. Pelger-Huet anomaly c. Toxic granulation d. Auer rods

c. Toxic granulation The cell is exhibiting toxic granulation. Toxic granulation is found in neutrophils and can be seen in certain infections and inflammations. Dohle bodies are light blue-gray bodies and are composed of aggregates of rough endoplasmic reticulum. They are associated with severe bacterial infections and are often seen with toxic granulation (although not in this image). Pelger-Huet anomaly is a benign neutrophilic disorder characterized by hyposegmented nuclei. Auer rods are fused primary granules and are therefore only seen in cells that would contain primary granules, such as myeloblasts and promyelocytes.

All of the following are associated with RBC macrocytosis EXCEPT? Please select the single best answer a. Alcoholism b. Hypothyroidism c. B12 deficiency d. Thalassemia

d. Thalassemia Thalassemia is usually associated with microcytosis. The other conditions can cause macrocytosis of RBCs. Macrocytosis occurs in vitamin B12 deficiency due to defective synthesis of DNA. In B12 deficiency nucleated red cells do not undergo normal mitotic divisions due to thymidine deficiency. Macrocytosis occurs in alcoholism due to decreased levels of folate from a folate deficient diet. Folate like Vitamin B12 is necessary for synthesis of DNA. Macrocytosis occurs in patients with autoimmune hypothyroidism due to production of antibodies which inhibit intrinsic factor production in the gastric mucosa. Intrinsic factor is necessary for Vitamin B12 absorption.

Brucella suis and Salmonella typhi can commonly infect the spleen, liver, and bone marrow. Which of the following reasons allow these two organisms to cause a systemic infection? The correct answer is highlighted below a. They both can only infect the spleen, liver, or bone marrow b. They are ingested by eosinophils and the eosinophils take them to the spleen, liver, and bone marrow c. They are both ingested by monocytes and/or macrophages within the spleen, liver, and bone marrow d. They both have the capability to suppress interleukins of the immune system, which will allow the spread of the organisms throughout the blood stream causing infections of the spleen, liver, and bone marrow

c. They are both ingested by monocytes and/or macrophages within the spleen, liver, and bone marrow is the correct answer because monocytes and macrophages will phagocytize infected neutrophils within the bloodstream as they enter the spleen, liver, and bone marrow. Monocytes/Macrophages are part of the reticuloendothelial system to remove debris and dead or dying cells out of the blood stream. Since these organisms can remain viable not only in neutrophils, but in monocytes and macrophages, granulomas can develop in these organs causing infection. They both can only infect the spleen, liver, or bone marrow is incorrect because Brucella suis can cause wound infections and has been linked to endocarditis. Salmonella typhi is a gastrointestinal pathogen that causes diarrhea and has been linked to infection of the gall bladder leading to a carrier status. They are ingested by eosinophils and the eosinophils take them to the spleen, liver, and bone marrow is incorrect because eosinophils do not ingest the organisms. The neutrophils are the cells that do the initial phagocytizing and then the monocytes/macrophages will remove the infected cells as they enter the spleen, liver, and bone marrow. They both have the capability to suppress interleukins of the immune system, which will allow the spread of the organisms throughout the blood stream causing infections of the spleen, liver, and bone marrow is incorrect because they do not suppress interleukins within the immune system. Brucella suis has the capability to interfere with the phagosome-lysosome fusion within macrophages and Salmonella typhi has many virulent factors that allow the organism to replicate and survive within the macrophages.

The 30 X 50 µm ovum illustrated in the image are most commonly observed by microscopic examination of transparent adhesive tape mounts of perianal skin of children who have complained of nocturnal anal pruritus. What is the most likely presumptive identification? (Looks like the letter D) a. Cystoisospora belli b. Necator americanus c. Enterobius vermicularis d. Trichuris trichiura

-Enterobius vermicularis is a correct response. The eggs are collected using a sticky paddle or a piece of clear cellophane tape pressed against the perianal region. Enterobius ova are oval in outline with flattening along one margin, simulating a deflated football. The shell is smooth and slightly thickened. A well-developed larva is commonly observed internally, which retracts away from the inner shell membrane, leaving an open space. -Cystoisospora belli is an incorrect response. Cystoioocysts are also oval in outline and have a smooth, thin outer shell. Internally, a single spherical sporocyst may be observed, but more typically mature oocysts are noted which contain two sporocysts. One is advised to search further in a smear preparation for detection of oocysts with double sporocysts to ensure an accurate identification. -Necator americanus is an incorrect response. Necator ova, although also oval in outline, are not flattened on one side and the outer shell is thin and transparent. Although the inner yolk sac retreats, leaving a clear space beneath the shell, further development into an embryo is not observed. -Trichuris trichiura is an incorrect response. Trichuris ova are easy to recognize with their barrel-shape and a distinctive protruding, convex, hyaline polar plug at each end. The shell is smooth but relatively thick and the internal developing embryo reaches the inner lining of the shell without leaving an open space.<

A deficiency of platelet receptor IIb/IIIa can be found in which of the following? a. Glanzmann Thrombasthenia b. Von Willebrand Disease c. Aspirin therapy d. Bernard Soulier Syndrome

-Glanzmann Thrombasthenia is an inherited disorder in which the platelets are missing the receptor IIb/IIIa which is responsible for binding fibrinogen for platelet aggregation. -Von Willebrand Disease is deficient in Von Willebrand Factor and not a platelet receptor. -Aspirin therapy inactivates Thromboxane A2 which binds to platelet receptor TPa. -Bernard Soulier Syndrome is deficient in platelet receptor Ib/IX/V which binds VWF.

A patient is group AB with one copy of the Se gene. What will they have in their secretions? a. A, B, & H b. Only B & H c. Only A & H d. Only H

A. A, B, and H antigens are found on red blood cells but they can also be found in secretions if the individual has at least one copy of a secretor gene (Se). The Se gene codes for a transferase that modifies type I precursors in secretions to form H. Once H is formed, immunodominant sugars can be added to produce A and B antigens in secretions. In this question, AB patients who are secretors will have A, B, and H substances secreted. Group B patients who are secretors will have both B and H substances secreted. Group A patients who are secretors will have both A and H substances secreted. Group O secretors will only secrete H substance.

The ketone component that is measured by the nitroprusside reaction is: a. Acetoacetic acid b. Beta-hydroxybutyric acid c. Acetone d. Glycine

A. Acetoacetic acid reacts with sodium nitroprusside to produce a purple color. B-hydroxybutyrate and acetone are derived from acetoacetic acid. However, B-hydroxybutyrate is not measured and only slightly sensitive to acetone in presence of glycine.

Which one of the following statements concerning Tamm-Horsfall protein is FALSE? a. It is the protein that is predominantely detected by a urine reagent strip test. b. Hyaline casts are formed by this protein in the distal tubule. c. It acts as a coating and lubricant in the tubules. d. It is produced by the tubules.

A. Albumin, not Tamm-Horsfall, comprises the majority of the protein that is detected by the urine reagent strip test. Tamm-Horsfall protein is produced by the tubules. Tubular epithelial cells would have great difficulty functioning without it because it acts as a coating and lubricant in the tubules. Hyaline casts are formed by this protein in the distal tubule. Increased levels of protein in the urine (proteinuria) is an indicator of renal disease. The glomerular membrane prevents the passage of high molecular weight proteins including albumin (mol wt 69,000). The reagent test strip reaction is based on the "protein error of indicators" principle which means that the point of color change of pH indicators is different when protein is present versus when protein is not present. This phenomenon occurs because proteins act as hydrogen ion acceptors at a constant pH. Albumin is the main protein which is measured by the urine protein pad on the reagent strip because it contains more amino groups to accept hydrogen ions than other groups. Albumin is also the protein which is found in the highest concentrations in the body.

Which form of hemoglobin cannot be measured using the cyanmethemoglobin method? a. Sulfhemoglobin b. Carboxyhemoglobin c. Oxyhemoglobin d. Hemoglobin

A. All hemoglobin derivatives except sulfhemoglobin are converted to cyanmethemoglobin. This is because sulfhemoglobin cannot be oxidized by potassium ferricyanide. The ferrous ions (Fe2+) in carboxyhemoglobin, oxyhemoglobin, or hemoglobin are first oxidized by potassium ferricyanide to ferric ions (Fe3+). This results in methemoglobin which then combines with cyanide ions (CN-) to form cyanmethemoglobin.

B lymphocytes and T lymphocytes are derived from: a. Hematopoietic stem cells b. Macrophages or monocytes c. Mucosa-associated lymphoid tissue (MALT) d. Granulocytes

A. All lymphocytes originate, and are therefore derived from, hematopoietic stem cells via hematopoiesis. The stem cells eventually mature into common lymphoid progenitor cells. The progenitor cells then differentiate into their distinct lymphocyte types. B cells mature into B lymphocytes in the bone marrow, while T cells migrate to and mature in the thymus.

Aspiration material recovered from a ruptured appendix abscess was inoculated to anaerobic culture media per laboratory protocol. Gray colonies (> 1 mm) grew anaerobically in 24 hours on BBE agar, blackening the media. The photograph illustrates the gram stain features of the organism. The bacterial species most likely associated with the abscess is: a. Bacteroides fragilis group b. Fusobacterium nucleatum c. Porphyromonas species d. Prevotella species

A. Bacteroides fragilis group are anaerobes most commonly recovered from human infections, and commonly associated with parappendiceal abscesses. This organism group grows readily on Bacteroides Bile Esculin (BBE) agar in an anaerobic atmosphere and produces gram negative bacilli with rounded ends, as seen in the photograph. Fusobacterium nucleatum can also be associated with intra-abdominal cavity abscesses; however, the bacterial cells are slender and tapered and growth is inhibited by 20% bile, and esculin is not produced. Characteristic of the Bacteroides fragilis group is the ability to grow in 20% bile and to hydrolyze esculin, two key characteristics that separate the B. fragilis group from Prevotella species and Porphyromonas species, which are negative for both.

All of the following tests are affected by the pneumatic tube system transport, EXCEPT? a. Albumin b. Potassium c. Plasma hemoglobin d. Acid phosphatase

A. CLSI states that the following tests are NOT affected by Pneumatic tube system transport: albumin, alkaline phosphatase, AST, chloride, creatinine, glucose, sodium, total bilirubin, total protein, BUN, uric acid, thrombin time, and WBC concentration. Pneumatic tube system is a convenient transport medium used in healthcare facilities. When it comes to transporting laboratory specimens, special strict protocols are followed to ensure integrity of the specimen, safety of the operator, and to prevent contamination issues. According to CLSI guideline, the tests negatively affected by pneumatic tube system transport are those influenced due to red cell disruption such as potassium, plasma hemoglobin, acid phosphatase, and lactate dehydrogenase.

A representative congenital neutrophil functional disorder is: Please select the single best answer a. Chédiak-Higashi syndrome b. Gaucher's disease c. Niemann-Pick Disease d. Systemic inflammatory response syndrome (SIRS)

A. Chédiak-Higashi syndrome represents a qualitative disorder of neutrophils. It is a rare disorder th is expressed as an abnormal granulation of neutrophils. Neutrophils with giant granules display impaired chemotaxis and delayed killing of ingested bacteria. Gaucher's disease is a rare genetic defect of macrophages. In this disorder a deficiency of ß-glucocerebrosidase, the enzyme that normally splits glucose from its parent sphingolipid, glucosylceramide. As a result of the enzyme deficiency, cerebroside accumulates in macrophage. Gaucher's cells are rarely seen in circulating blood but are found in the bone marrow, spleen, and other organs of mononuclear phagocyte system. Niemann-Pick Disease is similar to Gaucher's disease as an inherited abnormality of lipid metabolism. It is characterized by massive accumulation of sphingomyelin in the mononuclear phagocytes. Sepsis, severe sepsis, and septic shock are progressively severe stages of Systemic inflammatory response syndrome (SIRS). Sepsis begins when the innate immune system responds aggressively to the presence of bacteria. Toll-like receptors cause the antigen-presenting cell to produce proinflammatory cytokines.

From the following organisms, which one is considered to be the LEAST pathogenic to humans? a. Corynebacterium ulcerans b. Bacillus anthracis c. Listeria monocytogenes d. Nocardia brasiliensis

A. Corynebacterium ulcerans is correct because this organism is considered a zoonosis and can only occur with close animal contact and human infections are rare. This organism has been linked to causing bovine mastitis. Bacillus anthracis, Listeria monocytogenes, and Nocardia brasiliensis have been linked to causing serious human infections. Bacillus anthracis is the most virulent organism listed. Persons with cutaneous anthrax may have a 20% chance of fatality if untreated; virtually 100% of those with inhalation anthrax will succumb to the infection. Listeria monocytogenes can cause human infections through contaminated food or neonatal infections from colonized mothers during pregnancy. Nocardia brasiliensis can cause subcutaneous infections such as actinomycetoma that tend to remain localized to the skin.

Which type of toxicology is primarily concerned with medical and legal consequences of exposure to chemicals and drugs? a. Forensic toxicology b. Clinical toxicology c. Environmental toxicology d. Descriptive toxicology

A. Forensic toxicology is primarily concerned with the medical and legal consequences of exposure to chemicals and drugs. A major focus of forensic toxicology is establishing and validating the analytic performance of test methods used to generate evidence in legal situations, including cause of death. Clinical toxicology focuses on the relationships between xenobiotics and disease states. This area emphasizes not only diagnostic testing but also therapeutic intervention. Environmental toxicology includes the evaluation of environmental chemical pollutants and their impact on human health. Descriptive toxicology uses the results from animal experiments to predict what level of exposure will cause harm in humans.

Which of the following describes gastrin's role in digestion? a. Stimulates gastric acid-HCl secretion b. Stimulates secretion of a base to lower the stomach's pH c. Is secreted when the gastric fluid pH is low to buffer fluid d. Inhibits the secretion of intrinsic factor

A. Gastrin is secreted by the gastrointestinal mucosa cells in response to mechanical stress or high pH, both of which are produced by the presence of food in the stomach. Gastrin stimulates the stomach parietal cells to produce gastric hydrochloric acid.

The morphologic structure of select tapeworms that is most likely responsible for piercing the human intestinal wall is/are called: a. Hooklets b. Polar plugs c. Operculums d. Polar thickenings

A. Hooklets are structures attached to the rostellum of tapeworms (Cestodes). These hooklets are used to aid in attachment of the worm to the host's intestinal lining. An example of a cestode that has hooklets is Taenia solium. Polar plugs are characteristic features that aid in identifying the Trichuris species. The ends and the oval shape give the organism a lemon-drop appearance. These structures do not aid in the organism's attachment to the intestinal lining. A number of helminth eggs (primarily cestode & trematode eggs) each have an operculum. This lid structure under the appropriate conditions opens and allows the egg contents (larva) to escape and continue its development. This structure does not aid in the organism's attachment to the intestinal lining. Polar thickenings are characteristic features that aid in identifying the Hymenolepis species. These thickenings do not aid in the organism's attachment to the intestinal lining.

A culture was performed on a stool sample. After growth was obtained on the blood agar plate, the gram stain showed curved gram negative rods. The oxidase, catalase, and urease were positive. What is the most likely bacterial identification? a. Helicobacter pylori b. Campylobacter concisus c. Helicobacter cinaedi d. Arcobacter spp.

A. Identification methods for curved gram negative rods will include (in this order) oxidase, catalase, and urease. Helicobacter pylori will have the gram stain showing curved gram negative rods. It will have positive results for oxidase, catalase, and urease. Campylobacter concisus will have the gram stain showing curved gram negative rods. It will have positive results for oxidase along with a negative catalase result. Helicobacter cinaedi will have the gram stain showing curved gram negative rods. It will have positive results for oxidase and catalase along with a negative urease test. Arcobacter spp. will have the gram stain showing curved gram negative rods. It will have positive results for oxidase and catalase along with a negative urease test.

A culture was performed on a stool sample. After growth was obtained on the blood agar plate, the gram stain showed curved gram negative rods. The oxidase, catalase, and urease were positive. What is the most likely bacterial identification? a. Helicobacter pylori b. Campylobacter concisus c. Helicobacter cinaedi d. Arcobacter spp.

A. Identification methods for curved gram negative rods will include (in this order) oxidase, catalase, and urease. Helicobacter pylori will have the gram stain showing curved gram negative rods. It will have positive results for oxidase, catalase, and urease. Campylobacter concisus will have the gram stain showing curved gram negative rods. It will have positive results for oxidase along with a negative catalase result. Helicobacter cinaedi will have the gram stain showing curved gram negative rods. It will have positive results for oxidase and catalase along with a negative urease test. Arcobacter spp. will have the gram stain showing curved gram negative rods. It will have positive results for oxidase and catalase along with a negative urease test.

The granular, wrinkled, yellow-pigmented colonies shown in the upper photograph were recovered on chocolate agar after 48 hours of incubation from a respiratory specimen collected from a patient with cystic fibrosis. Note in the lower composite photograph that colonies on the surface of blood agar are flat, granular, and only slightly wrinkled. On MacConkey agar the colonies lack lactose fermentation and lack pyocyanin production. The spot oxidase test is positive, and nitrates are reduced. With these observations, select the presumptive identification of this isolate. a. Pseudomonas stutzeri b. Burkholderia cepacia c. Elizabethkingia meningoseptica d. Acinetobacter baumannii

A. Pseudomonas stutzeri is the correct response. Colonies on blood agar are spreading, convex and wrinkled with a yellow-white pigmentation. Colonies on MacConkey agar are characteristic of a non-fermenter and devoid of the green pigmentation indicating lack of pyocyanin production. A gram stain would reveal long gram negative bacilli comparable to Pseudomonas aeruginosa, along with other biochemical characteristics, including a positive oxidase reaction. Characteristic of Pseudomonas stutzeri is reduction of nitrates, hydrolysis of starch, and arginine negative. Burkholderia cepacia is incorrect because the colonies are relatively small, convex, and smooth devoid of "wrinkles", with gray yellow pigmentation. Colonies on MacConkey agar are non-pigmented characteristic of a non-fermenter and negative for pyocyanin production. The oxidase reaction is positive, nitrate denitrification is negative, lysine is positive, and is resistant to Polymyxin B. Elizabethkingia meningoseptica is incorrect because it produces circular, smooth colonies with light yellow pigment on media. It doesn't grow well on MacConkey. Acinetobacter baumannii is incorrect because colonies are white, convex, and smooth without wrinkles on blood agar. Colonies growing on MacConkey agar are smooth to finely wrinkled, with a light pink pigmentation, devoid of pyocyanin production. The oxidase reaction is negative. A Gram stain reveals gram negative coccobacilli that can falsely appear as a gram positive coccobacilli when grown in blood culture medium.

Which of the following organisms causes Q fever? a. Coxiella burnetii b. Anaplasma spp. c. Orientia tsutsugamushi d. Rickettsia rickettsii

A. Q fever, an acute systemic infection affecting the lungs, is caused by Coxiella burnetii. Anaplasma spp. causes human granulocytic anaplasmosis. Orientia tsutsugamushi causes scrub typhus. Rickettsia rickettsii causes Rocky Mountain spotted fever.

Why might serum ferritin (SF) alone be considered a less than optimal screening test for hereditary hemochromatosis (HH)? a. It is an acute phase reactant that is frequently elevated in a variety of clinical conditions. b. It does not assess an individual's iron status. c. It is an expensive test to perform. d. It is only performed in laboratories doing molecular assays.

A. Serum Ferritin (SF) is an acute phase reactant that is frequently elevated in a variety of clinical conditions, therefore it lacks the specificity needed for a good screening test. In hereditary hemochromatosis (HH), signs and symptoms usually don't present until mid-life and include joint pain, fatigue, and bronze discoloration of the skin. As the disease advances, hepatomegaly develops which can lead to cirrhosis of the liver. Laboratory findings of HH include increased levels of serum iron, serum ferritin, and serum transferrin. Ferritin helps determine a patient's iron stores and status. Ferritin is a fairly low cost test that is routine in many laboratories. A serum ferritin is typically ran on an automated chemistry analyzer.

An organism isolated from a sputum specimen and appeared as a dry, wrinkled, off-white colony on sheep blood agar. A Gram stain was performed and Gram positive branching rods were seen. In addition, a modified acid fast was performed and was positive. The organism was later identified as a Norcardia species. Which of the following antibiotics is the appropriate treatment? a. Sulfonamides b. Erythromycin c. Rifampin d. Streptomycin

A. Sulfonamides is the correct answer. Sulfonamides is a drug of choice in the treatment of Norcadia species infections. Other agents that can be used are amikacin, ceftriaxone, cefotaxime, linezolid, imipenem, and minocyline. The combination of a sulfa-containing agent and one of the primary agents is recommended for serious systemic Norcardia infections. Erythromycin and Rifampin are incorrect answers because both drugs are used in the treatment of Rhodococcus, Gordonia, and Tsuamurella species infections. Streptomycin is incorrect because this drug is used in the treatment of Streptomyces species infections.

An organism isolated from a sputum specimen and appeared as a dry, wrinkled, off-white colony on sheep blood agar. A Gram stain was performed and Gram positive branching rods were seen. In addition, a modified acid fast was performed and was positive. The organism was later identified as a Norcardia species. Which of the following antibiotics is the appropriate treatment? a. Sulfonamides b. Erythromycin c. Rifampin d. Streptomycin

A. Sulfonamides is the correct answer. Sulfonamides is a drug of choice in the treatment of Norcadia species infections. Other agents that can be used are amikacin, ceftriaxone, cefotaxime, linezolid, imipenem, and minocyline. The combination of a sulfa-containing agent and one of the primary agents is recommended for serious systemic Norcardia infections. Erythromycin and Rifampin are incorrect answers because both drugs are used in the treatment of Rhodococcus, Gordonia, and Tsuamurella species infections. Streptomycin is incorrect because this drug is used in the treatment of Streptomyces species infections.

This image was taken from a individual suffering from a severe organism-related anemia. Which organism could have caused this patient's anemia? a. Plasmodium sp. b. Ehrlichia sp. c. Bartonella sp. d. Escherichia sp.

A. The correct answer is Plasmodium sp. Since the image depicts intracellular ring-shaped red blood cell inclusions, the only choice that matches this morphology is Plasmodium species. Ehrlichia species is associated with an intracellular white blood cell parasitic inclusion. Bartonella species is associated with intracellular organisms inside red blood cells in single, chained, or paired morphologies. Escherichia species are gram negative rods that are typically extracellular gram negative rods that can cause sepsis, but are not generally associated with anemia.

The majority of all laboratory errors occur during which phase of laboratory testing? a. Preanalytic b. Analytic c. Postanalytic d. Errors are evenly distributed among preanalytic, analytic, and postanalytic phases of testing.

A. The correct answer is preanalytic. Studies indicate that the majority of all laboratory errors occur during the preanalytic phase of testing. Specimen handling and processing is a critical part of the testing process. Preanalytical errors can cause erroneous or misleading test results that may delay testing or give misleading results that will affect the patient's care. Some examples of preanalytic errors include the wrong test ordered, using an expired tube, misidentification of a patient, delay in transportation of sample, under filled sample tube, or using an incorrect needle size. Errors do occur in the analytic and postanalytic phases of testing, but the amount of errors in these phases pale in comparison to those identified in the preanalytic phase.

To determine if an elevated aPTT is caused by a factor deficiency or a factor inhibitor when the PT is normal, the FIRST step would be: a. Mix one part patient plasma and one part normal pooled plasma and repeat the aPTT b. Mix one part patient plasma and one part normal aged plasma and repeat the aPTT c. Perform a Factor IX assay to see if it is a Factor IX deficiency d. Perform a test for a lupus anticoagulant

A. The first step in this case is to begin the mixing study protocol where the patient sample is mixed with normal pooled plasma to determine whether or not there is a correction of the aPTT result upon re-running. If the specimen "corrects", a factor deficiency should be suspected. At this point, specific factor assays should be performed to identify and quantitate the specific factor deficiency. If the specimen does not "correct", a circulating factor inhibitor should be suspected and specific tests to identify the inhibitor should be performed. Mixing one part patient plasma with one part normal aged plasma may produce false results as time degrades coagulation proteins. Performing a Factor IX assay or lupus anticoagulant test should only be performed after identifying the cause of the increased aPTT.

What is the focus of both Lean principles and the Six Sigma methodology? a. Process improvement b. Quality control c. Reducing variation d. Reducing waste

A. The focus of both Lean and Six Sigma is process improvement. Quality control would be a Six Sigma focus, as would reducing variation. Six Sigma strategies seek to identify and eliminate causes of defects and errors. Lean principles focus on reducing waste, such as streamlining a process to reduce wait times.

The diagram indicates the banana-shaped gametocyte of the following Plasmodium species? a. Plasmodium falciparum b. Plasmodium ovale c. Plasmodium malariae d. Plasmodium vivax

A. The gametocytes of Plasmodium falciparum are the only malarial organisms that assume a characteristic banana shape. The gametocytes of Plasmodium ovale are round with dispersed chromatin. Banana shaped gametocytes are not produced. The gametocytes of Plasomodium malariae are round with compact chromatin. Banana shaped gametocytes are not produced. The gametocytes of Plasomodium vivax are round with dispersed chromatin. Banana shaped gametocytes are not produced.

The photograph on the right is of a 5% sheep blood agar plate inoculated with a throat culture. It is common practice to stab the agar with the inoculating needle, as seen at the tip of the arrow, in order to detect or accentuate the reaction of: a. Streptolysin O b. Streptolysin S c. Lecithinase d. Alpha prime hemolysin

A. The practice of stabbing the agar in areas of inoculation has the effect of submerging the growing bacteria under the surface of the agar, where conditions are relatively anaerobic. Oxygen labile hemolysin, or streptolysin O (SLO) is observed only under anaerobic conditions; therefore, will appear in the stab areas, accentuating the intensity of the hemolytic reaction as seen in this photograph. Streptolysin S (SLS) is the oxygen stable form of hemolysis seen around colonies when incubated aerobicaly. SLO and SLS are produced by Streptococcus pyogenes, the causative agent of strep throat. Lecithinase is an enzyme that can cause the hemolytic breakdown of erythrocytes in blood agar; however, appears as a broader zone of faint hemolysis concentric to the beta hemolytic zone, producing the so called double zone of hemolysis. Alpha prime hemolysis refers to a small zone of intact erythrocytes and non-hemolysis immediately adjacent to the colony, with the hemolytic zone outside this zone of intact erythrocytes, somewhat resembling a bulls eye.

The following characteristics are applicable to Interferon-Gamma Release Assays (IGRAs) except: a. Can be used in patients with active tuberculosis disease symptoms b. Preferable for patients who have received a bacillus Calmette-Guérin (BCG) vaccination c. Can be used instead of purified protein derivative (PPD) d. Cannot differentiate latent TB from active tuberculosis infection

A. Two currently approved commercially available Interferon-Gamma Release Assays (IGRAs) are approved by the Food and Drug Administration (QuantiFERON® and SPOT TB® test). Interferon-? release assays rely on the fact that T-lymphocytes will release IFN-? when exposed to specific antigens. Depending on which IGRA method is chosen either fresh whole blood or peripheral blood mononuclear cells (PBMCs) are used for testing. White blood cells from most persons that have been infected with M. tuberculosis will release interferon gamma (IFN-g) when mixed with antigens derived from M. tuberculosis. To perform an assay, fresh blood samples are mixed with antigens and controls. M. tuberculosis antigens mixture of synthetic peptides represent ESAT-6, CFP-10 and TB7.7 in QuantiFERON or ESAT-6 and CFP-10 in SPOT TB. The antigens, testing methods, and interpretation criteria for IGRAs differ. However, the IGRA method is not recommended for patients with active tuberculosis symptoms. In addition , an IGRA method should not be used in children younger than 5 years of age. Interferon-Gamma Release Assays (IGRAs) testing is preferable for patients who have received a bacillus Calmette-Guérin (BCG) vaccination (either as a vaccine or for cancer therapy). Prior BCG vaccination does not cause a false-positive IGRA test result. Interferon-Gamma Release Assays (IGRAs) can be used instead of purified protein derivative (PPD) skin test. It is recommended over tuberculosis skin testing in individuals 5 years and older and who have low or moderate risk of disease progression. Interferon-Gamma Release Assays (IGRAs)cannot differentiate latent tuberculosis from active tuberculosis infection.

All of the following statements are true regarding the reagent strip test procedure, EXCEPT? a. Urine should be centrifuged prior to dipping the reagent strip. b. Urine should be well mixed prior to dipping the reagent strip. c. When visually reading the reagent strip, the results should be read at the prescribed time suggested by the manufacturer after dipping the strip in the urine specimen. d. Prolonged immersion may wash out test reagents.

A. Urine should NOT be centrifuged prior to dipping the reagent strip. Urine should be well mixed prior to dipping the reagent strip. Prolonged immersion may wash out test reagents. When visually reading the reagent strip, the results should be read at the prescribed time suggested by the manufacturer after dipping the strip in the urine specimen.

Which values would be expected with metabolic alkalosis? a. Elevated pH, increased bicarbonate, normal PCO2 b. Elevated pH, decreased bicarbonate, normal PCO2 c. Lowered pH, normal bicarbonate, decreased PCO2 d. Increased pH, normal bicarbonate, increased PCO2

A. When using determining the acid-base status of a person, the parameters that may vary due to conditions of the patient would be the pH (acidosis = < 7.35 or alkalosis = > 7.45), bicarbonate (HCO3-1), and PCO2 (partial pressure of carbon dioxide).If the bicarbonate changes, it would be considered metabolic. If the bicarbonate increases, the pH increases and it would be metabolic alkalosis. For example, this could happen through excess administration of sodium bicarbonate.If the bicarbonate decreases, the pH decreases and this would be metabolic acidosis. For example, excessive loss from diarrhea.If the carbon dioxide changes, it would be considered respiratory. A decrease in PCO2 would actually increase the pH and it would be respiratory alkalosis which could be caused by hyperventilation conditions.An increase in PCO2 would actually decrease the pH and it would be respiratory acidosis which could be caused by hypoventilation conditions.

This Gram-positive coccus is catalase positive. The tests shown are a tube coagulase test inoculated with the isolate, and a MRSA screen plate containing 4% NaCl and 6 µg/mL of oxacillin. A 0.5 McFarland standardized inoculum of the isolate is inoculated to the plate. What step should you take next? a. Disregard the oxacillin test because coagulase negative staphylococci should not be tested on the oxacillin screen plate. b. Report the isolate as methicillin resistant Staphylococcus aureus. c. Report the isolate as methicillin susceptible Staphylococcus aureus. d. Test the isolate for cefoxitin susceptibility by disk diffusion testing.

B. "Report the isolate as methicillin resistant Staphylococcus aureus" is the correct answer since the coagulase plasma is clotted indicating it is coagulase positive and the organism is growing on the 4% NaCl and 6 µg/mL oxacillin MRSA screen plate, demonstrating resistance to oxacillin, methicillin, and nafcillin. It is true that coagulase negative staphylococci should not be tested on MRSA screen agar but this isolate is coagulase positive. Methicillin susceptible Staphylococcus aureus would not grow on the MRSA screen plate. This test is as confirmatory as cefoxitin disk diffusion testing, so further testing is not needed and would delay treatment.

Remnants of erythrocyte nuclei, nuclear fragments, or aggregates of chromosomes are called: a. Heinz bodies b. Howell-Jolly bodies c. Basophilic stippling d. Pappenheimer bodies

B. A Howell-Jolly body is composed of DNA, usually left from the nucleus of an immature red blood cell. It appears as a round, dark-staining inclusion in the cytoplasm of red blood cells. Howell-Jolly bodies can be found in various conditions including splenectomy and anemia. Heinz bodies are precipitated unstable hemoglobin that can be stained with a supravital stain. Basophilic stipplings are aggregated ribosomal RNA, often seen in lead poisoning. Pappenheimer bodies are iron inclusions that should be confirmed with an iron stain (Prussian blue).

A spinal fluid that is slightly hazy is briefly examined microscopically. The technologist performing the count decides to make a 1:10 dilution using 30 µL of sample. What volume of diluent should be used? a. 70 µL b. 270 µL c. 300 µL d. 30 µL

B. A simple 1:10 dilution, as this is the case, uses the formula: sample volume/(sample volume + diluent volume) = 1/10 or sample volume x 10 = sample volume + diluent volume 30 µL x 10 = 30 µL + diluent volume diluent volume = 300 µL - 30 µL Diluent volume = 270 µL So, to make a 1:10 dilution, 270 µL of diluent should be added to 30 µL of sample, .

When can a doctor order a panel made up of tests he/she specifically wants? a. Any time they want. b. Only if the laboratory ensures the ordering physician knows what tests are included in the panel or profile and has the appropriate procedure and diagnostic codes for each test. c. If approved by a department head. d. If the panel is used by the doctor at another laboratory and they have approved it.

B. According to Medicare compliance standards, doctors may order panels composed of tests they specifically want only if the laboratory ensures the ordering physician knows what tests are included in the panel or profile and has the appropriate procedure and diagnostic codes for each test. From the HHS-Model Compliance Plan for Clinical Laboratories: c. Physician Acknowledgments: Laboratories that agree to customize profiles in response to physician requests should require such requesting physicians to sign a Physician Acknowledgment. By signing the Physician Acknowledgment, the physician would affirm that: (1) the physician has requested the creation of a custom profile that includes the tests listed on the acknowledgment; (2) the physician has been informed of the reimbursement amount that Medicare (and where appropriate, Medicaid) will pay for each test included in each customized profile; (3) the physician understands that when ordering tests for which Medicare reimbursement will be sought, the physician should only order those tests which the physician believes are medically necessary for each patient; (4) the physician knows that using a customized profile may result in the ordering of tests for which Medicare or other federally funded health care programs may deny payment; (5) the physician will order individual tests or a less inclusive profile when not all of the tests included in the customized profile are medically necessary for an individual patient; (6) the physician has been informed that the OIG takes the position that a physician who orders medically unnecessary tests may be subject to civil penalties; and (7) if appropriate, the physician is aware that the laboratory makes available the services of a clinical consultant to assist the physician in ensuring that appropriate tests are ordered.

A 41-year-old male arrived at the clinic with complaints of fatigue, malaise, and overall discomfort. Upon testing, the patient's CBC results were notably abnormal with 30% blasts present along with anemia and thrombocytopenia. Bone marrow and cytochemical staining were subsequently performed. The cytochemical stains noted negative reactions to Myeloperoxidase and Sudan Black B but a positive reaction to alpha-naphthyl acetate. Which of the following conditions is most likely seen? a. Acute Erythroleukemia b. Acute Monocytic Leukemia c. Acute Promyelocytic Leukemia d. Acute Lymphocytic Leukemia

B. Acute Monocytic Leukemia is most likely present due to the positive reaction of alpha-naphthyl acetate stain. This is a type of nonspecific esterase stains which are known to positively identify monoblastic cells. Specific esterase stains would stain myelocytic cells positive. Both Acute Erythroleukemia and Acute Lymphocytic Leukemia would stain negative with all three stains mentioned in the case. Acute Promyelocytic Leukemia would stain positive with Myeloperoxidase and Sudan Black B, but would stain negatively with alpha-naphthyl acetate.

How many white blood cells (WBCs) would be considered normal for adult cerebrospinal fluid? a. Any number of WBCs is considered abnormal b. 0-5 WBCs/µL c. 6-10 WBCs/µL d. Up to 30 WBCs/µL

B. In an adult, 0 - 5 WBC/µL is considered normal. Children will have slightly higher cell counts. Up to 30 WBC/µL is within normal limits for newborns. Lymphocytes account for 60 - 100% of these cells.

Which of the following activities may lead to a HIPAA violation lawsuit? a. Faxing patient health information (PHI) to the patient's physician who order the laboratory tests. b. Throwing laboratory test requests into the regular trash. c. Discussing patient information with a patient's family member with the patient's permission. d. Logging off the computer after entering blood collection updates.

B. Although innocent in nature, throwing laboratory test requests into the regular trash; may lead to a HIPAA violation lawsuit. The regulations do not prevent faxing of PHI. Covered entities must have appropriate policies, procedures and processes in place to make faxing of PHI as secure as possible. Healthcare professionals should get prior permission before discussing the patient's health with family members to avoid a lawsuit. Logging off the computer after entering blood collection updates should not result in a HIPAA violation lawsuit.

Which of the following group B antigens is generally associated with a mixed field reaction? a. B b. B3 c. Bm d. Bx

B. B3 is characterized by a weaker than usual reaction with anti-B and by a mixed field reaction with the same reagent. B is characterized by a strong reaction with anti-B. Bm and Bx are characterized by a weaker than usual reaction with anti-B but no mixed field.

Chemicals such as sodium hydroxide and sulfuric acid should be labeled: a. Carcinogenic b. Corrosive c. Biohazard d. Irritant

B. Both sodium hydroxide and sulfuric acid are corrosive agents. Carcinogenic agents have the potential to cause cancer. An example of a carcinogenic agent is benzene. Biohazard agents have the potential to cause disease or infection. An example of a biohazard agent is Streptococcus pneumoniae. An agent is considered an irritant when it is a chemical, which is not corrosive, but which causes a reversible inflammatory effect on living tissue by chemical action at the site of contact. An example of an irritant is ammonia.

The most highly specific indicator for rheumatoid arthritis is: a. Rheumatoid factor b. Cyclic citrullinated peptide antibodies (CCP antibodies) c. Immune complexes d. Complement

B. CCP antibodies=RA CCP antibodies are a highly specific indicator for RA. Antibodies to CCP-1 (anti-CCP-1) were first described in 1998 followed by the introduction of commercial ELISA methods using the second-generation peptides (CCP-2). Anti-CCP IgG antibodies are present in about 69% to 83% of patients with RA and have specificities ranging from 93% to 95%. Compared to other assay methods for RF, CCP is considered to be more sensitive (specificity >95%). Citrullinated tenascin-C (cTNC) is a newer glycoprotein with an 98% accuracy rate for ruling out RA. Agglutination tests for RF, generally detects IgM RFs. Latex agglutination is sensitive but can produce a fairly high number of false-positive results. Because conventional procedures are semi-quantitative, they may be insensitive to changes in titer and may detect only those RFs that agglutinate. Soluble circulating immune complexes and cryoprecipitable proteins consisting of immunoglobulins, complement components, and RFs are demonstrable in the sera of some patient with RA. Complement levels are generally normal in patients with rheumatoid arthritis, except for patients with vasculitis.

A manual white blood cell count was performed by the hematology technologist. The cell counts for both sides were 152 and 164 respectively. All nine large squares were counted on each side. The dilution for this kit was pre-measured at 1:100. What should the technologist report as the white cell count? a. 177.5 x 109/L b. 17.5 x 109/L c. 1.75 x 109/L d. 175 x 109/L

B. Calculation: Cells Counted (in this case the average of both sides) X dilution factor (in this case 100) / # of squares counted (in this case 9) X area of each square (1 mm2) X 0.1 mm (depth factor) So, in this problem: 158 x 100 / 9 x 1 x 0.1 mm = 17555.55/mm3 (can be converted to 17.5 x 109/L*) *There are 1,000,000 mm3 in a liter (L). So 17555.55 X 1,000,000 = 17.5 x 109/L *knowing the ranges will help with these problems

Human dog bite infections may be caused by each of the following bacterial species except: a. Pasteurella multocida b. Capnocytophaga ochracea c. Capnocytophaga cynodegmi d. Neisseria weaveri

B. Capnocytophaga ochracea has its natural habitat in the oropharynx of humans and would not be involved in dog bite infections. All of the other bacterial species listed in this question reside in the oropharynx of dogs and other canines, and may cause human dog bite infections.

Which of the following statement is INCORRECT regarding the bone marrow biopsy specimen? a. Bone marrow cellularity can be accurately determined b. Cellular morphologic detail can be determined. c. Bone marrow architecture can be examined d. Prussian blue staining could be used to evaluate iron stores, and diagnosis of anemia.

B. Cellular morphologic details CANNOT be determined via bone marrow biopsies. Bone marrow cellularity can be determined accurately by comparing the amount of hematopoietic tissue with the amount of adipose tissue. Bone marrow core biopsy could be examined to reveal bone marrow architecture. Estimation of quantity of iron and its distribution is part of bone marrow examination. The Prussian blue stain provides the most direct means of assessing body iron stores.

Which lipid does not serve as a source of fuel, but can be converted to steroid hormones? a. Fatty acids b. Cholesterol c. Triglycerides d. Phospholipids

B. Cholesterol is an unsaturated steroid alcohol. Cholesterol is unique in that, unlike other lipids, it is not readily catabolized by most cells, and therefore, does not serve as a source of fuel. A small amount of cholesterol can also be converted by some tissue, such as the adrenal gland, testis, and ovary to steroid hormones such as glucocorticoids, mineralocorticoids, and estrogens. The majority of fatty acids are found as a constituent of triglycerides or phospholipids. Triglycerides contain three fatty acid molecules attached to one molecule of glycerol by ester bonds. Phospholipids are similar in structure to triglycerides except that they only have two esterified fatty acids.

Which of the following is the bacterial agent associated with antibiotic-associated pseudomembranous colitis? a. Campylobacter jejunii b. Clostridium difficile c. Shigella dysenteriae d. Bacillus cereus

B. Clostridium difficile is the causative agent of antibiotic-associated pseudomembranous colitis. The diagnosis of this infection is based on clinical symptoms and the detection of a C. difficile cytotoxin. Campylobacter jejuni produces a characteristic bloody diarrhea with white blood cells after the consumption of infected food sources. Culture detection is sufficient for diagnosis. Shigella dysenteriae produces dysentery with white blood cells. It is associated with contaminated water. Bacillus cereus produces watery diarrhea with white blood cells and vomiting. The infection is associated with the consumption of a pre-formed toxin from a food source such as meats, vegetables, and rice.

The following organism is an anaerobic organism that produces spores and a double-zone of beta-hemolysis. a. Pseudomonas aeruginosa b. Clostridium perfringens c. Enterobacter aerogenes d. Cutibacterium (Propionibacterium) acnes

B. Clostridium perfringens is an anaerobic organism that is known to cause gas gangrene, food poisoning, and necrotizing enteritis. The identification of this organism begins with the appearance of a Gram-positive, spore-forming bacillus that produces a double-zone of beta-hemolysis on blood agar. Pseudomonas aeruginosa is a Gram-negative bacillus that produces beta-hemolysis but it is not a double-zone of beta-hemolysis. This organism is frequently found in water sources including environmental and hospital environments. Many of the opportunistic infections are related to exposure to water such as the community-acquired 'swimmer's ear' (otitis externa) and hospital-acquired respiratory infections. Enterobacter aerogenes is a Gram-negative bacillus that produces no hemolysis on blood agar. It is an opportunistic pathogen associated frequently with contaminated medical devices such as respirators. Cutibacterium (Propionibacterium) acnes is an anaerobic Gram-positive bacillus but it does not produce any hemolysis on blood agar. Although it is known to cause acne and endocarditis, it is the most common anaerobic skin contaminate for blood cultures.

Which of the following tests would be used in the assessment of glomerular filtration? a. 24 hour urine protein b. Creatinine clearance c. PSP test d. Urea

B. Creatinine excretion is related to muscle mass and is fairly constant from day to day for a given individual. Plasma concentrations of creatinine are used to assess renal function. Creatinine clearance is based on the serum creatinine level and is used to measure glomerular filtration rate, or GFR. Creatinine is filtered by the glomerulus and is not reabsorbed; therefore it can be used to estimate the GFR.

Direct examination of treponemes can be performed by: a. Light microscopy examination b. Darkfield microscopy examination c. VDRL testing d. Rapid Plasmin Reagin (RPR) testing

B. Darkfield microscopy allows for direct observation of spirochetes from an active syphilitic lesion. It is the test of choice for examining symptomatic patients with primary syphilis. A darkfield examination is also suggested for immediate results in cases of secondary syphilis. The traditional logarithm for syphilis testing of suspected patients with syphilis begins with observations of classic lesions present, suggestive sexually transmitted infection history, and current sexually transmitted disease. Non-treponemal testing such as the Rapid Plasmin Reagin (RPR) is the next step. A reverse sequence testing algorithm begins with the same patient observations but the next step is to perform treponemal testing by chemiluminescence immunoassay or enzyme immunoassay. Light microscopy examination is not effective in the detection of spirochetes. Darkfield or fluorescent microscopy techniques are required for direct observation of spirochetes. The VDRL assay is a flocculation test that measures IgM and IgG antibodies to lipoidal material released from damaged host cells, to lipoprotein-like material, and possibly to cardiolipin released from the treponemes. Without some other evidence for the diagnosis of syphilis, a reactive nontreponemal test does not confirm T. pallidum, infection. Antilipoidal antibodies are antibodies that are not only produced as a consequence of syphilis and other treponemal diseases, but also may be produced in response to nontreponemal diseases of an acute or chronic nature in which tissue damage occurs. Rapid Plasmin Reagin (RPR) testing is a non-treponemal testing method. Traditionally, it is the next step in serological assessment. The RPR is the most widely used assay to determine the presence of regain, an antibody formed against cardiolipin, a lipid remnant of damaged cells, cholesterol, and lecithin, is used to detect the nontreponemal regain antibodies.

What is the BEST technique for monitoring the severity of hemolytic disease of the fetus and newborn (HDFN)? a. Amniocentesis b. Doppler ultrasound c. Antibody titration d. Cordocentesis

B. Doppler ultrasonography can measure blood flow velocity, which allows for the severity of fetal anemia to be determined without invasive methods. This creates a safer alternative than amniocentesis for fetal monitoring and has largely replaced serial amniocentesis for predicting severity of HDFN. Amniocentesis is a useful method that can demonstrate the severity of HDFN, but the process is invasive and, therefore, not as safe to perform as Doppler ultrasonography. Antibody titration is useful for determining when other procedures, such as Doppler ultrasonography, should be performed. However, the cutoff level of antibody is not standardized among facilities, and the antibody titer does not provide a definitive correlation with HDFN severity. Cordocentesis is an invasive method that is more difficult and dangerous to perform than amniocentesis because it collects a fetal blood sample. Because of this, Doppler ultrasonography is a less invasive method.

Illustrated in the photograph is a close in view of a 70 µm oval ovum with a thin, smooth outer shell and an inner membrane containing three pairs of hooklets (hexacanth embryo). This ovum is indigenous primarily in rats and mice and only rarely found in human feces. When found, the infected person usually has minimal symptoms; however, in heavy infections, particularly in those who are immune-suppressed, abdominal pain, diarrhea and anal pruritis may be experienced. What is the best presumptive identification of this tapeworm ovum? a. Hymenolepis nana b. Hymenolepis diminuta c. Taenia saginata d. Taenia solium

B. Hymenolepis diminuta is the correct response. H. diminuta is an uncommon tapeworm in humans and is typically found in rodents. It can infect humans after contamination of grains or flours with rat feces. Symptoms may include diarrhea, anorexia, nausea, headache, and dizziness. The characteristics of the ovum shown in the photo-micrograph include its relative large (70 µm) size and an oval outline. A thin smooth shell surrounds an inner -hexacanth membrane containing three pairs of hooklets. The absence of bipolar filaments on either side of the membrane is distinctive for H. diminuta. Hymenolepis nana is an incorrect response. Ova may appear similar to those of H. diminuta, but are smaller in size, averaging 35 µm, and more distinctly possess a pair of bipolar filaments extending from each side of the inner hexacanth membrane. Taenia saginata is an incorrect response. Ova identical to those of Taenia solium are suggestive of the ova of Hymenolepis species by also possessing three pairs of internal hooklets. The lack of an inner hexicanth membrane and an outer shell that is thick with distinctive radial striations are the exclusive characteristics. Taenia solium is an incorrect response. Taenia ova possess three pairs of internal hooklets but the lack of an inner hexicanth membrane and an outer shell that is thick with distinctive radial striations are the exclusive characteristics.

All of the following examples require biohazard labeling except? a. Sharps containers for contaminated waste b. Individual specimen containers c. Contaminated waste bags d. Transport containers for individual specimens

B. Individual specimen containers do not require biohazard labeling, however, contaminated waste bags, sharps containers for contaminated waste, and transport containers for individual specimens should always have biohazard labeling. According to OSHA, labeling is not required for: Regulated waste that has been decontaminated. Containers of blood, blood components, and blood products bearing an FDA required label that have been released for transfusion or other clinical uses. Individual containers of blood or OPIM that are placed in secondary labeled containers during storage, transport, shipment, or disposal. Specimen containers, if the facility uses Universal Precautions when handling all specimens, the containers are recognizable as containing specimens, and the containers remain within the facility.

Illustrated in the upper photograph to the right are small, gray-white to light yellow colonies surrounded by zones of beta hemolysis. Short, gram-positive bacilli with rounded ends that lie singly or in loose clusters are observed in gram stains. Two key identifying features, as illustrated in the lower photograph, are a narrow band of "umbrella" motility observed beneath the surface of semi-solid motility medium and the deep black slant of positive esculin activity. This species may be recovered during influenza-like infections during 2nd and 3rd trimester of pregnancy. With these observations, which of the following organisms is the identification of this isolate? a. Rothia dentocariosa b. Listeria monocytogenes c. Arcanobacterium haemolyticum d. Erysipelothrix rhusiopathiae

B. Listeria monocytogenes is the correct response. Colonies on blood agar are initially small, gray to yellow-white, surrounded by narrow zones of beta hemolysis that do not extend far beyond the outer margins. Microscopically, small gram-positive bacilli with rounded ends lying singly or in loose clusters are observed. In older cultures, the bacilli may arrange in Chinese letter formations suggesting diphtheroids. "Umbrella" motility is observed on semi-solid motility media (image to the left in the composite); esculin hydrolysis is positive (image to the right). Rothia dentocariosa is incorrect because colonies are bone-white to gray and often appear heaped and cerebriform characteristically in a "spoke wheel" pattern, particularly in areas of confluent growth. Hemolysis is not observed. Gram positive bacilli or cocco-bacilli are observed in diphtheroidal arrangements. Motility is negative, although esculin hydrolysis is also positive. Arcanobacterium haemolyticum is incorrect. Colonies are not discriminatory from L. monocytogenes because the colonies are small, beta hemolytic and, microscopically, gram-positive bacilli appear branching and in Chinese letter formations. However, to separate this species from Listeria monocytogenes, motility and esculin reactions are negative, and a positive reverse CAMP reaction is distinctive. Arcanobacterium haemolyticum causes pharyngitis and cellulitis. Erysipelothrix rhusiopathiae is incorrect because colonies are relatively small, 1 - 2 mm in diameter, convex, entire, and circular with light gray pigmentation. Beta hemolysis is absent. Slender gram positive bacilli or cocco-bacilli are observed microscopically, arranged singly or in short chains and devoid of spores. "Bottle brush" motility may be observed in semi-solid agar, but esculin hydrolysis is negative.

What is a lyophilized control? a. A control that does not resemble human blood. b. A control that is dehydrated to powder. c. A control that is outside of the expected range. d. A control that is made directly by the laboratory.

B. Lyophilized control materials have been dehydrated to powder, by the manufacturer, for stability and can be reconstituted in specific diluents or matrices. These matrices represent urine, blood, or cerebral spinal fluid. The expected values are acceptable values within the control limits. When the observed values fall outside the control limits, the lab tech must be aware of possible problems and need for further investigation before reporting potentially erroneous patient results. Today, laboratories more often purchase manufactured control materials for QC, instead of preparing the materials themselves.

The late steps of complement activation and formation of the "membrane attack" complex (MAC) results in: a. Immune complex removal b. Lysis of cells c. Opsonization in phagocytosis d. Polymorphonuclear (PMN) leukocyte activation

B. Lysis of cells is the result of the late steps of complement activation and formation of the "membrane attack" complex (MAC). When fully assembled in the correct proportion, C7, C6, C5b, and C8 form the MAC. The C5bC6C7C8 complex polymerizes C9 to form a tubule (pore) which spans the membrane of the cell being attacked, allowing ions to flow freely between the cellular interior and exterior. By complexing with C9, osmotic cytolysis Removal of an immune complex from tissues occurs with the activation of classic complement components C1q and covalently bonded fragments of C3 and C4. Opsonization in phagocytosis is facilitated by covalently bonded fragments of C3 and C4. Activation of polymorphonuclear (PMN) leukocytes is facilitated by C5a, C3a, and C4a.

A cause of thrombocytopenia due to decreased production of platelets is: a. Von Willebrand's disease b. Malignant infiltration of the bone marrow c. Glanzmann's thrombasthenia d. High doses of aspirin

B. Malignant infiltration of the bone marrow can result in thrombocytopenia because of insufficient space in the bone marrow for megakaryocyte production. Von Willebrand's disease, Glanzmann's Thrombasthenia, and aspirin all are disorders of platelet function, not platelet production.

Which of the following steps is the first step of the ethyl acetate concentration procedure? a. Centrifuge specimen to obtain four layers b. Mix fresh stool specimen with 10% formalin and strain to obtain a sediment c. Add normal saline, centrifuge, and decant the supernatant fluid d. Resuspend the sediment with 10% formalin and add ethyl acetate

B. Mix the fresh stool specimen with 10% formalin and strain to obtain a sediment is the correct answer because the purpose of the ethyl acetate concentration procedure is to remove fecal debris. Filtering the specimen removes large pieces of debris. (First Step) Add normal saline, centrifuge, and decant the supernatant fluid is incorrect because this is the washing step of the procedure to remove smaller pieces of debris. (Second Step) Resuspend the sediment with 10% formalin and add ethyl acetate is incorrect because this is the preservation step of the procedure. Ethyl acetate is added to introduce specific gravity differences into the testing environment. (Third Step) Centrifuge specimen to obtain four layers is incorrect because this is the last step in this procedure. By centrifuging the specimen after the addition of ethyl acetate, the parasites are heavier and will sink to the bottom of the tube. Formalin layer will be above the sediment. The debris/fat layer will be above the formalin, and the ethyl acetate layer will be at the top. (Last Step)

Carbapenemases are produced by various organisms such as Enterobacteriaceae, Pseudomonas aeruginosa, and Acinetobacter species. Detecting the presence of carbapenemase activity in these organisms is important to prevent spreading of the resistant organisms. Which of the following methods is used to detect the presence of carbapenemase producing organisms? a. D test b. Modified Hodge Test c. ESBL Test d. Beta-lactamase test

B. Modified Hodge Test is the correct answer because the presence of a carbapenemase can be detected by this method. This method is also referred to as the clover-leaf test. This test uses a carbapenem-susceptible Escherichia coli that is plated to a Mueller-Hinton agar and then carbapenem disks are placed onto the plate. The test organisms are inoculated in a straight line from the disks and growth of the Escherichia coli around the test organism at the intersection of the streak and the zone of inhibition indicates the presence of a carbapenemase. D test is incorrect. The D test is used to detect inducible clindamycin resistance by plating the test organism on a Mueller-Hinton plate and then placing erythromycin and clindamycin disks. If a D zone of inhibition is present after a 24 hour incubation at 37° C, the organism has an inducible resistance to clindamycin, due to the erm gene. ESBL Test is incorrect. This test detects organisms that produce extended spectrum beta-lactamases to the cephalosporin class of antibiotics. To detect an ESBL producing organism, antibiotic disks cefotaxime and cefotaxime-clavulanate and ceftaxidime and ceftaxidime-clavulanate are placed on a Mueller-Hinton agar plated with the test organism. If there is a zone difference between the antibiotic with and without the clavulanate, then the presence of an ESBL is indicated. Beta-lactamase test is incorrect. This test is a rapid test to detect penicillin resistance. Beta-lactamase breaks down the beta-lactam ring found in penicillin.

An acid-fast bacillus recovered from a post-surgery wound of a young child had the following characteristics: Rapid growth (3-5 days) on blood and chocolate agar Nitrate positive Niacin negative Urease positive Catalase 68ºC positive Iron uptake positive Which of the following Mycobacterium species does this represent? a. Mycobacterium tuberculosis b. Mycobacterium fortuitum c. Mycobacterium kansasii d. Mycobacterium avium-intracellulare

B. Mycobacterium fortuitum is correct because Mycobacterium fortuitum demonstrates rapid growth, is nitrate positive, niacin negative, and urease positive, while demonstrating a positive iron uptake test. M. fortuitum is known to grow on routine bacteria media. Mycobacterium tuberculosis is incorrect because Mycobacterium tuberculosis is niacin positive, catalase negative, and iron uptake negative. In addition, M. tuberculosis is not a rapid grower. Mycobacterium kansasii is incorrect because Mycobacterium kansasii is niacin negative, nitrate positive, and a photochromogen. Mycobacterium avium-intracellulare is incorrect because it is niacin and nitrate negative. In addition, M. avium-intracellulare is a nonphotochromogen.

Spectrophotometric absorbance is related to % transmittance in what way: a. Inversely and arithmetically b. Inversely and logarithmically c. Directly and arithmetically d. Directly and logarithmically

B. Particles in a solution absorb certain wavelengths while they transmit others. The color red we see in a solution for example, appears red to the human eye because the particles in the solution absorb all the wavelengths except red, which is transmitted and seen. As the concentration of particles in a solution increases, the amount of absorbed light increases as well while the percentage of light transmitted decreases. Absorbance (A) is the inverse log of transmitted (T) light. A = log(1/T) or A = -log(T)

If a phlebotomist is failing to invert whole blood specimen 5-10 times immediately after filling the tube, which of the following pre-analytical errors can occur? a. Hemoconcentration b. Microclots c. Hemolysis d. Lipemia

B. Pre-analytic and hidden errors can seriously affect laboratory results. Tubes shaken vigorously can break cells causing hemolysis; delayed mixing of tubes can cause microclots to form; a tourniquet in place for an extended period of time can result in hemoconcentration. Lipemia is a result of the diet of the patient, not an error created by the phlebotomist. If blood is to be collected from a patient who has fasted, the phlebotomist should ask if the patient has had any food in the past 8 hours. If the patient answers that they have, then tests may be cancelled.

Which of the following cells when found upon microscopic examination of the urine would be most indicative of kidney disease: a. WBCs and bacteria b. Tubular epithelial cells c. Squamous epithelial cells d. RBCs

B. Renal tubular epithelial cells may be found rarely in healthy patients, but in increased numbers, can indicate destruction of the kidney's tubules (nephrons). These cells are small and round. Conditions associated with the presence of these cells include: allograft rejection, viral infection, and other less common conditions. WBCs and bacteria may be indicative of a urinary tract infection. Squamous epithelial cells are the largest cells found in urine sediment and are normally shed from the lower linings of urethra. They typically do not have clinical significance. RBCs are seen due to vascular injury withing the GI tract or due to damage to the glomerular membrane.

The type of sporulation of the dematiaceous mold that is illustrated in this photomicrograph is called: (Picture shows a flower arrangement almost) a. Acrotheca b. Rhinocladiella c. Birds on a fence d. Acropetal

B. Rhinocladiella is the correct answer because production of single conidia in succession both laterally and around the tip of a straight phialide is called the rhinocladiella type of sporulation, characteristic of Fonsecaea pedrosoi. Acrotheca type sporulation is incorrect because this type of sporulation is characterized by the production of short chains of elliptical conidia in a circular arrangement from the tips of branching phialides. Birds on a fence is incorrect because this type of arrangement of microconidia is characteristic of Trichophyton rubrum. This type of sporulation is somewhat reminiscent of the rhinocladiella type sporulation except that the conidia are more loosely held and alternate more from one side of the hyphae to the other. (Trichophyton rubrum is a hyaline rather than a dematiaceous fungus.) Acropetal is incorrect because this term refers to a type of sporulation where chains of conidia are formed with each new daughter cell produced from the previous one, leaving the oldest cell at the base of the chain. This type of sporulation is characteristic of Aspergillus species and Penicillium species.

The average bone marrow cellularity in a normal adult (50 years old) is: a. 30 b. 50 c. 65 d. 70

B. The overall bone marrow cellularity is estimated by comparing the amount of hematopoietic tissue with the amount of adipose (fat) tissue. An easy way to calculate it is to subtract the age from 100 and adds plus or minus 10. In this example 100 minus 50 equal 50. So the normal bone marrow cellularity is between 40 and 60 30 as a result is too low 65 and 70 are too high. Formula for cellularity: 100-pt age

This suspicious form measures 18 µm and was seen in ova and parasite exam from a stool specimen. The most likely identification is: (Trophozoite with bloat central karyosome) a. Chilomastix mesnili b. Iodamoeba butschlii c. Pseudoparasite d. Dientamoeba fragilis

B. The Iodamoeba butschlii cyst is an amoeba characterized by its size (8-20µm), shape, a single nucleus with a large karyosome and absence of peripheral chromatin. The trophozoite may also contain a glycogen-filled vacuole, a characteristic also helpful in identifying the organism. Chilomastix mesnili is a lemon-shaped flagellate. The flagella may not be visible on stain specimens. The cyst measures 6-10 µm in length. A single nucleus and a distinct oral groove (cytostome) that is curved giving the appearance of a "shepherd's hook" is also visible. This picture is a characteristic of Iodamoeba b?tschlii with distinct features such as a visible karyosome and no peripheral chromatin. It is not a pseudoparasite. Dientamoeba fragilis is a flagellate that does not have a cyst stage. The trophozoite is described as 5-15 µm, 1-2 nuclei with a karyosome composed of a cluster of granules, and no peripheral chromatin.

Hydrogen ion concentration (pH) in blood is usually determined by means of which of the following electrodes? a. Silver b. Glass c. Platinum d. Platinum-lactate

B. The active part of a pH measuring electrode is the glass tip which senses the hydrogen ion concentration. Usually the glass is composed of amorphous silicon dioxide embedded with sodium ions coated in a special a gel layer. When the surface of the glass is exposed to water, the alkali metal ions of the glass and the hydrogen ions (H+) in the solution undergo an ion exchange reaction. Depending on the pH of the solution being measured, hydrogen ions (H+) will migrate into or out of the gel layer. In an alkaline solution, hydrogen ions migrate out of the gel layer and a negative charge is developed on the outer gel layer. Hydrogen ion does not cross through the glass membrane, it is the Na+ which crosses and allows for a change in free energy. It is important to remember that the glass tip of the electrode must be hydrated, so a pH electrode needs to be kept moist at all times.

Twenty microliters (20 µL) of blood are diluted with 1.98 mL of diluent. Both sides of the hemocytometer are charged with the diluted sample and the entire ruled area of the center square is counted on both sides. A total of 356 platelets are counted (combined total of the two large squares). What is the platelet count per liter? a. 100 x 109/L b. 178 x 109/L c. 356 x 109/L d. 712 x 109/L

B. The correct answer is 178 x 109/L. 20 µL is equal to 0.02 mL. Therefore, the total volume of the dilution is 2.00 mL and the dilution is 1:100. The standard hemocytometer formula is: # of platelets counted x the dilution = platelets/µL number of squares counted x 0.1 (0.1 represents the depth of the chamber) The formula can also be simply stated as: # of platelets counted x dilution x 10 (depth) = platelets/µLnumber of squares counted 356 x 100 x 10 = 178 x 103 platelets/µL or 178 x 109 platelets/L2

_______________ is predominantly associated with skin and soft tissue infections (SSTIs), such as abscesses, cellulitis, folliculitis and impetigo. a. Healthcare-associated MRSA b. Community-acquired MRSA c. Escerichia coli d. Neisseria meningitidis

B. The correct answer is community-acquired MRSA. Community-acquired MRSA infections are predominantly skin and soft tissue infections (SSTIs), such as abscesses, cellulitis, folliculitis, and impetigo. Outbreaks can occur that are typically seen in communities such as prisons, day care centers, and military recruits. Incidence of infection in pediatric patients is increasing due to these outbreaks. The strain responsible for these infections is called USA300 MRSA and is now being associated with significant bacteremia as well. Healthcare-associated MRSA occur mostly in the elderly population and include infections such as bacteremia and pneumonia. Escherichia coli is able to cause abscesses, cellulitis, folliculitis and impetigo, but it is most commonly associated with urinary tract infections (UTI) Neisseria meningitidis does not typically cause abscesses, cellulitis, folliculitis or impetigo. It is most commonly associated with bacterial meningitis.

All of the following observations are consistent with the laboratory identification of Cryptococcus neoformans, EXCEPT: a. Colonies that appear mucoid on primary isolation culture medium. b. Negative urease test. c. Cornmeal agar morphology displaying spherical, irregular cells with no pseudohyphae. d. Presence of capsular material.

B. The correct answer is negative urease test. Cryptococcus neoformans would produce a rapidly positive urease test. All the remaining observations are consistent with the laboratory identification of C. neoformans. Upon observing mucoid appearing colonies on a primary isolation culture medium, perform a rapid urease test and set up a cornmeal agar preparation. If the urease test is rapidly positive, observe the cornmeal agar morphology. If pseudohyphae are absent, and particularly if the yeast cells are spherical, irregular in size and widely separated (presence of capsular material), inoculate the surface of a bird seed agar plate with a small portion of the unknown colony. If a brown pigment is produced within 48-72 hours, a definitive identification of Cryptococcus neoformans can be made.

All of the following observations are consistent with the laboratory identification of Cryptococcus neoformans, EXCEPT: The correct answer is highlighted below a. olonies that appear mucoid on primary isolation culture medium. b. Negative urease test. c. Cornmeal agar morphology displaying spherical, irregular cells with no pseudohyphae. d. Presence of capsular material.

B. The correct answer is negative urease test. Cryptococcus neoformans would produce a rapidly positive urease test. All the remaining observations are consistent with the laboratory identification of C. neoformans. Upon observing mucoid appearing colonies on a primary isolation culture medium, perform a rapid urease test and set up a cornmeal agar preparation. If the urease test is rapidly positive, observe the cornmeal agar morphology. If pseudohyphae are absent, and particularly if the yeast cells are spherical, irregular in size and widely separated (presence of capsular material), inoculate the surface of a bird seed agar plate with a small portion of the unknown colony. If a brown pigment is produced within 48-72 hours, a definitive identification of Cryptococcus neoformans can be made.

What is the anatomical feature of a tapeworm that possesses both male and female reproductive structures? a. Brood capsule b. Proglottid c. Rostellum d. Scolex

B. The correct answer is proglottid. Proglottids comprise is the major portion of the body of tapeworms and they contain both male and female reproductive organs. The uterine branches will become packed with eggs once mature. The brood capsule is a cyst containing a scolex, typically found in Echinococcus granulosus. The rostellum is the crown of the scolex that may or may not have hooklets to assist the tapeworm in attaching to the intestinal wall. The scolex is the anterior portion of the tapeworm that has hooklets and/or suckers to allow the worm to attach to the intestinal mucosa.

The three main types of peripheral blood cells are: a. White blood cells, red platelets, and thrombocytes b. White blood cells, red blood cells, and platelets c. Leukocytes, erythrocytes, red blood cells d. Thrombocytes, platelets, and leukocytes

B. The correct answer is white blood cells (also known as leukocytes), red blood cells (also known as erythrocytes) and platelets (also known as thrombocytes).

Cholesterol is used by the body for which of the following functions? a. Carrying dietary triglycerides to the cells through the blood b. Maintaining acid-base balance c. Precursor of hormone synthesis d. Major source of fuel for the body

C. Cholesterol is a precursor of hormone synthesis. It is not a major source of fuel itself, but does facilitate triglyceride transport to serve the fuel needs of the body. Chylomicrons carry dietary triglycerides to the cells. Bicarbonate helps to maintain acid-base balance.

A 65 y/o male with type 2 diabetes presented to ED complaining of febrile episodes following his last dialysis treatment. The microscopic examination of his urine showed numerous white blood cells and bacteria. His urine was sent to the laboratory for culture and sensitivity. Urine colony count was >100,000 CFU/mL for a single organism that was identified as Klebsiella pneumoniae. Observe the reactions of this organism on the Kirby Bauer antibiotic susceptibility plate and review the susceptibility results on the AST report form. The Hodge Test was positive. Which resistance mechanism is this organism demonstrating? a. AmpC Beta Lactamase b. KPC - Klebsiella pneumoniae c. CarbapenemaseK-1 Beta Lactamase d. ESBL (Extended-Spectrum Beta Lactamase

B. The correct response is KPC. The Kirby Bauer antibiotic susceptibility plate reveals a highly resistant organism. KPC is a Class A plasmid-mediated betalactamase. As seen in the susceptibility test results, this organism hydrolyzed cephalosporins, cephamycins, carbapenems and monobactams. This is a multidrug resistant Enterobactericeae (CRE) that is a KPC which was confirmed by a positive Hodge Test. The patient requires contact isolation. AmpC beta-lactamases are cephalosporinases that hydrolyze all beta lactam antibiotics except cefepime and carbapenems. Since this organism is resistant to cefepime and the carbapenems, it is not an AmpC. K-1 Betalactamase, typically found in Klebsiella oxytoca is a penicillinase that can hydrolyze aztreonam, cefurosime and ceftriaxone. Hyper-producers of K-1 betalactamase show greater activity against ceftriaxone than cefotaxime and also greater activity against aztreonam than ceftazidime. This organism was resistant to cefotetan, aztreonam, ceftazidime and ceftriaxone; therefore, it is not a K-1 betalactamse. ESBLs hydrolyze penicillins, cephalosporins and monobactams but are sensitive to cephamycins and carbapenems. This organism is resistant to cefoxitin (cephamycin) and ertapenem (carbepenem) so it is not an ESBL. Some ESBLs (CTX-M) however, can hydrolyze carbapenems if coupled with a porin change. ESBLS show a clavulanic effect, not seen with this organism.

What is the characteristic RBC that is uniquely associated with HbSS? a. Target cell (codocyte) b Sickle cell (drepanocyte) c. Polychromatophilic cell d. Spherocyte

B. The drepanocyte or sickle cell is the shape the red blood cell assumes when the hemoglobin is deoxygenated and polymerizes. This is the result of the inherited mutation in HbS whereby a polar amino acid in the beta globin chain is substituted by a non-polar amino acid. This occurs mainly in patients who are homozygous for this mutation (HbSS). Although target cells can be seen in HbSS patients' blood, they are not specific to this condition. Target cells can be seen in other hemoglobinopathies, and other anemias as well. Polychromatophylic RBCs can be seen in their smears as well, but they can be seen in almost all smears of patients who are compensating for anemia. Spherocytes are rounded-up cells, and are characteristic of Hereditary Spherocytosis and other anemias, but are not associated with Hb S.

What is the characteristic RBC that is uniquely associated with HbSS? a. Target cell (codocyte) b. Sickle cell (drepanocyte) c. Polychromatophilic cell d. Spherocyte

B. The drepanocyte or sickle cell is the shape the red blood cell assumes when the hemoglobin is deoxygenated and polymerizes. This is the result of the inherited mutation in HbS whereby a polar amino acid in the beta globin chain is substituted by a non-polar amino acid. This occurs mainly in patients who are homozygous for this mutation (HbSS). Although target cells can be seen in HbSS patients' blood, they are not specific to this condition. Target cells can be seen in other hemoglobinopathies, and other anemias as well. Polychromatophylic RBCs can be seen in their smears as well, but they can be seen in almost all smears of patients who are compensating for anemia. Spherocytes are rounded-up cells, and are characteristic of Hereditary Spherocytosis and other anemias, but are not associated with Hb S.

The genotype pictured, --/-a This drawing depicts which alpha chain genotype? a. Alpha thalassemia major (Hydrops fetalis) b. Alpha thalassemia intermedia (Hemoglobin H Disease) c. Alpha thalassemia trait d. Silent carrier

B. The genotype pictured, --/-a, is known as Alpha thalassemia intermedia or Hemoglobin H disease. That is, three of the four alpha genes are deleted. These patients tend to form tetrads of beta globin chains, known as Hemoglobin H. Alpha thalassemia major or Hydrops fetalis occurs when all four alpha genes are deleted. These patients produce Hemoglobin Bart which is the formation of gamma globin tetrads. This condition is incompatible with life. Alpha thalassemia trait is the deletion of two of the four alpha genes. The silent carrier has one alpha gene deleted. They tend to have no clinical manifestation of a hemoglobinopathy.

The migration rate of proteins on cellulose acetate is primarily the result of: a. Molecular content of the protein b. Ionic charge on the proteins c. Concentration of the proteins d. Type of proteins

B. The ionic charge of the sample proteins in protein electrophoresis is a paramount factor in establishing the migration rates for the proteins being analyzed. Major serum proteins carry a negative charge and when a current is applied to the medium, the proteins move towards the anode (positive electrode). Cellulose acetate electrophoresis separate serum proteins into 5 fractions: albumin, a1-globulins, a2-globulins, ß-globulins, and ?-globulins.

The Echinococcus granulosus life cycle requires both an intermediate host (sheep) and definitive host (dogs), as humans are an accidental intermediate. While the eggs are ingested by humans and sheep, what structures need to be ingested by the definitive host, dogs, in order to complete the life cycle of Echnicoccus? a. Eggs b. Cysts c. Proglottids d. Adult worms

B. The life cycle of Echinococcus granulosus is incomplete when humans become involved as the intermediate host, whereas when sheep serve in this role, the life cycle is completed. This organism also requires a definitive host which is typically a canine. The life cycle starts with mature adult worms that release eggs and proglottids in canine feces that are ingested by sheep (intermediate host). Eggs released in sheep feces are then ingested by humans (accidental host) and form hydatid cysts in liver, lungs and brain. If the eggs are able to hatch in the sheep, they are carried in the blood stream to other locations and mature into hydatid cysts. Once these cysts are ingested by canines, the life cycle is complete as the cysts can mature into adult worms, starting the process over again.

Which of the following choices best describes the primary function of antibodies? a. Protection of B-lymphocytes b. Binds with antigen c. Fixes complement d. Stimulates the immune response

B. The primary function of an antibody is to bind with its specific antigen to facilitate clearance of that antigen. Antibodies are produced by B-lymphocytes as part of the humoral response, but they do not protect B-lymphocytes. The complement system may be activated through the formation of antigen-antibody complexes, but this is not the primary function of antibodies. Antigens stimulate an immune response; antibodies are produced when an immune response is initiated.

Which of the following is the only artery that does not transport oxygenated blood? a. Femoral b. Pulmonary artery c. Carotid d. Aorta

B. The pulmonary artery brings carbon dioxide and oxygen poor blood from the heart to the lungs. The femoral artery is located in the groin area and brings oxygenated blood to the lower extremities. The carotid artery is found on the side of the neck and brings oxygenated blood to the head and brain area. The aorta is the largest artery in the human body. It splits into smaller arteries which distribute oxygenated blood throughout the body.

A blood sample is left on a phlebotomy tray for four hours before it is delivered to the laboratory. Which group of tests could still be performed? A. Glucose, Na, K, Cl, PCO2 B. Uric acid, BUN, creatinine C. Total and direct bilirubin D. CK, ALT, ALP, ACP

B. Uric acid, BUN, and creatinine are not affected by the delay in testing. Glucose can be utilized by the red blood cells in the sample while the specimen is left standing. Bilirubin is sensitive to light and should also be separated from the cells and analyzed quickly. Many enzymes such as CK, ALT, ALP, ACP can deteriorate quickly if not separated from the cells and refrigerated.

Which one of these Gram-positive species has a high incidence of vancomycin-resistance? a. Aerococcus species b. Enterococcus faecium c. Group D streptococci d. Streptococcus pyogenes

B. Vancomycin-resistance in Enterococcus faecium is close to 90% in blood cultures in the U.S. In addition to Enterococcus faecium, E. faecalis is also isolated in nosocomial infections such as UTI or bacteremia. Leuconostoc and Pediococcus are other Gram-positive cocci that demonstrate resistance to vancomycin. Aerococcus species, Group D streptococci, and Streptococcus pyogenes are usually susceptible to vancomycin. Aerococcus is an opportunistic pathogen causing infection similar to other gram-positive cocci. Group D streptococci are part of the viridans group found as normal flora, but may cause infections such as bacteremia and endocarditis. Streptococcus pyogenes is found as normal flora existing on skin and in the oropharynx, but may cause serious infections such as pharyngitis, necrotizing fasciitis, TSS, and rheumatic fever.

Satellitism observed in cultures is most commonly associated with which of the following bacteria? a. Neisseria meningitidis b. Haemophilus influenzae c. Pasteurella canis d. Bordetella pertussis

B. he correct answer is Haemophilus influenzae. Haemophilus influenzae (which requires X factor and V factor) will grow in the hemolytic zone of Staphylococcus aureus on blood agar plates. Sheep blood agar plates (SAP) provide Haemophilus with X factor (hemin) and the hemolysis of red blood cells by S. aureus releases V factor (NAD). The X and V factors (hemin and NAD, respectively) diffuse into the surrounding medium and promote the growth of Haemophilus in the area surrounding Staphylococcus colonies in a process called satellitism. For Haemophilus spp., the satellite test substitutes for the V factor test. Neisseria meningitidis and Pasteurella canis do not have any special growth requirements. They are both able to grow independently on SAP and chocolate agar, but do not grow on MacConkey agar. Bordetella pertussis requires special medium for isolation, but it isn't able to obtain any of the required nutrients from the normal growth of other organisms in satellitism.

This form, which measures 30 µm by 15 µm was found in stool. What is the identification? a. Diphyllobothrium latum egg b. Paragonimus westermani egg c. Clonorchis sinensis egg d. Fasciola hepatica egg

C. Clonorchis sinensis eggs are characterized by an operculum on one end of the egg, and a small knob on the other end of the egg. Also, there is a distinct rim around the operculum, which is referred to as shoulders. The size of the eggs is approximately 30 µm by 15 µm. Diphyllobthrium latum eggs also have an operculum and terminal knob, but no shoulders. The egg is also much larger with an average size of 65 µm by 48 µm. Paragonimus westermani eggs also have an operculum and shoulders, but do not have a terminal knob. They do show a terminal shell thickening. The eggs are also much larger, ranging from 78-120 µm by 45-60 µm. Fasciola hepatica eggs also have an operculum, but they do not have a terminal knob or shoulders. The eggs are also much larger, about 128-150 µm by 60-90 µm.

Which autoimmune disorder is characterized by a homogenous pattern in an immunofluorescence (IF) microscopy test for anti-nuclear antibodies (ANAs)? a. Sjogren's syndrome b. CREST syndrome c. Systemic Lupus Erythematosus (SLE) d. Scleroderma

C. A homogeneous ANA pattern is seen in more than 95% of patients suffering from Systemic Lupus Erythematosus (SLE). This pattern characterizes anti-DNA nucleoprotein antibodies, i.e. antibodies to nDNA, dsDNA. In addition to a homogeneous ANA pattern related to SLE, a peripheral or rim pattern of nDNA can be seen in active SLE and a speckled pattern related to the presence of anti-Smith antibody can be observed. Because the detection of ANAs is not exclusively diagnostic for SLE, their presence cannot confirm the disorder. However, the absence of ANAs can be used to help rule out SLE. Sjogren's syndrome can be associated with a homogeneous ANA pattern, a peripheral or rim ANA pattern, a speckled ANA pattern or a nucleolar ANA pattern. CREST syndrome is characterized by a discrete, speckled ANA pattern with antibody specificities to centromere, DNA, RNA and ENA. A centromere pattern is found infrequently in patients with SLE and other disorders. Scleroderma is characterized by a nucleolar pattern. Although Sjogren's syndrome and undiagnosed illnesses manifesting Raynaud's phenomenon can also demonstrate the presence of the 4-6s RNP antibody.

Which of the following is considered an indication for Rh immune globulin (RhIG) administration in postpartum women? a. Long term protection b. Antibody blocking c. Passive protection d. Active immunity

C. Active immunization induced by the D antigen can be prevented by the concurrent administration of the corresponding RBC antibody (anti-D). Rh-negative non-immunized mothers expecting Rh-positive infant(s) should receive Rh immune globulin (RhIG) in the third trimester or at 28 weeks' gestation. Postpartum Rh-negative non-immunized mothers should receive RhIG immediately after delivery of an Rh-positive newborn. For passive protection, RhIG therapy can be administered within 72 hours after delivery as it may be effective against formation of anti-D. There is no long term protection against the development of Rh antibodies due to potential for patient exposure in subsequent pregnancies. The mechanism for RhIG therapy does not block antibody; it attaches to the fetal Rh-positive RBCs and removed by macrophages in the maternal spleen.

Which of the following microcytic anemia's is the most common anemia other than Iron Deficiency Anemia (IDA) and accounts for more than one-third of anemia in hospitalized patients without blood loss? a. Pernicious anemia b. Megaloblastic anemia c. Anemia of chronic disease/inflammation d. Immune hemolytic anemia

C. Anemia of chronic disease, also called anemia of chronic inflammation, is the most common anemia after IDA. It accounts for one-third of anemic hospitalized patients that have not experienced blood loss. Both pernicious anemia and megaloblastic anemia are macrocytic anemias. Pernicious anemia happens to be a form of megaloblastic anemia. Immune hemolytic anemia tends to be a normocytic anemia. Compared to anemia of chronic disease and IDA, it occurs less frequently.

Which type of granulocyte has water soluble granules which may fade during the rinsing phase of the staining process using the Wright's stain? a. Neutrophilic segs b. Eosinophils c. Basophils d. Neutrophilic bands

C. Basophils contain substances such as histamine and heparin which are important in inflammatory and allergic responses. Many of the basophil's granules are water soluble, and thus could wash out or get lighter if the rinsing step during the staining process is prolonged. Segmented neutrophils, eosinophils, and banded neutrophils all contain cytoplasmic granules, but they are not as susceptible to fading during the rinsing step of the Wright's stain procedure.

What biosafety level (BSL) is required by the Centers for Disease Control and Prevention (CDC) for working with Mycobacterium tuberculosis, if there is a possibility that aerosols might be released? a. BSL-1 b. BSL-2 c. BSL-3 d. BSL-4

C. Biosafety Level-3. BSL-3 safety level is recommended by the CDC , if it is probable that aerosols might be released. There are four biosafety levels . Risk assessment is an important part of biosafety. The primary risks that determine levels of containment are infectivity, severity of disease, transmissibility, and the nature of the work conducted. In addition, the origin of the microorganism or agent in question, and the route of exposure is important. Biosafety Level 3 (BSL-3) builds upon the requirements of BSL-2. Microorganisms that can be encountered in a designated BSL-3 laboratory can be either indigenous or exotic. They can cause serious or potentially lethal disease through respiratory transmission. One example of a microbe that is typically worked with in a BSL-3 laboratory is Mycobacterium tuberculosis. Biosafety Level 1(BSL-1) is the least hazardous level. The risk of disease in healthy adults presents minimal potential hazard to staff and the environment. An example of a microorganism that is typically studied in a BSL-1 laboratory is a nonpathogenic strain of E. coli. Biosafety Level 2 (BSL-2) builds upon BSL-1 requirements. A BSL-2 laboratory poses moderate hazards to laboratory staff and the environment. Encountered microorganism are typically indigenous and associated with diseases of varying severity. An example of a microorganism is typically worked with at a BSL-2 laboratory is Staphylococcus aureus. Non-aerosol-producing manipulations (eg, preparing direct smears for acid-fast staining when done in conjunction with training and periodic checking of competency) can be performed using SL-2 practices and procedures, containment equipment, and facilities. When working with Zika virus in the laboratory, Zika virus preparations may be handled under BSL-2 precautions. Laboratories should perform a risk assessment to determine if there are certain procedures or specimens that may require higher levels of biocontainment, e.g., use of a biosafety cabinet for potential aerosol generating activities or suspicion that the specimen may contain a pathogen that requires BSL-3 precautions. Biosafety Level 4 (BSL-4) builds upon the containment requirements of BSL-3 and is the highest level of biological safety. There are only a small number of BSL-4 labs in the United States and globally. Microorganisms encountered in a BSL-4 lab are dangerous and exotic, posing a high risk of aerosol-transmitted infections. Infections caused by these microbes are frequently fatal and are without treatment or vaccines. Two examples of microbes encountered in a BSL-4 laboratory include Ebola and Marburg viruses.

In which order should the following capillary specimens be collected according to Clinical and Laboratory Standards Institute (CLSI)? Serum specimens Blood gas specimens Ethylenediaminetetraacetic acid (EDTA) specimens Other additive specimens a. Serum specimens, blood gas specimens, EDTA, other additive specimens b. EDTA, serum specimens, blood gas specimens, other additive specimens c. Blood gas specimens, EDTA, other additive specimens, serum specimens d. Other additive specimens, serum specimens, blood gas specimens, EDTA

C. Blood gas specimens should be collected first because any delays in the collection may introduce room air and alter the values of the sample. EDTA samples are to be collected second in order to prevent the formation of platelet clumps and clotting. Other additive specimens are to be collected third and serum specimens last, since they need to clot.

A body fluid was sent to the microbiology laboratory for culture and after 48 hours of non-CO2 incubation, a small catalase positive, non-spore forming, diphtheroid shaped Gram positive rod grew on sheep blood agar. This organism was considered to be a possible contaminate, since this organism is part of the normal skin microbiota. Which of the following organisms grew in the body fluid? a. Staphylococcus epidermidis b. Cutibacterium (Propionibacterium) acnes c. Corynebacterium species d. Bacillus subtillis

C. Corynebacterium species is the correct answer as this organism has a diphtheroid or palisading morphology and is found on human skin. The colonies tend to be pinpoint, small, white to gray colonies on sheep blood agar. This organism group is catalase positive, non-acid fast, and a non-spore forming Gram positive rod. This group is known to contribute to contamination of sterile body sites as a result of inadequate cleansing of the skin prior to specimen collection. Staphylococcus epidermidis is incorrect answer. Staphylococcus epidermidis colonies grow as white to cream in color and are non-hemolytic on sheep blood agar. Gram stain morphology will appear as Gram positive cocci in clusters and biochemically the organism is catalase positive, and coagulase negative. This organism makes up part of the normal human microbiota of the skin and can contribute to culture contamination of sterile body sites, if the area is not cleansed appropriately. Cutibacterium (Propionibacterium) acnes is the incorrect answer. Cutibacterium (Propionibacterium) species is part of normal skin flora and is often considered a skin contaminant in blood cultures and sterile body fluids. This organism grows as a small, white Gram positive pleomorphic organism, but can present with diphtheroid-like rod morphology. It is also catalase positive, nitrate reduction positive, and sensitive to vancomycin and kanamycin. This organism can be separated from Corynebacterium species because Cutibacterium (Propionibacterium) acnes is an anaerobic organism. Bacillus subtillis is incorrect because this organism is a large, spore-forming, Gram positive rod. This organism is found as normal flora within the intestines and is not typically found as a skin contaminate.

Eosinophilia is commonly found in which of the following disorder? a. Pernicious anemia b. Liver disease c. Parasitic infections d. Infectious mononucleosis

C. Eosinophilia is most commonly seen as a result of allergic reactions, medication reactions and parasitic infections. Pernicious anemia is characterized by macrocytic erythrocytes (MCV>100). It is caused by an absence of intrinsic factor that leads to cobalamin deficiency. Liver disease leads to a nonmegaloblastic anemia. Hemolysis can occur and also impaired bone marrow response, folate deficiency and blood loss. Infectious mononucleosis is a viral infection characterized by a large percentage of reactive lymphocytes on the peripheral blood smear.

What should be done FIRST if a donor unit of red blood cells is found to be incompatible at the antiglobulin phase but compatible at immediate spin with several different recipients? a. Repeat the ABO grouping on the incompatible unit using different sera b. Do a panel made up of red cells having all low-frequency antigens c. Perform a direct antiglobulin test (DAT) on the donor unit d. Obtain a new specimen and repeat the minor crossmatch

C. If a donor unit of red blood cells is found to be incompatible at the antiglobulin phase with several recipients, the possibility of antibody coating the red blood cells is likely. A DAT should be performed on the donor unit. The antiglobulin phase of a crossmatch involves the addition of anti-human globulin (AHG). If donor cells are coated with antibody from a positive DAT the AHG will crosslink the donor cells and the crossmatch with be incompatible. ABO grouping does not need to be repeated since ABO incompatibility would be seen at immediate spin. It is unlikely that multiple recipients have alloantibodies to a low frequency antigen present on the donor unit. Antibodies to low frequency antigens are uncommon due to the low percent of patients stimulated (since not many donor cells have the antigen) to produce antibodies during a transfusion. Lastly, a new specimen is not needed for a minor crossmatch since plasma is not being transfused.

Which of the following anthrax infections is also known as "woolsorter's disease"? a. Cutaneous anthrax b. Gastrointestinal anthrax c. Inhalation anthrax d. Injectional anthrax

C. Inhalation anthrax is also known as "woolsorter's disease". Spores are inhaled into the pulmonary parenchyma. The severe phase of the disease has a high mortality rate. Cutaneous anthrax can occur when wounds are contaminated with anthrax spores. About 99% of anthrax cases in the world are cutaneous. A small pimple or papule forms at the site of inoculation 2-3 days after exposure. A small dark necrotic area known as an "eschar" forms. Usually, the infection remains localized. Gastrointestinal anthrax occurs after ingestion of spores. Less than 1% of cases of anthrax are GI. It presents with bloody diarrhea and other classic GI symptoms such as abdominal pain, nausea, and vomiting. Injectional anthrax occurs after injection drug use and results from the direct injection of spores into tissue. The absence of an eschar, and the fact that this form is much more severe (necrotizing fasciitis, organ failure, shock, coma can occur) than cutaneous anthrax make it clinically distinct.

In an urinalysis, tests are performed that serve as a method of screening for urinary tract infections. Which of following tests detects the presence of inflammatory cells? a. Niacin test b. Nitrate reduction test c. Leukocyte esterase test d. Catalase test

C. Leukocyte esterase test is the correct answer because this test detects inflammation, which indicates the presence of polymorphonuclear cells. These cells have granules that have leukocyte esterase activity. However, this test is not sensitive enough for determining pyuria in patients with acute urethral syndrome. Niacin test is incorrect because this test is used for differentiating Mycobacteria species. All the Mycobacteria species can reduce niacin with the exception of Mycobacterium tuberculosis. Nitrate reduction test is incorrect answer because this test looks for the presence of urinary nitrite, which indicates that an urinary tract infection is present. Organisms most often associated with urinary tract infections can reduce nitrate to nitrite; however, the nitrate reduction test may be negative, if the organism reduces nitrate completely to nitrogen gas or lacks the enzyme to reduce nitrate. The Enterobacteriaceae group is the group most often associated with urinary tract infections and has nitrate reduction ability. Catalase test is incorrect because this test is used to most often to differentiate Staphylococcus species from Streptococcus species from culture growth. However, this can be performed by adding 1.5-2.0 mL of urine to a tube containing dehydrated substrate. Hydrogen peroxide will then be added to the specimen tube and mixed. The formation of bubbles will indicate a positive test for catalase.

Toxic levels of lithium can cause lethargy, apathy, muscle weakness up to and including seizures and coma. What is the toxic level of lithium? a. 0.5-1.2 mmol/L b. 1.2-1.5 mmol/L c. 1.5-2 mmol/L d. >2 mmol/L

C. Lithium has a very low therapeutic level due to the highly toxic effects. Concentrations starting at 1.5 mmol/L and above can affect the CNS system in serious ways.

An aqueous 2.00 M HCl solution is prepared with a total volume of 0.475 L. The molecular weight of HCl is 36.46 g/mol. What mass of hydrochloric acid (in grams) is needed for the solution? a. 4.7 g b. 8.2 g c. 34.6 g d. 42.7 g

C. Molar concentration is defined as moles (mol) of solute per liter (L) of solution. Molar concentration= mol solute / L solution Rearrange this equation to solve for the moles of solute. Input the molar concentration and volume will tell you the moles of HCl. mol solute= molar concentration X L solution =2.00 M X 0.475 L =0.950 mol The moles of HCl then need to be converted to grams using the molecular weight. g HCl = 0.950 mol X (36.46 g/1 mol) =34.6 g The correct answer is 34.6 g.

The photomicrograph is a representative field from a peripheral blood smear. Which of the following MCV values correlates best with the peripheral blood picture? a. 68 fL b. 82 fL c. 105 fL d. 149 fL

C. Most of the cells observed in the peripheral blood smear are slightly larger than normal red blood cells. This is based on the fact that the majority of the red blood cells are larger than the nucleus of the small lymphocyte in the field, which can be used as a reference for the normal size of a red blood cell. These red blood cells are classified as macrocytes. The reference interval for MCV is 80-100 fL. An MCV value below 80 fL correlates with a finding of microcytes; an MCV value above 100 fL correlates with a finding of macrocytes. 105 fL is the best option since the red blood cells are only slightly larger than the nucleus of the small lymphocyte. An MCV of 68 fL correlates with a finding of microcytes. An MCV of 82 fL correlates with a finding of normocytes. An MCV of 149 fL correlates with very large macrocytes, much larger than those seen in this peripheral blood smear.

Which of the following causative agents is most likely associated with the "swimming pool granualoma" skin lesions depicted in the attached photograph? a. Mycobacterium kansasii b. Mycobacterium ulcerans c. Mycobacterium marinum d. Mycobacterium xenopi

C. Mycobacterium marinum has an optimal growth temperature of 28°-32º C, which is approximately the temperature of human skin. It has been implicated in diseases of fishes and has been isolated from aquariums and other bodies of water. Cutaneous infection occurs when traumatized skin comes into contact with salt water or inadequately chlorinated freshwater containing the organism. This may be in a body of water such as a pool or in an aquarium tank. The typical infection presentation is a tender red or blue subcutaneous nodule at the primary inoculation site such as the elbow, knee, toe or finger. The nodules are referred to as "swimming pool granuloma". M. kansasii and M. xenopi are primarily agents of pulmonary infections. Neither have been reported in association with cutaneous infections. Cutaneous infections with Mycobacterium ulcerans present as a painless nodule under the skin after traumatic inoculation from vegetation. This organism is not associated with water and infection does not show the typical "swimming pool granuloma" presentation.

A blood collection system that has a retractable needle malfunctions and causes an injury to a patient. Which of the following actions must be taken? a. The phlebotomist must be reprimanded b. The phlebotomist must be retrained c. The manufacturer of the device must be notified. d. The patient must be given a Hepatitis C vaccine.

C. One of the primary laboratory-related areas in the Safe Medical Devices Act of 1990 entails devices used during phlebotomy procedures. If it appears that a device has caused injury, it is important that the device and packaging be saved and any serial or lot numbers noted. An incident report should also be completed within 24 hours. The incident report must then be handled by the institution's "Risk Management" department (if applicable), who will file a report to the manufacturer. The problem was not related to the phlebotomist's competency to do his or her job. Retraining or reprimanding is not the required action for a medical device that malfunctions. The patient would not be given a Hepatitis C vaccine as a result of the incident.

Which of the following organisms shows a characteristic "test tube brush-like" pattern at 22 degrees Celsius? a. Listeria monocytogenes b. Pseudomonas fluorescens c. Erysipelothrix rhusiopathiae d. Enterococcus casseliflavus

C. Only Erysipelothrix rhusiopathiae produces the "pipe cleaner" or "test tube brush" growth appearance seen in the tube of soft gelatin agar shown in the photograph. These lateral extensions after 48-72 hours of incubation at 22°C are produced by both the smooth (S-form) and rough (R-form) type colonies, but more pronounced by the latter. The motility of Listeria monocytogenes is also most active at room temperature incubation, but is confined to a thin umbrella-shaped flare about 2mm beneath the surface of the agar. The motility of Pseudomonas species appears as a diffuse opacity in the agar, and that of Enterococcus casseliflavus as a diffuse, short lateral extension away from the stab line.

A large container of peritoneal fluid is received in the laboratory (about 50 mL). How should the tech process this sample for aerobic culture? a. Use a sterile pipette and add a drop of fluid per agar plate b. Insert a swab into the specimen container and inoculate all agar plates c. Centrifuge the specimen and use the sediment to inoculate the agar plates d. Add 1 mL of saline to 1 mL of fluid, vortex, and inoculate plates

C. Peritoneal fluid is a sterile fluid and should be inoculated directly to the agar plates. However, since there is such a large volume, centrifugation is required, and the sediment is then used for inoculation of the agar plates. Concentration of the specimen helps to recovery all bacteria present in the sample. Typically, fluids are centrifuged for 20 minutes at 3000 g. Sterile pipettes should be used for inoculation, and body fluids are typically inoculated directly to the agar plates, but due to the large volume, concentration of the specimen via centrifugation is needed first to enhance recovery of the organisms. Swabs are typically not used to inoculate fluids. If there is a sterile body fluid, the number of organisms present in the sample can be low, and the best way to promote organism recovery is to use a drop of the fluid instead of a swab for inoculation. Adding saline to the fluid would dilute out the fluid and decrease the ability to recover organisms present in the fluid.

Which of the following conditions would produce the results listed below in an anemic patient? MCV = 115 fL MCH = 30 pg MCHC = 34 % Please select the single best answer a. Sickle cell anemia b. Aplastic anemia c. Iron deficiency anemia d. Pernicious anemia

C. Pernicious anemia is the only choice that is a macrocytic anemia, meaning the MCV is increased. In this case, the MCV is 115 (normal range 80-100 fL), which means this patient is suffering from a macrocytic anemia. Sickle cell anemia, aplastic anemia, and iron deficiency anemia are all microcytic or normocytic anemias.

Parasites which can be found in urinary sediment include all of the following EXCEPT: a. Enterobius vermicularis b. Trichomonas vaginalis c. Plasmodium malariae d. Schistosoma haematobium

C. Plasmodium malariae is the only parasite that is listed that could NOT potentially be seen in urine sediment. Plasmodium is a blood parasite. Both Enterobius vermicularis and Trichomonas vaginalis could potentially arrive in the urine as contaminants from other sources; Enterobius from fecal contamination and Trichomonas from vaginal secretions. Of the parasites that are listed, only Schistosoma haematobium is a true urinary tract parasite.

In a patient with suspected malaria, which parasite demonstrates the following findings on a blood smear? Enlarged RBCs, some with fine brownish granules > 12 parasites in some cells Amoeboid structures Schuffner's dots a. Plasmodium falciparum b. Plasmodium ovale c. Plasmodium vivax d. Plasmodium malariae

C. Plasmodium vivax characteristically displays Schuffner's dots and often enlarged RBCs along with brownish granules. Plasmodium vivax can also have 12-24 merozoites present in each cell, actually filling the entire RBC. This parasite also has very irregular shapes often referred to as "Amoeboid". Plasmodium falciparum do not display Shuffner's dots. Plasmodium ovale does display Shuffner's dots in all stages, but is not as amoeboid as Plasmodium vivax. Plasmodium ovale characteristically has about 6-12 merozoites. Also, Plasmodium ovale characteristically shows enlarged, ovoid RBCs with fimbriated edges. Plasmodium malariae do not display Shuffner's dots and contains 6 to 12 merozoites.

Which personal protective equipment is mandated by OSHA to be used by healthcare workers when exposed to patients suspected or confirmed to have pulmonary Tuberculosis. a. Safety goggles b. Face shield c. Respiratory protection d. Rubber or nitrile gloves

C. Recommendations for personal protective measures to prevent illness or injury from exposure to chemicals or biological hazards, such as personal protective equipment (PPE) (for example, appropriate types of eye, face, skin, or respiratory protection needed based on hazards and potential exposure). Specialized masks and respirators are required measures for use by persons who are occupationally exposed to patients with suspected or confirmed cases of pulmonary TB. This requirement is mandated by OSHA. Classification of biological agents based on hazard to personnel: •Biosafety Level 1 - agents of relative low risk for causing disease in healthy adults (agents used for lab instructions such as Bacillus subtilis). Standard PPE for a BSL1 agent includes gown, gloves, and goggles with proper hand washing practices. •Biosafety Level 2 - agents that pose a moderate threat to a healthy adult (most patient specimens encountered in routine clinical laboratory settings, including common agents of infectious diseases; i.e. Salmonella spp. and Shigella spp.) Safety equipment and PPE can vary depending on the type of specimen and possible infectious agent. Tuberculosis would require respiratory protection. Specimens that could be a splash hazard require a face shield. Samples that could become aerosols are required to be used under a biosafety cabinet. •Biosafety Level 3 - agents not usually encountered in the clinical lab on a regular basis (many highly infectious viruses (certain arboviruses and arenaviruses, cultures of TB, and some systemic fungi). Use of a biosafety cabinet is required. gloves, gown, and eye protection should be worn. Respirators or face shields should be used depending on specimen type. •Biosafety Level 4 - agents not routinely found in a clinical lab, pose a high risk of aerosol transmission, are frequently fatal, and have no treatment or vaccine (Ebola, a filovirus, and other arboviruses and arenaviruses). All work is confined to a biosafety cabinet that with engineering features that prevent air flow. Full PPE as specified by the type of infectious agent must be worn at all times. Special disposal of used PPE and decontamination protocols may be required.

Which of the following specimen processing examples is considered a processing error and can have a negative impact on the test results? a. Transporting an ammonia specimen to the laboratory in an ice slurry. b. Allowing a red top blood tube to fully clot before centrifugation. c. Rimming the clot in a serum collection tube prior to centrifugation d. Aliquoting serum to a transfer tube after centrifugation is complete.

C. Rimming the clot in a serum collection tube can cause contamination due to aerosols and can also hemolyze the specimen. It should be avoided. All the other choices are appropriate processing examples.

All of the following analytes would be significantly increased in a plasma sample as a result of hemolysis, EXCEPT? a. Potassium b. LDH c. Sodium d. AST

C. Sodium is the most abundant cation in the extracellular fluid (ECF) and largely determines the osmolality of the plasma. Sodium concentration in the ECF is much larger than inside the cells. Higher levels of sodium in the blood could occur with excess water loss or decreased water intake. Potassium, LDH (lactate dehydrogenase), and AST (aspartate aminotransferase) are found in much higher concentration in intact RBCs than in serum, so when cells are lysed due to hemolysis, those compounds spill into the plasma and falsely increase the result. Potassium is the major intracellular cation in the body, with a concentration 20 times greater inside the cells than outside. The highest levels of total LDH are seen in pernicious anemia and hemolytic disorders. AST intracellular concentration is approximately 7000 times higher than the extracellular concentration.

Which of the following tests is collected in a culturette and tested in the microbiology department? a. Triglycerides b. Sputum screen c. Streptococcus screen d. BUN

C. Streptococcus screen is collected on a culturette (throat swab) and submitted to microbiology for Strep screen testing. Triglycerides are collected in a gel-barrier tube and tested in the chemistry department. Sputum screen is tested in the microbiology department for tuberculosis and requires for the sputum specimen to be collected in a sterile container. BUN is the test that needs to be collected in a serum gel tube and goes to the chemistry department for testing.

The Thayer-Martin agar plate seen in this illustration exhibits marked growth. The most likely organism found here would be: a. Staphylococcus aureus b. Streptococcus pyogenes c. Neisseria gonorrhoeae d. higella sp.

C. Thayer-Martin agar is a selective agar for culturing Neisseria gonorrhoeae or Neisseria meningiditis. Thayer-Martin agar contains three antibiotics to supress normal flora organisms in specimens. Vancomycin is used to inhibit gram positive organisms, Colistin is used to inhibit gram negative organisms, and Nystatin is used to inhibit yeast. In this image, there is heavy growth noted, which means that the antibiotics included in Thayer-Martin agar are not inhibiting bacterial growth. Since the agar is selective for N. gonorrhoeae and N. meningiditis, the correct answer is N. gonorrhoeae. Staphylococcus aureus and Streptococcus pyogenes are not correct because they are gram positive organisms. The Vancomycin in the Thayer-Martin agar will inhibit the growth of both of these organisms. Shigella species is not correct because it is a gram negative organism. The Colistin in the Thayer-Martin agar would prevent the growth of Shigella species.

The hematology analyzer reported an elevated white blood cell count and flagged for manual review due to the suspected presence of immature cells. What is the arrowed cell's identity, and what name is given to its inclusion? (picture not included) A. Lymphoma cell with ingested bacteria B. Promyelocyte with overlying platelet C. Blast with Auer rod D. Blast with a Döhle body

C. The arrowed cell is a blast containing an Auer rod. Auer rods are red-purple in color. They are composed of fused primary granules. One or more Auer rods may be observed in the cytoplasm of myeloblasts. Myeloblasts have between 0-20 primary granules. Lymphoma cells are medium to large cells with blue cytoplasm that may or may not contain vacuoles. The chromatin is less condensed than resting B and T lymphocytes. Lymphoma cells are not frequently seen phagocytizing bacteria. Promyelocytes are often slightly larger than myeloblasts but they have fewer and less prominent nucleoli and slightly coarser chromatin. Promyelocytes have more than 20 primary granules. Döhle bodies are blue-gray in color. They are typically found in band and segmented neutrophils.

The infectious diseases listed below are listed with the most commonly associated bacterium, EXCEPT: a. Haemophilus aegyptius - "Pink eye" b. Pseudomonas aeruginosa - "Swimmer's ear" c. Aspergillus niger - "Black lung" d. Burkholderia pseudomallei - "Meliodosis"

C. The correct answer is Aspergillus niger - "Black lung". Aspergillus niger is actually the most common fungal cause of "swimmer's ear". "Black lung" is actually caused by long term exposure to coal dust, not a microorganism. "Pinkeye" is a general term for acute infection of the conjunctiva, but classically has been attributed to Haemophilus aegyptius, thought by most microbiologists to be a variant of Haemophilus influenzae. Pseudomonas aeruginosa is the most common bacterial cause of "swimmer's ear". This occurs when water is retained in the outer ear canal, giving the perfect habitat for bacteria (or fungi) to grow. Melioidosis is a glanders-like infection (fever, malaise, cutaneous cellulitis, ulceration, and abscess formation) that affects animals and humans. It is caused by the bacterium, Burkholderia pseudomallei, which is endemic in regions of Thailand, Vietnam, and Southeast Asia.

A person who lived in the United Kingdom between the years of 1980 and 1996 for 3 or more months is indefinitely deferred from donating blood or blood products based on an increased risk of which of the following? a. Malaria b. Leishmaniasis c. Creutzfeldt-Jakob disease (CJD) and/or variant Creutzfeldt-Jakob disease (vCJD) d. Chagas disease

C. The correct answer is CJD and/or vCJD. If a potential blood donor spent more than 3 months in the United Kingdom between 1980 and 1996, or 5 or more years in France from 1980 to the present, they are indefinitely deferred due to an increased risk of CJD and/or vCJD. If they have ever been diagnosed with CJD or vCJD, they are permanently deferred. Persons with malaria or who have been infected with malaria are deferred for a period of 3 years after they are asymptomatic. The United Kingdom (UK) is not a malaria-endemic area and traveling to the UK in and of itself, would not be cause for deferral based on a potential malaria infection. Traveling to an area that is endemic for Leishmaniasis may be cause for a 12-month deferral from the time you left the area. The UK is not an Leishmaniasis endemic area. It is endemic in tropical and subtropical areas of the Middle East, Mediterranean, Africa, Central and South America, and Asia. Chagas disease is endemic in parts of Central and South America and Mexico. The UK is not an endemic area of Chagas disease. All potential donors are tested one time for Trypanosoma cruzi, the causative agent of Chagas disease. If the potential donor tests nonreactive for Chagas disease, the donor doesn't need to be retested. The rational for this is that most persons with Chagas disease have a chronic infection that was acquired while in a endemic area.

Which of the following is the prodrug of phenobarbital? a. Fosphenytoin b. Ethosuximide c. Primidone d. Gabapentin

C. The correct answer is Primidone. A prodrug is an inactive compound that once metabolized in the body, becomes biologically active. These help with steady-state kinetics. Primidone is metabolized into phenobarbital once absorbed by the gastrointestinal tract. Fosphenytoin is a prodrug but is the prodrug for phenytoin. Ethosuximide and Gabapentin are not prodrugs and are biologically active upon absorption.

The middle layer of the walls of veins and arteries is called: a. Tunica adventitia b. Tunica intima c. Tunica media d. Lumen

C. The correct answer is Tunica media. The walls of arteries and veins consist of 3 layers. They include: Tunica adventitia - composed of connective tissue is the outer layer of the blood vessel wall Tunica media - middle layer made up of smooth muscle and elastic tissue Tunica intima - the inner layer made up of endothelial cells The lumen is the space within the blood vessel where the blood flows.

What is a hemiacetyl? a. A compound with a carbonyl group b. A compound with an alcohol c. A compound with both a carbonyl and alcohol group. d. A compound that is part ketone

C. The correct answer is a compound with both a carbonyl and alcohol group. A compound with a carbonyl group on the terminal carbon is an aldehyde, as opposed to a ketone that has the carbonyl group on a middle carbon. When an alcohol group is added to an aldyhyde, it is called a hemiacetyl while an alcohol added to a ketone is a hemiketal.

Bacterial contamination is MOST likely in which of the following blood products? a. Packed red blood cells b. Frozen red blood cells c. Platelets d. Fresh frozen plasma

C. The correct answer is platelets. Bacterial contamination of platelets is most likely because they are stored at room temperature. The estimated rate of bacterial contamination in platelets is around 10%. Bacterial contamination is possible in packed red blood cells stored in the refrigerator, but the estimated rate of bacterial contamination is only around 0.2% Frozen red blood cells are not likely to have bacterial contamination. Fresh frozen plasma is not likely to have bacterial contamination.

All of the following are types of quality and process improvement tools, EXCEPT: a. Six Sigma b. Lean c. Sentinel events d. Root-cause analysis

C. The correct answer is sentinel event. A sentinel event is any unfavorable event that is unexpected and results in death or serious injury. This is not a quality or process improvement tool. Six sigma is a process used to evaluate preanalytic and analytic processes to minimize or eliminate waste. Lean methodology is a process that aims to improve patient safety and the quality of healthcare by eliminating unnecessary processes and redirecting efforts to a patient's needs. Root-cause analysis is an investigation and problem-solving approach used after an event or problem has occurred. The purpose of all quality and process improvement tools is to improve outcomes and standardize our processes.

You are a phlebotomist called to the floor to collect a blood specimen from a non-English speaking elderly patient. Upon entering the room, you find the patient with his adult son who speaks English. Which of the following scenarios is the most appropriate way to identify the patient prior to collection? a. Proceed directly to the patient and verify that the requisition information matches the patient's identification bracelet before performing the venipuncture b. Enter the room, look directly at the patient, introduce yourself, and explain in a loud voice why you are in the room and what you are about to do before verifying the patient's identification and performing the venipuncture. c. Enter the room, introduce yourself to both the patient and his son. Ask the son to state the father's full name and then ask the if he minds translating what you are about to do. Confirm the patient identification by verifying the requisition information with the patient's ID bracelet before performing the venipuncture procedure. d. Enter the room, introduce yourself to the son, ask the son to identify the patient and proceed with the venipuncture procedure.

C. The correct scenario is to enter the room, introduce yourself to both the patient and his son. Ask the son to state the father's full name and then ask the if he minds translating what you are about to do. Confirm the patient identification by verifying the requisition information with the patient's ID bracelet before performing the venipuncture procedure. In the first scenario, there is no verbal communication with the patient. This can cause unnecessary stress and confusion for a patient who does not speak English. The better approach is to have the family member involved in the process and to help translate. In the second scenario, speaking loud would not help ease the confusion of the patient, it may even frighten him. It is not an appropriate action. In the last scenario, the patient is totally ignored. Proper respect should be shown to the patient, he should be acknowledged even if he doesn't speak the language.

An investigation into a pre-analytical cause for an unanticipated laboratory finding should START with which of the following actions? a. Re-collection of the sample by a different phlebotomist. b. Verification of the sample identity, which includes at least two unique identifiers, such as the patient name, medical record number, and date of birth. c. Re-analysis of the sample from a secondary source, such as the primary tube, second collection tube, or alternative aliquot tube. d. Analysis of the same sample aliquot by an alternative analytical method to rule out method interference.

C. The first step in any investigation of a result that may be considered critical, significantly different than the previous result on the patient, out of analytical range, or unanticipated for other reason is to positively verify the patient identification.

Which of the following statements regarding peak level in therapeutic drug monitoring is true? a. Peak level is the point at which the drug is lowest in the patient's body. b. Peak level is assessed by collecting a blood sample right before the next dose of the drug is given. c. Peak level is the point at which the drug is highest in the patient's body. d. Peak level is assessed by collecting a urine sample right after the next dose of the drug is given.

C. The lowest level of the drug in the patient's body is called the trough level. The peak for a drug is when the level of the drug in the patient's body is the highest. To assess drug concentrations during the trough phase, blood should be drawn immediately before the next dose. To assess peak levels, the time for drawing depends on the route of administration: •Intravenous (IV): 15 - 30 minutes after injection/infusion •Intramuscular (IM): 30 minutes - one hour after injection •Oral: One hour after drug is taken (assumes a half-life of > two hours)

When collecting a urine specimen for workplace drug testing, what is the required minimum volume that must be collected for the sample to be accepted? a. 25 mL b. 35 mL c. 45 mL d. 55 mL

C. The minimum required urine specimen volume is 45 mL. Majority of specimens are split urine specimens. That is, at least 30 mL are poured by the collector into one container (specimen A) and at least 15 mL are poured into a second container (specimen B). Specimen A is the one tested initially, specimen B is the backup in case the results from specimen A are suspicious or in dispute.

Which of the following red blood cell morphologies are frequently artifactual? a. Drepanocytes, echinocytes, dacryocytes b. Drepanocytes, dacryocytes c. Echinocytes, stomatocytes d. Stomatocytes, acanthocytes, elliptocytes

C. The presence of echinocytes in stained peripheral blood smears is typically artifactual. When echinocytes are artifactual, they are commonly referred to as crenated red blood cells. Artifactual echinocyte formation occurs for many reasons: (1) increased pH (alkaline environment) due to substances released from the glass slide, called the "glass effect"; (2) blood stored at 4°C; (3) slow drying or drying in a humid environment; and (4) excess EDTA due to an under-filled collection tube (short draw). True echinocytes, also called burr cells, are mainly seen in liver disease, uremia, and pyruvate kinase deficiency. To determine if echinocytes are artifactual, a wet preparation can be made. If echinocytes are seen on the stained peripheral blood smear but not the wet preparation, the echinocytes are artifactual. Stomatocytes are common artifacts in stained peripheral blood smears. Codocytes (target cells) can also be artifactual if prepared in a high humidity environment or blown dry. Drepanocytes (sickle cells), dacryocytes (teardrop cells), and elliptocytes are not frequently seen as artifacts.

Uric acid is the final breakdown product of what two purines? a. Adenine and thymine b. Thymine and cytosine c. Adenine and guanine d. Guanine and thymine

C. The purines adenine and guanine are broken down into uric acid. Cytosine and thymine are pyrimidines which are mostly broken down into ammonia, carbon dioxide and water, but not into uric acid.

Compared to the primary immune response, the secondary immune response typically demonstrates: a. Less antibodies produced b. A longer lag time c. More antibodies produced d. A lower affinity

C. The secondary immune response typically presents with a greater number of antibodies that are produced as compared to the primary immune response. The primary immune response typically presents with the generation of less antibodies as compared to the secondary immune response. The secondary immune response presents with a shorter lag time as compared to the primary immune response. The secondary immune response typically presents with antibodies that have a higher affinity as compared to those from the primary immune response.

Which of the following analytes can be detected by the urine reagent strip in a point-of-care setting? a. Yeast, glucose, pH, and blood b. Crystals, bilirubin, urobilinogen, and protein c. Protein, glucose, leukocytes, and bilirubin d. Crystals, yeast, casts, and pH

C. The urine reagent strip is used to determine the presence of bacteria, blood bilirubin, leukocytes, protein, urobilinogen, pH, and specific gravity. Crystals, yeast, and casts are identified by performing microscopic urinalysis which is considered a moderate complexity test and must be performed by a medical laboratory thechnician (MLT) or a medical laboratory scientist (MLS).

This cell has pink granules in it's cytoplasm. What type of WBC is described? a. Neutrophil b. Monocyte c. Eosinophil d. Lymphocyte

C. This is an eosinophil that has been disrupted by the mechanical process of making the blood smear. Eosinophils are granulocytic white blood cells which are identified by their large orange/red granules and a dark purple nucleus usually segmented into two lobes. With few exceptions, a mechanically disrupted cell such as this one would not be reported as part of a WBC differential. Neutrophils are the most numerous granulocytic white blood cells. The neutrophils' cytoplasm stains pink, with bluish/lavender granules, and they have multilobulated nuclei which stains dark blue/purple. Monocytes are nongranular and the largest white blood cell by size. They have a folded nucleus and an irregular cellular shape. The cytoplasm stains gray-blue and often contains vacuoles. Lymphocytes are nongranular white cells with a large dark staining nucleus and very little cytoplasm which stains light blue. They are the smallest white cells by size and typically second most numerous after neutrophils.

An adult patient presents with a temperature of 103°F (39.4°C). A CBC is ordered and the white blood cell (WBC) count is 15.0 x 109/L (reference interval= 4.0 -11.0 x 109/L) with 10% band neutrophils and 2% metamyelocytes. What can be concluded at this point with these clinical and laboratory results? The correct answer is highlighted below a. This is most likely a parasitic infection. b. This is most likely normal for this patient. c. This is most likely a bacterial infection or inflammatory response. d. This most likely is a viral infection.

C. This most likely is either a bacterial infection or systemic inflammatory response. Increased temperature, increased white blood count, and immature cells of the neutrophilic line are all indicators of such. Neither symptoms nor results are indicative of a parasitic infection. The white blood cell line which responds most to these type of infections is the eosinophil cell line. Elevated temperature as well as immature neutrophils are not considered normal or within range. Symptoms and CBC results can vary in viral infections. However, a response by the lymphocytes is much more likely than by the neutrophils.

What is the shelf-life of whole blood collected in CPDA-1? a. 21 days b. 28 days c. 35 days d. 48 days

C. Whole blood collected with CPDA-1, or citrate-phosphate-dextrose-adenine, has a storage (shelf) life of 35 days from the date of collection. Whole blood collected with CPD (citrate-phosphate-dextrose), CP2D (citrate-phosphate-dextrose-dextrose), or ACD (acid-citrate-dextrose) has a storage (shelf) life of 21 days from the date of collection.

Absorbance (A) of a solution may be derived from percent transmittance using which formula? a. 1+ log%T b. 2+ log%T c. 1 - log%T d. 2- log%T

D. A = 2 - log %T , is the formula used to calculate absorbance from percentage transmittance data. This Beer-Lambert Law relates the transmittance of light to absorbance by taking the negative logarithmic function, base 10, of the transmittance observed by a sample, which results in a linear relationship to the intensity of the absorbing species and the distance traveled by light. The Beer-Lambert Law states that the absorbance of a sample is directly proportional to the path length of the sample holder and the concentration of the sample. So, if all the light passes through a solution without any absorption, then absorbance is zero, and percent transmittance is 100%. If all the light is absorbed, then percent transmittance is zero, and absorption is infinite.

Which of the following best describes a hemoglobinopathy? a. Any problem involving hemoglobin destruction. b. Any problem associated with hemoglobin production. c. A deletion of the loci of one or more hemoglobin chains. d. A substitution of an amino acid in the hemoglobin chain.

D. A hemoglobinopathy is a substitution of an amino acid in the hemoglobin chain. When an amino acid substitution occurs, a new protein is formed. Hemoglobin destruction or problems with hemoglobin production are not exclusive to hemoglobinopathies. Other anemias that are not considered hemoglobinopathies may also involve increased hemoglobin destruction or problems with hemoglobin production. However, other anemias demonstrating hemoglobin production problems do not form an abnormal hemoglobin protein. Deletion of a hemoglobin chain loci results in decreased production of that chain (thalassemia), and not production of a different hemoglobin protein.

Each of the cells below can be found in all types of body fluids EXCEPT: a. Neutrophils b. Macrophage/monocytes c. Lymphocytes d. Mesothelial cells

D. Body fluids can have peripheral blood leukocytes that migrate through the vascular endothelium into extravascular spaces, in addition to epithelial cells sloughed from the cells that line the membrane. Neutrophils, lymphocytes and macrophage/ monocytes can be found in all types of body fluid differentials. Mesothelial cells are found only in serous body fluids including pleural fluid, peritoneal fluid, and pericardial fluid.

Which of the following is the only vessel that allows for the exchange of oxygen (O2) and carbon dioxide (CO2)? a. Arteries b. Arterioles c. Veins d. Capillaries

D. Capillaries are the smallest blood vessels. They are the connection between arterioles and venules. They are the only vessel that allows for the exchange of O2 and CO2. Arteries carry oxygen rich blood away from the heart. Arterioles are vessels that connect arteries and capillaries. They are an extension of arteries that carry oxygen rich blood to capillaries. It is in the capillaries where oxygen can be released to tissues. Veins carry oxygen deficient blood and carbon dioxide back to the lungs and heart.

Suppose a patient with stiffness in her fingers has a positive antinuclear antibody (ANA) test with a centromere pattern at a 1:1280 titer. What is the most likely diagnosis? a. Systemic lupus erythematosus (SLE) b. Rheumatoid arthritis c. Sjogren's syndrome d. CREST syndrome

D. Centromere antibodies are present in over half of patients with the CREST syndrome, an acronym named after the major features of this autoimmune disorder: calcinocis, Raynaud's phenomenon, esophageal dysmotility, sclerodactyly, and telangiesctasia. While the ANA result alone is not diagnostic, this patient is having clinical symptoms of stiff fingers that fits nicely with the centromere ANA pattern. In Systemic lupus erythematosus (SLE), the ANA pattern is usually homogeneous or speckled; in rheumatoid arthitis and Sjogren's syndrome, it is usually specked.

In which of the following disorders would you probably observe coarse basophilic stippling on a Wright-stained peripheral blood smear? a. Hemolytic anemia b. Sideroblastic anemia c. Multiple myeloma d. Lead poisoning

D. Coarse basophilic stippling may be observed on the Wright-stained peripheral blood smear in cases of thalassemia or lead poisoning. In hemolytic anemias, a hallmark abnormality seen on the Wright-stained peripheral blood smear are spherocytes. In sideroblastic anemia, the Wright-stained peripheral blood smear will reveal microcytic, hypochromic red blood cells. In multiple myeloma the peripheral blood smears commonly have rouleuax formation. No other red blood cell abnormalities are typically noted.

A blood smear represented by the photograph was submitted for hematologic review. Based on the erythrocyte morphology and the accompanying histogram, which of the following choices is the most likely situation or condition? (Some cells are large, some are smaller. Two peaks on the histogram) a. Normal smear b. Iron deficiency anemia c. Aplastic anemia d. Dimorphic red cell population

D. Dimorphic or bimodal cellular distribution is a term used to describe two circulating red cell populations. One is the patient's basic red cell population while the other is a second population with distinct morphological features. Note the normal lymphocyte for size comparison with the two populations of red blood cells. In addition, the graphic illustration demonstrates two peaks in the red blood cell histogram. Dimorphic red blood cell populations can be found in conditions/situations such as: red blood cell transfusions or hemolytic processes involving a reticulocyte response. This may also occur when patients are given erythropoietin therapy. Cold agglutinins cause rbc agglutination and clusters of cells may appear as a separate larger rbc population on the rbc histogram. It is important to recognize when a population of cells in the peripheral smear is not in context with anticipated laboratory findings and the clinical situation. A normal peripheral smear will demonstrate a unimodal distribution on the rbc histogram. Iron deficiency anemia will typically demonstrate small rbcs (microcytes) thus a low mean cell volume (MCV). There will often be an increased red cell distribution width (RDW) indicating anisocytosis, but there are not two separate populations of rbcs. Aplastic anemia is a consequence of failure the hematopoietic process. All cell lines are affected. Red blood cells are usually normochromic and normocytic with a normal RDW.

A phlebotomist needs to collect the following tubes during a routine venipuncture: Green (heparin) Light blue (sodium citrate) Lavender (EDTA) Gray (sodium fluoride) Which tube should be collected last? a. Green b. Light blue c. Lavender d. Gray

D. Gray is the tube that should be collected last based on the tubes listed in the question. Blood collection tubes must be filled in a specific order to avoid specimen contamination from the additive in the preceding tube. This order of draw is an accepted drawing sequence based on CLSI Standard GP41: 1. Tubes or bottles for blood cultures - SPS (yellow top tube) 2. Light-blue top tubes (sodium citrate) 3. Serum tubes (with or without clot activator) 4. Green top tubes (sodium or lithium heparin) 5. Lavender or pink top tubes (Potassium EDTA) 6. Gray (Sodium fluoride and sodium or potassium oxalate) Clair-Clear Blew-Blue Red-Red Goldstein's-Gold Gross-Green Mini-Mint Penis-Pink and his Gray-Gray Pubes-Purple

Hemoglobin D (HbD) is elevated in all of the following EXCEPT? a. Hemoglobin D disease b. Hemoglobin SD disease c. Hb D/beta-thalassemia d. Alpha thalassemia

D. HbD is not present in alpha thalassemia. HbD is an inherited autosomal recessive variation of HbA that occurs in the ß-globin protein chain of HbA. The formation of HbD occurs by substitution of glutamic acid for glutamine at codon 121 of the ß-chain. HbD disease, also known as HbDD, occurs when two copies of the Hb D variant gene are inherited. It is also possible to inherit a condition known as Hb D/beta-thalassemia (Hb D/ß-thal). Although rare, HbSD does exist.

The production of and development of blood cells is called: a. Hemoglobin b. Hematocrit c. Hemostasis d. Hemopoiesis

D. Hemopoiesis is the production of and development of blood cells. Hemopoiesis usually takes place in the bone marrow. Hemostasis is the process by which the body stops the bleeding after an injury, it is also known as the coagulation process. Hemoglobin is the iron containing pigment found in red blood cells whose function is to bind oxygen and carbon dioxide and help their transport through the circulatory system. Hematocrit is the percentage of red blood cells in whole blood.

The production of and development of blood cells is called: a. Hemoglobin b. Hematocrit c. Hemostasis d. Hemopoiesis

D. Hemopoiesis is the production of and development of blood cells. Hemopoiesis usually takes place in the bone marrow. Hemostasis is the process by which the body stops the bleeding after an injury, it is also known as the coagulation process. Hemoglobin is the iron containing pigment found in red blood cells whose function is to bind oxygen and carbon dioxide and help their transport through the circulatory system. Hematocrit is the percentage of red blood cells in whole blood.

All of the following are examples of entities covered by the HIPAA Privacy Rule, EXCEPT? a. Hospitals b. Physician's offices c. Health care billing companies d. Hospital and laboratory accrediting agencies

D. Hospital and laboratory accrediting agencies are considered business associates and not covered entities. Business associates provide services to covered entities. Covered Entities include health care providers including hospitals and physician offices; health plans, such as health insurance companies; and healthcare clearinghouses, such as billing companies. The privacy of individuals who are seeking health care is protected under HIPAA, but individuals are not considered covered entities.

Which of the following substances is a disinfectant commonly used in clinical laboratory settings? a. Insulin b. Tincture of iodine c. Acid citrate dextrose (ACD) d. Hypochlorite solution

D. Hypochlorite solution, also known as bleach, is a disinfectant not an antiseptic and should be used on inert surfaces only. Insulin is not a disinfectant, it is a substance produced by the pancreas and released in the blood stream to help glucose absorption by the tissues. Tincture of iodine (2% solution) is a substance used as a skin antiseptic in the clinical laboratory. Care must be taken as it can be an irritant for some patients. If a patient is sensitive to the tincture of iodine, a different antiseptic should be used such as isopropyl alcohol. ACD is an anticoagulant used in the collection of blood donations. Its function is to ensure that the red blood cells continue to maintain their oxygen carrying capability.

If the parents are group A and B respectively, what are the possible blood groups of their children? a. A and B only b. A and B and AB only c. A and B and O only d. A and B and AB and O

D. If the parents are phenotypes A and B, then their possible allele combinations are genotypes AA or AO and type BB or BO. The following blood types are possible when one allele from each parent is passed to the child: AO and BO parents = possibility of phenotypes (and genotypes) AB (AB), A (AO), B(BO), or O(OO) children AO and BB parents = possibility of phenotypes (and genotypes) AB(AB) or B(BO) children AA and BO parents = possibility of phenotypes (and genotypes) AB(AB) or A(AO) children AA and BB parents = possibility of phenotypes (and genotypes) AB(AB) only children Since we do not know the parents' allele combination, the best answer choice is A and B and AB and O.

A specimen from a 23-year-old female patient who is a Bombay phenotype arrives in the blood bank. You observe the following reactions upon tube testing. Which of the following statements is true? a. Type AB red blood cells can be transfused to this patient b. Type O red blood cells can be transfused to this patient in an emergency c. Type O NEG blood can be transfused to this patient d. Patient can receive only Bombay phenotype blood

D. Individuals with the Bombay phenotype react as group O in forward and back type testing. However, individuals with the Bombay phenotype produce anti-H in addition to anti-A and anti-B expected in group O individuals. H substance (L-fucose) is present on the red cells of 99.99 percent of the population and is expressed to the greatest extent on the red cells of type O individuals. Therefore, it would be very dangerous to transfuse this patient with any red cells that express the H antigen. Such an individual can only receive blood from a donor of the Bombay phenotype.

You are working to validate a new method for the detection of C-reactive protein (CRP) in serum. To assess precision, you run a medium level control 30 times and determine the mean to be 4.0 mg/dL with a standard deviation of 0.04 mg/dL. What could be said about the precision of this run assuming a CV less than 3% is acceptable? a. It is not acceptable, with a coefficient of variation of 100%. b. It is not acceptable, with a coefficient of variation of 10%. c. It is acceptable, with a coefficient of variation of 0.1%. d. It is acceptable, with a coefficient of variation of 1%.

D. It is acceptable with a coefficient of variation of 1%. Precision is often measured by extrapolating and evaluating the coefficient of variation, or CV. To find the percent CV, you must divide the standard deviation by the mean and multiply the result by 100. 0.04 mg/dL divided by 4.0 mg/dL is 0.01. Multiplied by 100 = 1%, which is well below 3%. This indicates that the precision of this particular run is acceptable according to the parameters given.

All of the following would are considered molecular testing techniques used in the clinical laboratory to identify unique nucleic acid sequences, EXCEPT? a. Enzymatic cleavage of nucleic acids b. Enzymatic amplification of target sequences c. Hybridization with nucleic acid probes d. Immunoassays

D. Nucleic acid-based methods are designed to detect changes at the DNA (deoxyribonucleic acid) or RNA (ribonucleic acid) level rather than to detect a synthesized gene product, such as a protein detected in immunoassays. Molecular testing technique used in the clinical laboratory to identify unique nucleic acid sequences include enzymatic cleavage of nucleic acids, enzymatic amplification of target sequences, hybridization with nucleic acid probes, and gel electrophoresis.

Which of the following antibodies is most often implicated as a cause of a delayed hemolytic transfusion reaction (DHTR)? a. Anti-S b. Anti-M c. Anti-Fya d. Anti-Jka

D. Of the antibodies that are listed, anti-Jka is most often reported as the cause of DHTR. Anti-Jkb is also well documented as a cause of DHTR. The antibody titer of these two antibodies commonly decreases rapidly in vivo, which contributes to the higher incidence of DHTRs. Anti-S has been implicated in severe Hemolytic Transfusion Reactions (HTRs) but less often in DHTRs. Anti-M is more commonly an IgM antibody and even when it reacts at 37°C it rarely causes HTRs. Antibodies that are also implicated in a DHTR include Kell, Rh, and Duffy system antibodies. *Kidd abs are just kidding.

The study of how variations in the human genome affect a given individual's response to medications is called? a. Drug half-life b. Therapeutic window c. Pharmacokinetics d. Pharmacogenomics (Pharmacogenetics)

D. Pharmacogenomics (Pharmacogenetics)- The study of how variations in the human genome affect a given individual's response to medications. Therapeutic window- The range between a drug's effective dose and its toxic dose. Drug half-life- The amount of time it takes for a drug's concentration in the body to decrease by 50%. Pharmacokinetics- The study of drug disposition in the body - how and when drugs enter the circulation, how long they stay there, and how they are eliminated.

Which testing method is appropriate for identifying specific allergens? a. Complement fixation b. C-reactive proteins c. Radioimmunoassay (RIA) d. Radioallergosorbent tests (RAST)

D. Radioallergosorbent tests (RAST) is a procedure that detects the presence of IgE (and IgG) antibodies to allergens; a method used to measure antigen-specific IgE by means of a noncompetitive solid-phase immunoassay. Complement fixation is a traditional procedure that detects the presence of a specific antigen-antibody reaction by causing in vitro activation of complement. If complement is not fixed, lysis of the preantibody-coated reagent erythrocytes (RBCs) occurs. The acute phase protein, CRP, is used clinically for monitoring infection, autoimmune disorders, and more recently, healing after a myocardial infarction. Radioimmunoassay (RIA) is an older and less frequently used laboratory technique that uses radioactive substances to evaluate immunoglobulins. Traditional RIA is done with specific antibodies in liquid solution. Solid-phase RIA uses antibody-bound to a solid support, e.g. glass beads.

In the clinical laboratory, Safety Data Sheets (SDS) shall be maintained and be available to employees in the: a. Housekeeping Department b. Nursing Office c. Personnel Department d. Workplace area

D. Safety Data Sheets (SDS) shall be maintained and be available to employees in the workplace area. The SDS system provides clinical laboratory personnel with procedures for handling or working with various chemicals and substances in a safe manner. SDSs includes information such as physical data (such as boiling points.), toxicity, health effects, first aid, reactivity, storage, disposal, protective equipment, and spill-handling procedures. SDS binders are required for all clinical laboratories and must be easily retrievable for laboratorians to use when needed, therefore they would not be placed in the housekeeping department, nursing office, or personnel department.

All of following are considered normal flora of the gastrointestinal tract EXCEPT: a. Lactobacillus b. Clostridium c. Peptostreptococcus d. Shigella

D. Shigella is a gastrointestinal pathogen transmitted by food and water. It causes a diarrheal illness characterized by gross blood and pus in the stool. It is among the most communicable of the bacterial diarrheal diseases due to the low infective dose required to cause illness. Lactobacillus, Clostridium and Peptostreptococccus are all members of the normal gastrointestinal flora.

Which one of the following may cause a FALSE-NEGATIVE result with antiglobulin techniques? a. Dust or dirt is present in the glass tubes used for antiglobulin testing. b. Patient or donor blood specimen was collected in silicone gel tubes. c. Patient or donor blood specimen has a positive DAT. d. Addition of AHG reagent is delayed for 40 minutes or more after final saline wash.

D. The AHG reagent must be added to the cells immediately following washing. Antibodies may elute from the cells if the cells are allowed to sit in saline without the addition of the AHG reagent. The AHG reagent can be neutralized by the presence of the antibodies now present in the saline causing a false negative result, or too little IgG antibody remains attached to the cells for detection by the AHG reagent. False positive reactions can result from dust or dirt in the glassware used for testing (clumping could occur that is mistaken for agglutination). False positive reactions can result from the use of samples collected in silicone gel tubes due to complement attachment. If the patient or donor has a positive DAT, antibody coating the patient or donor RBCs can interfere with antiglobulin testing, for instance, phenotyping with antisera requiring an antiglobulin phase of testing.

If a patient has a white blood count of 40,000/µL, what would be the MOST useful staining procedure to distinguish between bacterial infection and chronic myelogenous leukemia (CML)? A. Wright's stain B. Peroxidase (i.e. Myeloperoxidase) C. Alpha-naphthyl acetate D. Leukocyte alkaline phosphatase (LAP)

D. The LAP stain is used to determine if an increase of cells is due to chronic myelogenous leukemia or a leukemoid reaction due to infection or similar conditions. Cells from a noncancerous reaction stain positive with many intense blue granules; cells from chronic myelogenous leukemia have few blue granules. Wright stain does not provide any differential information other than possible cell classification. The myeloperoxidase stain is used to distinguish between the immature cells in acute myeloblastic leukemia (cells stain positive) and those in acute lymphoblastic leukemia (cells stain negative). Alpha-naphthyl acetate is a nonspecific esterase that is used to distinguish immature monocytic cells from myeloblasts. The use of molecular and cytogenetic techniques to detect the translocation between chromosomes 9 and 22 (resulting in the formation of the BCR-ABL1 gene) are currently the favored tests to diagnose a suspected case of chronic myelogenous leukemia.

Of the following, which has the longest half-life? a. Carbamazepine b. Valproic Acid c. Phenytoin d. Phenobarbital

D. The correct answer is Phenobarbital, which has a half-life of 70-100 hours. Carbamazepine: 4-8 hours; Valproic Acid: 11-17 hours; Phenytoin: 6-24 hours. All of these medications are anti-convulsants that treat different types of seizure disorders, depending on the type and the patient. Phenobarbital is a slow-acting medication that is slowly absorbed and metabolized by both the liver and kidneys. Carbamazepine has a high degree of variability with absorption and more toxic side effects than the others, so it is usually prescribed when patients do not respond to other medications. Valproic acid is rapidly absorbed by the gastrointestinal tract is eliminated completely by the liver. Phenytoin, while an anti-convulsant, is also used as prophylaxis in traumatic brain injuries to prevent functional loss of tissue; it is metabolized by the liver.

Which of the following organisms is both a transient colonizer of skin, as well as the most common cause of bacterial skin infections? a. Cutibacterium (Propionibacterium) acnes b. Streptococcus pyogenes c. Escherichia coli d. Staphylococcus aureus

D. The correct answer is Staphylococcus aureus. Staphylococcus aureus is a transient colonizer of skin, while coagulase negative staphylococci are permanent skin colonizers. Staphylococcus aureus is responsible for up to 45% of skin and soft tissue infections. Cutibacterium (Propionibacterium) acnes is s permanent skin colonizer and is not is not a common cause of skin infections. Streptococcus pyogenesis not a skin colonizer, but does fall at #7 on the list of the most common bacterial causes of skin and soft tissue infections. Escherichia coli is not a skin colonizer, but does fall at #4 on the list of most common bacterial causes of skin and soft tissue infections.

A stool specimen can be suspected of harboring Vibrio cholerae if it possess which of the following characteristics? a. If the stool is well formed and shows blood streaks. b. If the stool is soft, dark, and positive for occult blood. c. If the stool contains a high concentration of segmented neutrophils. d. If the stool is watery with a high pH and flecks of mucus.

D. The diarrhea caused by V. cholera is watery and contains large concentrations of sodium, bicarbonate, and other electrolytes, producing an alkaline pH. The diarrhea is caused by the action of cholera toxin on the intestinal epithelial cells, with the end result of stimulating adenyl cyclase (cyclic AMP), which in turn inhibits the readsorption of sodium across the brush border membrane and stimulates the excretion of bicarbonate and potassium into the bowel lumen. A stool specimen comprised of primarily fluid and flecks of mucous (known as rice water stools) is a distinctive feature when cholera toxin activity is present. In the absence of mucosal invasion by the bacteria, blood or neotrophils are not found in the stool specimen.

Which of the following describes the basis for the freezing point Osmometer? A. The freezing point depression is inversely proportional to the amount of solute in the solution. B. The freezing point depression varies as the logarithm of the concentration of solute. C. The freezing point depression is raised by an amount that is inversely proportional to the concentration of dissolved particles in the solution. D. The freezing point depression is lowered by an amount that is directly proportional to the concentration of dissolved particles in the solution.

D. The freezing point depression is lowered by an amount that is directly proportional to the concentration of dissolved particles in the solution. Increase serum osmolality occurs with increase in solute which is directly proportional to the freezing point depression. The freezing point depression does not have a logarithmic or inverse relationship with solute concentration.

The glucose result on a urine specimen from an infant is negative on the urine chemical reagent strip, what is the next step? a. It can be reported that the specimen is negative for other reducing substances such as galactose. b. It must be followed by the Acetest® tablet test to check for other reducing substances. c. There is nothing else to do, report the result as negative for glucose. d. It must be followed by the copper reduction test (Clinitest®) to check for other reducing substances.

D. The method used on the urine chemical reagent strip is specific for glucose. An additional test, such as Clinitest®, should be used to check for other reducing substances. Clinitest® is a reagent tablet that is based on the classic Benedict's copper reduction reaction. This additional step is an easy way of screening for inborn metabolic problems, in particular, galactosemia. Acetest® is used as a confirmatory test for ketones and can be used to test urine, serum, and whole blood.

Which one of the following is a true statement about Lupus Anticoagulant (LA)? a. LA is a specific inhibitor found mainly in patients with the autoimmune disease, Systemic Lupus Erythematosus (SLE), b. LA is common in hemophiliacs c. LA prolongs coagulation leading to bleeding problems in patients. d. LA is an immunoglobulin.

D. The only true statement listed about Lupus Anticoagulant is that it is an immunoglobulin, which acts as an inhbiitor of phospholipid-dependent coagulation in vitro, but causes a tendency to thrombosis in vivo. Although LA can be found in some patients with SLE, it can be associated with many other diseases as well, including AIDS, malignancies, and others. Although hemophiliacs may be at risk for developing certain antibody inhibitors to Factor VIII products, these are not the same as the antiphospholipid antibody of LA. Although LA prolongs coagulation times in laboratory testing, in vivo the antibodies can attach to phospholipids and induce coagulation.

All of the following vacutainer tube colors are correctly paired up with the anticoagulant found in them EXCEPT? a. Light blue - Sodium Citrate b. Lavender - EDTA c. Gray - Potassium oxalate d. Red - Silica

D. While some plastic red top tubes contain silica, silica is not an anticoagulant. It is included in plastic tubes to aid in facilitating the clotting process, not preventing it. All the other tube colors and anticoagulants are correctly paired up with one another.

What is the correct sequence of the immunologic Epstein-Barr virus (EBV) response in infectious mononucleosis? Key:VCA= viral capsid antigenVCA/IgM =viral capsid antibody (IgM)EA=early antigenEBNA=Epstein-Barr nuclear antigen a. VCA, EBNA, VCA/IgM, EA b. EA, EBNA, VCA, VCA/IgM c. VCA/IgM, EBNA, VCA, EA d. EBNA, VCA, EA, VCA/IgM

The correct sequence of the immunologic Epstein-Barr virus (EBV) response in infectious mononucleosis is --VCA (viral capsid antigen), EBNA (Epstein-Barr nuclear antigen), VCA/IgM (viral capsid antibody (IgM), and EA(early antigen). Epstein-Barr infected B lymphocytes express a variety of new antigens encoded by the virus. Infection with EBV results in the expression of VCA, EA, and EBNA with corresponding antibody responses. Patients with nasopharyngeal carcinoma have elevated titers of IgA antibodies to EBV replicative antigens, including VCA. These IgA antibodies commonly precede the appearance of the tumor and serve as a prognostic indicator of remission and relapse. VCA is the very first, more obvious antigen to be detectable in infectious mononucleosis. VCA is produced by infected B cells and can be found in the cytoplasm. EBVA is found in the nucleus of all EBV-infected cells. Although the synthesis of EBNA precedes EA synthesis during the infection of B cells, EBNA does not become available for antibody stimulation until after the incubation period of infectious mononucleosis, when activated T lymphocytes destroy the EBV genome-carrying B cells. As a result, antibodies to EBNA are absent or barely detectable during acute infectious mononucleosis. Anti-VCA IgM appears in the sequence at about 3 weeks after infection. It is usually detectable early in the course of infection but is low in concentration and disappears with 2 to 4 months. Anti-VCA IgG is usually detectable with 4 to 7 days after the onset of signs and symptoms, and persists for an extended period, perhaps lifelong. EA is the last serologic marker to be detectable at about the 5th week in the EBV response. Incorrect Answers:EA, EBNA, VCA, VCA/IgM. EA is the last serologic marker to be detectable at about the 5th week in the EBV response. Two components of EA are diffuse early antigen, EA-D, found in the nucleus and cytoplasms of the B cells, and restricted early antigen EA-R, usually found as a mass only in the cytoplasm. EBNA precedes EA synthesis. VCA is the very first more obvious antigen to be detectable in infectious mononucleosis. Anti-VCA IgM appears in the sequence at about 3 weeks after infection. VCA/IgM, EBNA, VCA, EA. Anti-VCA IgM appears in the sequence at about 3 weeks after infection. EBNA precedes EA synthesis. VCA is the very first more obvious antigen to be detectable in infectious mononucleosis. EA is the last serologic marker to be detectable at about the 5th week in the EBV response. EBNA, VCA, EA, VCA/IgM. EBNA precedes EA synthesis. VCA is the very first more obvious antigen to be detectable in infectious mononucleosis. Anti-VCA IgM appears in the sequence at about 3 weeks after infection.

Below are the laboratory results on a 20-year-old overweight male. His waist circumference is 41 inches and blood pressure is 105/70. Fasting Blood Glucose: 130 mg/dLTriglycerides: 190 mg/dLHDL: 50 mg/dL The National Heart, Lung, and Blood Institute (NHLBI) and American Heart Association (AHA) have created a set of parameters to define the presence of metabolic syndrome. Does this patient meet the criteria for metabolic syndrome diagnosis? a. No, because his HDL is > 40 mg/dL b. No, because he is not hypertensive c. Yes, he meets the NHLBI and AHA criteria for metabolic syndrome d. No, since height and weight were not reported, his BMI cannot be calculated

c. His measurements meet the NHLBI and AHA criteria for metabolic syndrome diagnosis. Three of the five parameters are abnormal: waist circumference, fasting blood glucose, and triglycerides. BMI is not a parameter in this set of criteria. Fasting blood glucose, triglycerides, HDL-C along with waist circumference and blood pressure are the measurements used in the NHLBI and AHA criteria. There are five basic parameters described below as metabolic risk factors. A patient must have at least three of the five risk factors to be diagnosed with metabolic syndrome. A large waistline. In women, =35 in. (88 cm); in men, =40 in. (102 cm) High triglyceride level, =150 mg/dL High fasting blood glucose, =100 mg/dL Low HDL cholesterol. In women, <50 mg/dL; in men, <40 mg/dL High blood pressure, that is, =130/85 mm Hg

Which of the following tests should be collected in serum gel tube and goes to the chemistry department? a. BUN b. Zinc c. White blood cell (WBC) count d. Reticulocyte count

A. BUN is the test that needs to be collected in a serum gel tube and goes to the chemistry department for testing. -Zinc is also tested in the chemistry department, but is collected in a royal blue tube. -WBC count is collected in a lavender top and is tested in the hematology department. -Reticulocyte count is collected in a lavender tube and tested in the hematology department.

Cultures from a post-abdominal cellulitis specimen grew Gram negative pleomorphic rods with the following characteristics: Grows on KV agar but does not show fluorescence Produces black colonies BBE agar Resistant to penicillin Which of the following is the MOST likely identification? a. Bacteroides fragilis b. Fusobacterium nucleatum c. Prevotella species d. Veillonella species

A. Bacteroides fragilis is the correct answer because Bacteroides fragilis is able to grow on KV agar and does not show fluorescence, while producing black colonies BBE agar. BBE agar is selective and differential for B. fragilis. B. fragilis is resistant to penicillin and the most frequently recovered anaerobe in the clinical laboratory. Fusobacterium nucleatum is incorrect because Fusobacterium nucleatum are slender gram negative rods with pointed ends that will not grow on KV agar, but will fluoresce chartreuse in color on anaerobic blood agar. Prevotella species is incorrect because Prevotella species will grow on KV agar, but the colonies can fluoresce brick red on anaerobic blood agar and black pigment on KV agar. Veillonella species is incorrect because Veillonella species are gram negative, tiny diplococci in clusters that can fluoresce red on anaerobic blood agar.

The granules that basophils contain are composed of: a. Histamine b. Serotonin c. Myeloperoxidase d. Alkaline phosphatase

A. Basophilic granules are composed of histamine. This characteristic associates increased basophils during allergic reactions. Serotonin is found in dense bodies in platelets. Myeloperoxidase is an enzyme found in myeloblasts. Alkaline phosphatase is an enzyme found in neutrophils.

Which of the following may be seen in a CSF sample of a patient that has a subarachnoid (intracranial) hemorrhage (SAH)? a. Macrophages containing hemosiderin b. Uneven distribution of blood in CSF tubes collected c. Clot is present in CSF sample d. Supernatant of CSF sample does not have xanthochromia

A. Characteristics of a subarachnoid hemorrhage (SAH) include even distribution of blood in tubes collected, no clot formation in the sample, as well as a xanthochromic supernatant to the sample. Another characteristic that correlates with a SAH is hemosiderin deposits in macrophages. As red blood cells degenerate, the breakdown products are seen in macrophages as dark, granular, iron-laden hemosiderin deposits.

Which of the following types of automated chemistry analyzers allows for each specimen and accompanying reagents to have their own space? Please select the single best answer a. Discrete analyzer b. Random access analyzer c. Multi-channel analyzer d. Parallel analyzer

A. Discrete = Each specimen and accompanying reagents have their own space Random Access = Specimens can be analyzed out of sequence Parallel = All specimens analyzed at the same time Multi-channel = Each specimen may be subjected to multiple analyses

A left arm abscess collected from an 8-year-old male patient grew small white colonies on 5% sheep blood agar after 24 hours incubation at 35°C in 5-10% CO2. The Gram stain of the organism showed small tightly grouped Gram positive cocci. Which of the following organisms is the most likely identification? a. Staphylococcus epidermidis b. Streptococcus milleri c. Corynebacterium xerosis d. Enterococcus faecium

A. Epidermidis From the choices provided, the clusters of small, gram positive cocci are most consistent with Staphylococcus epidermidis. Like all staohylococci, S. epidermidis form Gram positive cocci in clusters when Gram stained. S. epidermidis typically grows as small white to gray colonies on 5% sheep blood agar within 24 hours. Streptococcus milleri are small gram positive cocci found in chain formation when Gram stained. Colonies are small and demonstrate alpha hemolysis on 5% sheep blood agar and may take up to 48 hours to grow. Corynebacterium xerosis may grow as small white colonies on 5% sheep blood agar. They may take up to 48 hours and will form colonies smaller than those of S. epidermidis. Gram stain results of C. xerosis appear as gram positive bacilli in a coryneform formation. Enterococus faecium are gram positive cocci found in chain formation when Gram stained. Colonies typically grow on 5% sheep blood agar within 24 hours, are gray, and show gamma hemolysis.

Which of the following describes gastrin's role in digestion? a. Stimulates gastric acid-HCl secretion b. Stimulates secretion of a base to lower the stomach's pH c. Is secreted when the gastric fluid pH is low to buffer fluid d. Inhibits the secretion of intrinsic factor

A. Gastrin is secreted by the gastrointestinal mucosa cells in response to mechanical stress or high pH, both of which are produced by the presence of food in the stomach. Gastrin stimulates the stomach parietal cells to produce gastric hydrochloric acid.

The cestodes or tapeworms can be identified by the unique appearance of their eggs. Examine the image at the right and consider the following choices. Each is a correct match of the egg with its species, EXCEPT: (Picture not included-tried to describe) a. Large and irregular with 15-25 eggs and hooklets = Echinococcus granulosus b. Unembryonated, with a thin shell and indistinct operculum=Diphyllobothrium latum c. Hexacanth or lancet-shaped with six hooklets = Hymenolepis nana d. Dark brown shell which surround the oncosphere with a radially striated shell and 6 hooklets= Taenia spp.

A. Image A represents the egg of Dipylidium caninum, not Echinococcus granulosus. The egg packets of D. caninum are large and irregular and contain 15-25 globular eggs, with an onchosphere and 6 hooklets. The eggs of E. granulosus are hexacanth and resemble those of Taenia spp. Image B shows the egg of Diphyllobothrium latum which is unembryonated, with a thin shell and indistinct operculum. Cleavage fills the entire egg. Image C represents the egg of Hymenolepis nana, which is hexacanth or lancet-shaped with six hooklets. It is round to oval with a thin, smooth membrane and an outer shell. There is a clear space between the membrane and shell and 4-8 filaments emerge from its inner membrane. Image D represents the eggs of Taenia species, T. solium and T. saginata as the eggs are indistinguishable. The eggs have a dark brown shell which surround the oncosphere with a radially striated shell and 6 hooklets. The gravid proglottids must be examined to distinguish these two Taenia species.

Inflammation of a vein is known as: a. Phlebitis b. Embolism c. Thrombosis d. Aneurysm

A. Phlebitis is the inflammation of the vein, usually in the legs. Thrombosis is the obstruction of a blood vessel caused by a stationary blood clot. Embolism is the moving and lodging of an embolus, which could be a blood clot, a gas bubble, fat globule, or other foreign material, through the blood stream. This can cause a blockage of a blood vessel away from the embolus' origin site. Aneurysm is a balloon-like bulge filled with blood due to a weakness in the vessel's wall. As an aneurysm's size increases, there is a higher risk of blood vessel rupture causing internal bleeding and subsequent hypovolemic shock leading to death.

A patient with an infectious mononucleosis infection presents in the emergency room. Physicians order a spinal tap, which is immediately sent to the laboratory for review. What is the cell in the image below from this patient's cerebrospinal fluid sample? The correct answer is highlighted below a. Reactive Lymphocyte b. Monocyte c. Macrophage d. Mesothelial Cell

A. Reactive Lymphocyte The cell depicted with the arrow in this image is an atypical (reactive) lymphocyte. These cells are commonly found in certain viral infections, especially infectious mononucleosis. Notice the larger size and abundant cytoplasm present in this lymphocyte. There is also apparent vacuolization, which is a key feature of atypical lymphocytes. The chromatin pattern of this cell, as well as the overall shape, color, and size, rules out the monocyte, macrophage, and mesothelial cell choices.

What would you predict is the effect on iron absorption if erythopoietic activity is increased over a prolonged period of time? a. It would increase. b. It would decrease. c. It would remain the same. d. It is impossible to predict.

A. The absorption of iron would increase. As erythropoietic activity continues, it will deplete tissue iron, which results in transferrin receptor concentrations to increase. Transferrin is the protein that transports iron from the site of absorption to the developing erythrocytes. Unless there is a problem with the stomach or the duodenal and jejunal mucosal surfaces, the absorption of iron will increase, not decrease, or stay the same. This normal homeostatic mechanism is easy to predict. Once again, this is providing mucosal surfaces and stomach secretions are functioning properly.

A sample has reactions occurring at immediate spin and AHG in a panel that show varying reaction strengths. There is no obvious pattern that matches a particular panel cell or single antigen profile and the auto-control was negative. Which of the following is the most likely cause? Please select the single best answer a. An IgM and an IgG antibody b. An IgG antibody only c. An IgM antibody only d. An autoantibody

A. The cause would be the presence of both an IgM and an IgG antibody. Laboratorians should think multiple antibodies when reactions are occurring at two different phases (IS and AHG), varying strengths in reactions and no definite pattern. Patterns can sometimes be recognized if you look at each phase individually. For example: reactions at immediate spin may match an M antibody and reactions at AHG may match a D antibody. Varying strengths in reaction could also indicate dosage occurring. IgM antibodies most often react at IS, room temperature or colder. IgG antibodies are most often detected at the AHG phase. Since the autocontrol is negative, the positive reactions are caused by an alloantibody and not an autoantibody.

Illustrated in this photograph is a blood agar plate inoculated with a beta hemolytic streptococcus obtained from a throat culture of a patient with acute pharyngitis. A 0.04 µg bacitracin (left) disk and a SXT disk (right) had been placed in the areas of streaking. The reactions observed allow the most likely presumptive identification of a streptococcus belonging to Lancefield group: a. Group A b. Group B c. Group C d. Group G

A. The correct answer is group A. The zone of growth inhibition around the bacitracin (left) disk is presumptive evidence for a beta hemolytic streptococcus, group A. Further confirmation is the resistance to SXT (right). Group A streptococci are often recovered from throat infections; therefore, he reactions shown in this photograph along with the site of recovery allows the most likely report of, "beta-hemolytic streptococcus, presumptive group A." Group B streptococci demonstrate resistance to bacitracin. While there are occasional strains that may show sensitivity to bacitracin, group B streptococci would be an extremely rare isolate from throat cultures. It should be remembered that approximately 10% of group C streptococci may produce narrow zones of hemolysis around the bacitracin (left) disk, is also susceptible to SXT (right). Group G streptococci may produce a narrow zone of hemolysis around the bacitracin, but are susceptible to SXT, differentiating them from group A streptococci.

The trichrome stain is most commonly used to identify which class of parasites? a. Protozoa b. Nematodes c. Cestodes d. Trematodes

A. The correct answer is protozoa. Trichrome stains are used to identify intestinal protozoa, such as Entamoeba histolytica, Endolimax nana, and Giardia lamblia. Trophozoite and cyst morphology can be better visualized using permanent stains, such as the trichrome stains, due to their small size. Wet mounts are generally used for rapid identification of nematodes, cestodes and trematodes.

A patient has a hypercellular bone marrow and is suspected of having a myeloproliferative disorder. Laboratory features include: excess megakaryocyte proliferation and atypia, marked marrow fibrosis, and the JAK2 mutation. The most likely diagnosis is: a. Primary Myelofibrosis (PMF) b. Chronic Myelogenous Leukemia (CML) c. Chronic Neutrophilic Leukemia (CNL) d. Essential Thrombocythemia (ET)

A. The listed features are all diagnostic of PMF. The megakaryocyte proliferation and marrow fibrosis are most characteristic. CML does not have the JAK 2 mutation and has excess proliferation in many of the myelocytic lines. CNL is characterized by neutrophilia and the CSFR mutation. ET has excess megakaryocyte proliferation and the JAK2 mutation, but platelet counts are higher. Also, though there can be some fibrosis, it is not as characteristic or excessive as in PMF.

What percentage solution of sodium hypochlorite (bleach) is recommended as a routine laboratory disinfectant? Please select the single best answer a. 5% b. 10% c. 15% d. 20%

B. 10% Any blood, body fluid, or other potentially infectious material spill must be promptly cleaned up, and the area or equipment must be disinfected immediately. Safe cleanup includes the following recommendations: •Alert others in the area of the spill. •Wear appropriate protective equipment. •Use mechanical devices to pick up broken glass or other sharp objects. •Absorb the spill with paper towels, gauze pads, or tissue. •Clean the spill site using a common aqueous detergent. •Disinfect the spill site using approved disinfectant or 10% bleach, using appropriate contact time. •Rinse the spill site with water. •Dispose of all materials in appropriate biohazard containers.

What is the purpose of a t-test? a. Determining the mean of two different groups b. Comparison of two means from matched groups c. Comparison of frequency data d. Preparing a Chi-square test

B. A t-test is used to compare two means from matched groups. The t-distribution has one parameter called the degree of freedom. Student's t-test is useful to determine whether there is a statistical difference (or bias) between two methods. There are two different calculations associated with Student's t-test depending on if the data are paired or unpaired. For most clinical laboratory applications, the formula for paired t-test is used. Student's t-test is the ratio of the difference of two means to the standard deviation of that difference.

Which of the following organisms display the characteristic "Medusa head" on 5% sheep blood agar (SBA) after 18 hours of incubation at 35°C? a. Yersinia pestis b. Bacillus anthracis c. Francisella tularensis d. Brucella abortus

B. After 18 hours of incubation on SBA at 35°C, the slightly undulate margin of B. anthracis may show curling, displaying a so-called "Medusa head" appearance. This characteristic is also described as comma-shaped protrusions. Colonies have a ground glass appearance and are non-hemolytic, two characteristics that may also be used to differentiate B. anthracis from other Bacillus species. A distinguishing characteristic of Yersinia pestis is its preference and faster growth that occurs at 25°C. At 24 hours of incubation on SBA at 35°C, colonies are only pinpoint and translucent with a gray-white color. Colonies begin to demonstrate a "fried egg" morphology at 48 hours of incubation on SBA. F. tularensis can grow poorly or not at all on SBA, producing only tiny, pin-point, translucent colonies at best after 18-24 hours. This organism prefers cysteine-enriched media such as chocolate (CHOC), Thayer-Martin (TM), and buffered charcoal-yeast extract (BCYE). It would be difficult to see individual colonies on SBA growth that is less than 24 hours old. Although most isolates of Brucella abortus may grow on 5% sheep blood agar and chocolate agar, enriched agars and special incubation conditions are often required to achieve optimal growth. Colonies appear small and translucent after at least 48 hours of incubation.

Of the dimorphic fungal species listed below, which has the longest time of recovery in primary culture? a. Coccidioides immitis b. Histoplasma capsulatum c. Sporothrix schenckii d. Blastomyces dermatidis

B. The correct answer is Histoplasma capsulatum. Histoplasma capsulatum will take the longest time to appear in primary culture. On average, it will take seven to ten days before colonies appear. Clinical manifestations of histoplasmosis infections include pulmonitis, mediastinitis, pericarditis and mucocutaneous lesions. Sporothrix schenckii will usually appear in culture more rapidly than the other species, appearing in as little as 24-48 hours. Coccidioides immitis will generally appear in about five days. Blastomyces dermatidis usually appear in about five to ten days.

The Echinococcus granulosus life cycle requires both an intermediate host (sheep) and definitive host (dogs), as humans are an accidental intermediate. While the eggs are ingested by humans and sheep, what structures need to be ingested by the definitive host, dogs, in order to complete the life cycle of Echnicoccus? a. Eggs b. Cysts c. Proglottids d. Adult worms

B. Cysts The life cycle of Echinococcus granulosus is incomplete when humans become involved as the intermediate host, whereas when sheep serve in this role, the life cycle is completed. This organism also requires a definitive host which is typically a canine. The life cycle starts with mature adult worms that release eggs and proglottids in canine feces that are ingested by sheep (intermediate host). Eggs released in sheep feces are then ingested by humans (accidental host) and form hydatid cysts in liver, lungs and brain. If the eggs are able to hatch in the sheep, they are carried in the blood stream to other locations and mature into hydatid cysts. Once these cysts are ingested by canines, the life cycle is complete as the cysts can mature into adult worms, starting the process over again.

Direct examination of treponemes can be performed by: a. Light microscopy examination b. Darkfield microscopy examination c. VDRL testing d. Rapid Plasmin Reagin (RPR) testing

B. Darkfield microscopy allows for direct observation of spirochetes from an active syphilitic lesion. It is the test of choice for examining symptomatic patients with primary syphilis. A darkfield examination is also suggested for immediate results in cases of secondary syphilis. The traditional logarithm for syphilis testing of suspected patients with syphilis begins with observations of classic lesions present, suggestive sexually transmitted infection history, and current sexually transmitted disease. Non-treponemal testing such as the Rapid Plasmin Reagin (RPR) is the next step. A reverse sequence testing algorithm begins with the same patient observations but the next step is to perform treponemal testing by chemiluminescence immunoassay or enzyme immunoassay. Light microscopy examination is not effective in the detection of spirochetes. Darkfield or fluorescent microscopy techniques are required for direct observation of spirochetes. The VDRL assay is a flocculation test that measures IgM and IgG antibodies to lipoidal material released from damaged host cells, to lipoprotein-like material, and possibly to cardiolipin released from the treponemes. Without some other evidence for the diagnosis of syphilis, a reactive nontreponemal test does not confirm T. pallidum, infection. Antilipoidal antibodies are antibodies that are not only produced as a consequence of syphilis and other treponemal diseases, but also may be produced in response to nontreponemal diseases of an acute or chronic nature in which tissue damage occurs. Rapid Plasmin Reagin (RPR) testing is a non-treponemal testing method. Traditionally, it is the next step in serological assessment. The RPR is the most widely used assay to determine the presence of regain, an antibody formed against cardiolipin, a lipid remnant of damaged cells, cholesterol, and lecithin, is used to detect the nontreponemal regain antibodies.

What is a human infection caused by flukes called? a. Elephantiasis b. Distomiasis c. Zoonoses d. Swimmer's itch

B. Distomiasis is an infection cause by flukes (trematodes). Infections are caused by ingestions of the metacercariae, the second larval stage, from raw or undercooked fish. Laboratory diagnosis of infection is by finding eggs in the feces or sputum. Elephantiasis is a condition caused by the presence of filarial roundworm, Wuchereria bancrofti. This worm lives in the lymphatics of the host, and elephantiasis develops after chronic infection. Accidental human infection from an animal is known as zoonosis. Some animal diseases can be transmitted to humans through bites, contact with a carcass, or an arthropod vector. Diseases include Rabies, Tularemia, and Lyme disease. Swimmer's itch is an allergic reaction caused by the presence of select schistosomes. The schistosomes do not invade and mature in humans; instead it penetrates the skin and dies. The body then develops an allergic reaction.

This non-hemolytic organism was isolated from the blood culture of a patient. The image shows the organism's reactions in 6.5% NaCl broth and on bile esculin agar. With further testing, this isolate will most likely be identified as: (Picture not included: No growth in 6.5% broth and black stain in the bile esculin agar) a. Streptococcus pneumoniae b. Enterococcus spp. c. Streptococcus agalactiae d. Streptococcus bovis

B. Enterococcus spp. is the correct answer because it is 6.5% NaCl positive (growth with color change from purple to yellow) and bile esculin positive (growth and darkening of the slant). Streptococcus bovis is 6.5% NaCl negative, and bile esculin positive. Streptococcus pneumoniae and Streptococcus agalactiae are bile esculin negative.

The common name of which parasite is the sheep liver fluke? a. Fasciolopsis buski b. Fasciola hepatica c. Clonorchis sinensis d. Paragonimus westermani

B. Fasciola hepatica is commonly known as the sheep liver fluke. When the organism infects sheep, it causes a disease known as liver rot. Humans ingest metacercaria on raw water vegetation. Once ingested the larvae migrate through the intestinal wall and peritoneal cavity and travel to the liver. In general terms, the common names of these particular parasites are derived from the area of the human body where they tend to inhabit. Knowledge of the parasite life cycles is helpful in making this determination. Fasciolopsis buski is commonly referred to as the giant intestinal fluke, Clonorchis sinensis is the Chinese liver fluke and Paragonimus westermani is known as the lung fluke.

Ideally, what testing should be done prior to starting a new lot of control material in order to establish its mean and standard deviation? Please select the single best answer a. Should be tested consecutively by 20 different technologists b. Should be tested in 20 separate runs c. Should be tested 10 consecutive times on each of three work shifts d. Should be tested 10 times by the same person

B. Ideally, 20 separate runs of testing should be performed prior to starting a new lot of control material in order to establish its mean and standard deviation. It is best to include data from different shifts and different operators. If 20 runs are not possible, at least seven runs with three replicates per run can be used. Testing should be performed by multiple individuals when possible, but it is often not possible for testing to performed by at least 20 individuals. Testing does not need to be performed consecutively. 10 runs will not provide enough data to determine the mean and standard deviation with statistical accuracy. 10 tests will not provide enough information to determine the mean and standard deviation. Testing by multiple individuals is ideal to reduce the chance of systemic error.

What is the maximum interval during which a recipient sample may be used for compatibility testing if the patient has recently been transfused or was pregnant within the past 3 months? a. 24 hours b. 3 days c. One week d. Two weeks

B. If the patient has been recently transfused or pregnant within the past 3 months, then the maximum interval during which a recipient sample may be used for crossmatching is 3 days within the scheduled transfusion. Additionally, the maximum testing interval for compatibility testing is 3 days in cases when relevant medical and/or transfusion history is unknown. Donor and recipient samples are stored for a minimum of 7 days (or one week) following transfusion. Both 24 hours and two week time intervals are not included in compatibility testing protocols.

Consider the following matches of parasites with their common names. Each is a correct match EXCEPT: a. Blood fluke = Schistosoma japonicum b. Pork tapeworm =Taenia saginata c.Chinese liver fluke = Clonorchis (Opisthorchis) sinensis d. New World Hookworm = Necator americanus

B. It is important not only to recognize parasites by their scientific names but also by their common names. Familiarity with key identification characteristics as well as an understanding of organism life cycles may prove helpful in this process. The scientific name for the pork tapeworm is Taenia solium, not Taenia saginata, which is the common name for the beef tapeworm. Schistosoma japonicum is the scientific name for the blood fluke. Clonorchis (Opisthorchis) sinensis is the scientific name for the Chinese liver fluke. The scientific name for New World Hookworm is Necator americanus while the scientific name for the Old World Hookworm is Ancylostoma duodenale.

Normal adult CSF may have 0-5 white blood cells (WBCs)/µL. Which of the following cell types account for 60-100% of these WBCs? a. Neutrophils b. Lymphocytes c. Monocytes d. Eosinophils

B. Lymphocytes Typically, majority of cells seen in a normal adult CSF specimen are lymphocytes and monocytes (70:30 ratio). This ratio is reversed in children.

Select the primary reagent components used in the Jaffe reaction for creatinine. a. Alkaline copper (II) sulfate b. Saturated picric acid and sodium hydroxide c. Sodium nitroprusside and phenol d. Phosphotungstic acid

B. Saturated picric acid and sodium hydroxide are the main reagents used in the Jaffe reaction for creatinine determination. These chemical components usually cause the creatinine reagent to have a bright red-orange color.

Recovered from a sputum sample, this suspicious form measures 112 µm by 55 µm. What is the organism? a. Schistosoma mansoni egg b. Paragonimus westermani egg c. Hymenolepsis nana egg d. Diphyllobothrium latum egg

B. The correct answer is Paragonimus westermani egg. Paragonimus westermani is a fluke egg known to be found in human sputum. The characteristic size, shape and the presence of an obvious operculum aid in its identification. Schistosoma mansoni eggs possess a large lateral spine, Hymenolepsis nana eggs contain a six-hooked oncosphere within a rigid membrane with two polar thickenings, and Diphyllobothrium latum eggs possess an operculum, are smaller than Paragoniumus westermani but are typically found in the intestines.

Antibodies directed against granulocyte antigens have been implicated for all of the following EXCEPT? a. Febrile transfusion reactions b. Acute hemolytic transfusion reactions c. Transfusion-related acute lung injury (TRALI) d. Neonatal alloimmune neutropenia (NAN)

B. The correct answer is acute hemolytic transfusion reactions. Antibodies against granulocytes have not been suggested as a cause of acute hemolytic transfusion reactions. Antibodies directed against granulocyte antigens have been implicated in NAN, TRALI, febrile transfusion reactions, and others.

Which of the following is a TRUE statement when performing maintenance or repair tasks on electrical equipment? a. It is unnecessary to take any additional measures when performing repairs on a piece of equipment, except to turn it off. b. Tagout is the placement of a tag on the energy-isolating device notifying staff: "Do Not Operate this Equipment." c. The Occupational Health and Safety Administration (OSHA) does NOT have any standards that are aimed at protecting workers from injury when performing maintenance or repair tasks on electrical equipment. d. Employees performing maintenance or repair tasks on electrical equipment do not require training.

B. Tagout is the placement of a tag on the energy-isolating device notifying staff: "Do Not Operate this Equipment." Extra measures are necessary to reduce the risk of shock or injury due to the unexpected startup of equipment or the release of hazardous energy while a machine or instrument is out of service. Simply turning it off is NOT sufficient. The OSHA Control of Hazardous Energy standard, 29 CFR 1910.147, requires lockout/tagout (LOTO) procedures to protect workers whenever maintenance or repair tasks are being performed on electrical equipment. All employees who are authorized to lockout machines or equipment and perform the service and maintenance operations need to be trained in recognition of applicable hazardous energy sources in the workplace, the type and magnitude of energy found in the workplace, and the means and methods of isolating and/or controlling the energy.

History records an outbreak with the sudden onset of high fever, headache, and abdominal pain among 93 persons who had attended a public church dinner the week before. The photograph of an XLD agar plate is representative of the organism that may have been recovered from blood cultures from any one of these 93 persons. The most likely identification is: (Picture has colonies with black dots on it) a. Escherichia coli b. Salmonella serotype Typhi c. Shigella sonnei d. Edwardsiella tarda

B. The clinical history as presented is classic for typhoid fever. The HE agar plate shows the recovery of non-lactose fermenting colonies (clear appearance) some of which have black centers. This picture is consistent with Salmonella serotype Typhi, although Edwardsiella tarda is a possibility but can be ruled out because it does not cause a typhoid fever type syndrome. Escherichia coli will appear yellow without H2S production on XLD agar, and doesn't fit the clinical presentation in this case. Shigella sonnei would appear clear on XLD agar, but would not produce H2S, and doesn't fit the clinical presentation in this case.

An error was made in a handwritten blood bank record. Which of the following methods is acceptable for correcting the error to remain in compliance with current good manufacturing practices requirements? a. Use white correction fluid to completely cover the error. b. Draw a single line through the error so that it can still be read and add your initials next to it. c. Erase the error as much as possible and write the correction on top of the obliterated result. d. Use black marker to black-out the incorrect text and write the correction on top of the obliterated text.

B. The correct answer is "Draw a single line through the error so that it can still be read and add your initials next to it". 21 CFR 600.12 requires records to be legible and indelible. Records cannot be covered up with correction fluid or tape, or be obliterated. Corrections to a record should be made in a manner that allows the error to still be read so that it can be reviewed. The correction should also include the date the correction was made and the person who made the correction.

The glycoproteins neuraminidase (NA) and hemagglutinin (HA) are used to subtype the following virus: a. Human Papilloma Virus (HPV) b. Herpes Simplex Virus (HSV) c. Influenza A Virus d. Hepatitis B Virus (HBV)

B. The glycoproteins neuraminidase (NA) and hemagglutinin (HA) are used to subtype the influenza A virus. These typable glycoproteins project from the envelope and allow for strain differentiation. Determining the NA and HA is useful in vaccine development disease surveillance. The Human Papilloma Virus (HPV) is divided into various genotypes based on the viral DNA sequence. They do not have an envelope from which the glycopeptides would protrude. There are two types of Herpes Simplex Virus (HSV); HSV-1 and HSV-2. HSV-1 has been linked to the oropharyngeal mucosa and HSV-2 has been linked to genital sites but this division is definitive. Hepatitis B Virus (HBV) is a DNA virus that is a significant cause of liver damage (hepatitis) related to morbidity and mortality.

The sediment from a freshly collected urine specimen is examined microscopically. In addition to bacteria, what other finding from the list below would further indicate the presence of a urinary tract infection (UTI)? a. 0 - 2 cuboidal epithelial cells/high power field (HPF) b. > 100 white blood cells (WBCs)/ HPF c. 10 - 20 squamous epithelial cells/ HPF d. Rare hyaline cast/ low power field (LPF)

B. The presence of white blood cells, especially in large quantities as in this case, is very indicative of urinary tract infections as the white cells are migrating to the urinary tract to fight the bacterial infection. Hematuria, increased pH, and proteinuria are also usually present in UTIs.

Identify the crystals in the image. (Picture not included, flat sheets with notches) Please select the single best answer a. Leucine crystals b. Cholesterol crystals c. Bilirubin crystals d. Cystine crystals

B. These are cholesterol crystals. Cholesterol crystals appear in acid urine as large, flat, transparent plates with notched corners. Leucine crystals are associated with liver disease. They are found in acid to neutral pH urine specimens and are yellow in color. Concentric circles and the radial striations are noted microscopically. Bilirubin crystals are yellow, needle-like crystals that may appear in clumps. They are found in acid urine and are associated with liver disease. Cystine crystals are present in acidic urine, are typically colorless, and have a characteristic hexagonal shapes. These crystals are associated with cystinuria.

Which of the following organisms are Gram negative? a. Bacillus b. Vibrio c. Staphylococcus d. Nocardia

B. Vibrio species are Gram negative, oxidase-positive bacilli which are inhabitants of water. These organisms are the causative agents of cholera (V. cholerae) to wound infections. Bacillus, Staphylococcus, and Nocardia are all Gram positive bacteria. Bacillus species are environmental bacteria that cause foodborne gastroenteritis (B. subtilis and B. cereus) and anthrax (B. anthracis). Staphylococcus are found on skin and fomites and cause UTIs, wound infections, toxic shock syndrome, among other infections. S aureus is the most commonly isolated, pathogenic Staphylococci. Nocardia species cause skin and lymphocutaneous infections. Pulmonary infections may disseminate hematologically to the CNS.

A urine specimen produced yellow foam when shaken. What is the is the cause of the yellow foam? a. Phosphates b. Bilirubin c. Normal d. Red blood cells

B. Yellow foam- presence of bilirubin White precipitate upon refrigeration- presence of phosphates Clear/straw- Normal Pink- presence of red blood cells visible microscopically

Using the formula for creatinine clearance, calculate the clearance based on the values given below: Urine creatinine: 100 mg/dL Plasma creatinine: 1.9 mg/dL Urine volume: 1375 mL/24hrs Surface Area: 1.2 m2 Round to 1 decimal place. a. 1.04 x 105 mL/min/m2 b. 1.04 x 105 mL/min c. 72 mL/min/m2 d. 72 mL/min

C. 72 mL/min/m2 The formula for creatinine clearance (CrCl) is: UV/P x 1.73/SA Where U is the urine creatinine in mg/dL, V is urine volume in mL/min, P is plasma creatinine in mg/dL, and SA is surface area of the patient in m2. Setting up this problem, the first thing that needs to be done is to correct units. CrCl is reported in mL/min and mL/min/m2 if correcting for body surface area. 1375 mL/24 hrs is converted to 0.95 mL/min by dividing by 1440 (hours in a day times minutes in a day) and can then be entered into the formula: (100 mg/dL)(0.95mL/min)/(1.9 mg/dL) x 1.73/1.2 m2 Here, you can see the mg/dL cancel out and the units you are left with are mL/min/m2. Now, you just calculate and you will get 72 mL/min/m2.

Antibody identification interpretations would be considered correct 95% of the time or have a P value of 0.05 (5% probability that the result is due to chance) if you have: a. 2 positive reactions to rule in an antibody and 2 negative reactions to rule out an antibody b. 1 positive reaction to rule in an antibody and 3 negative reactions to rule out an antibody c. 3 positive reactions to rule in an antibody and 3 negative reactions to rule out an antibody d. 3 positive reactions to rule in an antibody and 1 negative reaction to rule out an antibody

C. A P value is used to statistically determine the probability that a certain set of events or results will happen by random chance. A P value of 0.05 means that there is a 5% chance that the pattern of reactivity is due to something other than the suspected antibody. This can also be interpreted as the data will be correct 95% of the time. Using 3 cells to rule in and 3 cells to rule out gives the highest probability of the above listed answers (P value of 0.05). By comparing the patterns of reactivity and non-reactivity, we can more safely assume that an observed pattern is not the result of chance alone.

Which one of the following is the correct definition of isoelectric point (pI)? Please select the single best answer a. Buffer formation of a positively charged ionic cloud that can affect the migration of the negative ionic cloud of the sample b. The ability of a molecule to have both negatively and positively charged groups c. The pH where a molecule has a net charge of zero d. The movement of charged particles in an electrical field

C. A molecule that has both negatively charged groups and positively charged groups is described as amphoteric. These charges can be changed by changing the pH of the solution. The pH where there is an equal number of positive and negative charges is the isoelectric point, that is the molecule has a net charge of zero.

A body fluid was sent to the microbiology laboratory for culture and after 48 hours of non-CO2 incubation, a small catalase positive, non-spore forming, diphtheroid shaped Gram positive rod grew on sheep blood agar. This organism was considered to be a possible contaminate, since this organism is part of the normal skin microbiota. Which of the following organisms grew in the body fluid? (Picture not included, Chinese letters arrangement) a. Staphylococcus epidermidis b. Cutibacterium (Propionibacterium) acnes c. Corynebacterium species d. Bacillus subtillis

C. Corynebacterium species is the correct answer as this organism has a diphtheroid or palisading morphology and is found on human skin. The colonies tend to be pinpoint, small, white to gray colonies on sheep blood agar. This organism group is catalase positive, non-acid fast, and a non-spore forming Gram positive rod. This group is known to contribute to contamination of sterile body sites as a result of inadequate cleansing of the skin prior to specimen collection. Staphylococcus epidermidis is incorrect answer. Staphylococcus epidermidis colonies grow as white to cream in color and are non-hemolytic on sheep blood agar. Gram stain morphology will appear as Gram positive cocci in clusters and biochemically the organism is catalase positive, and coagulase negative. This organism makes up part of the normal human microbiota of the skin and can contribute to culture contamination of sterile body sites, if the area is not cleansed appropriately. Cutibacterium (Propionibacterium) acnes is the incorrect answer. Cutibacterium (Propionibacterium) species is part of normal skin flora and is often considered a skin contaminant in blood cultures and sterile body fluids. This organism grows as a small, white Gram positive pleomorphic organism, but can present with diphtheroid-like rod morphology. It is also catalase positive, nitrate reduction positive, and sensitive to vancomycin and kanamycin. This organism can be separated from Corynebacterium species because Cutibacterium (Propionibacterium) acnes is an anaerobic organism. Bacillus subtillis is incorrect because this organism is a large, spore-forming, Gram positive rod. This organism is found as normal flora within the intestines and is not typically found as a skin contaminate.

An electrolyte panel is composed of which of the following tests? Please select the single best answer a. Sodium, potassium, glucose, chloride b. Potassium , glucose, chloride, albumin c. Sodium, potassium, chloride, CO2 d. Chloride, cholesterol, chloride, glucose

C. Electrolytes are essential for the normal function of nerve, muscles, and heart activities. An electrolyte panel is composed of sodium, potassium, chloride, and CO2. Glucose is a carbohydrate, albumin is a protein, and cholesterol is a lipid.

What is a hematoma? Please select the single best answer a. Is the accumulation of interstitial fluid under the skin causing swelling. b. Is the flow back of blood mixed with additive from the collection tube into the patient's vein. c. Is the discoloration (bruising) of the skin often caused by blood leakage from the vein into the surrounding tissue during venipuncture. d. Is the damaging or destruction of the RBCs, permitting the hemoglobin to be released into the fluid portion of the specimen.

C. Hematoma is the discoloration (bruising) of the skin often caused by blood leakage from the vein into the surrounding tissue during venipuncture. Edema is the accumulation of interstitial fluid under the skin causing swelling. Reflux of additive is the flow back of blood mixed with additive from the collection tube into the patient's vein. Hemolysis is the damaging or destruction of the RBCs, permitting the hemoglobin to be released into the fluid portion of the specimen.

Which of the following combination of globin chains comprise Hemoglobin H? a. Two alpha, two beta b. Two alpha, two gamma c. Four beta chains d. Four alpha chains

C. Hemoglobin H is a tetramer made of four beta globin chains. Hemoglobin H occurs when there is very limited alpha chain availability used to make normal hemoglobin A. Hemoglobin H forms in those affected with alpha thalassemia major as well as in people with the combination of two-gene deletion alpha thalassemia and hemoglobin Constant Spring. Two alpha, two beta is normal adult Hgb A Two alpha, two gamma is fetal hemoglobin Hgb F Four alpha chains cannot form a tetrad because they are highly insoluble and precipitate within the cell.

Which symptom of hemolytic disease of the fetus and newborn (HDFN) is associated with low levels of glucuronyl transferase? a. Anemia b. Increased reticulocyte count c. Jaundice d. Cardiac failure

C. Jaundice In HDFN, anemia generally occurs in utero as maternal IgG antibodies cross the placenta and bind to the corresponding fetal antigen. Red cell destruction is caused by macrophages in the fetal liver and spleen. The fetus attempts to compensate for anemia with increased erythropoiesis in the liver and the spleen, causing an increased reticulocyte count as immature red cells enter the circulation. Jaundice in a newborn suffering from HDFN is associated with low levels of glucuronyl transferase, a liver enzyme that conjugates bilirubin into its water-soluble, excretable form. Cardiac failure may occur because of untreated, uncompensated anemia.

Which of the following causes schistocyte formation in disseminated intravascular coagulation (DIC)? a. Degranulation of promyelocytes b. Excessive fever in patients with DIC c. Microthrombi disposition in small vessels d. Excessive platelet size

C. Microthrombi disposition in small vessels Schistocytes are the result of erythrocytic membrane damage caused by sheering of red blood cells as they pass through a fibrin mesh of clot formation occurring in the blood vessels. This intravascular clot formation process occurs in DIC (disseminated intravascular coagulation), TTP (thrombotic thrombocytopenic purpura), and HUS (hemolytic uremic syndrome). Therefore, schistocytes may be seen in all three of these conditions. The primary granules of malignant promyelocytes in acute promyelocytic leukemia have procoagulant activity. Patients with acute promyelocytic leukemia may develop DIC. While the primary granules are responsible for initiating DIC in these patients, they are not directly responsible for schistocyte formation. While DIC may be preceded by septicemia or viremia, the fever itself is not responsible for schistocyte formation. Excessive platelet size is not directly relevant to schistocyte formation.

Patients who develop severe sepsis or septic shock commonly have __________ plasma lactate values. a. Decreased b. Normal c. Increased d. Markedly decreased

C. Patients who develop severe sepsis or septic shock commonly have increased plasma lactate values. This is considered Type B lactic acidosis and is of metabolic origin. Other causes for Type B lactic acidosis include diabetes mellitus, leukemia, liver or renal disease, and poisoning with ethanol, methanol, or salicylate.

Donation of which apheresis blood product more than once every four weeks requires monitoring of total plasma protein and antibody levels? a. Red cell apheresis b. Plateletpheresis c. Plasmapheresis d. Leukapheresis

C. Plasma levels of total protein, IgG, and IgM levels must be monitored every four months in plasmapheresis donors because levels of these and other proteins present in plasma decrease following plasmapheresis. Blood components from red cell apheresis, plateletpheresis, and leukapheresis do not contain enough volume of plasma to necessitate the monitoring of plasma proteins.

In a patient with suspected malaria, which parasite demonstrates the following findings on a blood smear? •Enlarged RBCs, some with fine brownish granules •> 12 parasites in some cells •Amoeboid structures •Schuffner's dots a. Plasmodium falciparum b. Plasmodium ovale c. Plasmodium vivax d. Plasmodium malariae

C. Plasmodium vivax characteristically displays Schuffner's dots and often enlarged RBCs along with brownish granules. -Plasmodium vivax can also have 12-24 merozoites present in each cell, actually filling the entire RBC. This parasite also has very irregular shapes often referred to as "Amoeboid". -Plasmodium falciparum do not display Shuffner's dots. -Plasmodium ovale does display Shuffner's dots in all stages, but is not as amoeboid as Plasmodium vivax. Plasmodium ovale characteristically has about 6-12 merozoites. Also, Plasmodium ovale characteristically shows enlarged, ovoid RBCs with fimbriated edges. -Plasmodium malariae do not display Shuffner's dots and contains 6 to 12 merozoites.

Which risk marker has both positive and negative cardiovascular risk integrated into its measurement? a. LpPLA2 b. Oxidized LDL c. Apo B-100/Apo A-1 d. hs-CRP

C. The Apo B-100/Apo A-1 ratio has a positive risk marker (Apo B-100) and a negative risk marker (Apo A-1). Apo A-1 is the major protein found in HDL and removes cholesterol from extrahepatic tissue, while Apo B-100 is the major protein found in LDL. Risk diminishes as Apo A-1 goes up (a negative marker) and risk increases in proportion to Apo B-100 concentrations (a positive risk marker). This 'double-edged sword' provides good risk prediction. The other risk markers in question do not have this dual-advantage.

A flagellar stain would show peritrichous flagella for each of the following organisms except: a. Bordetella bronchiseptica b. Alcaligenes faecalis c. Pseudomonas aeruginosa d. Escherichia coli

C. The correct answer is Pseudomonas aeruginosa. Pseudomonas aeruginosa, and other Pseudomonas species, are motile via a single polar flagellum. All of the other motile strains listed possess peritrichous flagella. Peritrichous flagella are flagella that occur on all sides of the bacterium. Polar flagella are flagella that extend from one end of the bacterium. Lophotrichous flagella are polar flagella that multiply in tufts at one end or the other.

All of the following parasites are correctly matched with its respective primary symptom, EXCEPT: a. Enterobius vermicularis - perianal itching b. Plasmodium falciparum - fever and chills c. Trichomonas vaginalis - diarrhea d. Endolimax nana - usually no symptoms

C. The correct answer is Trichomonas vaginalis. Trichomonas vaginalis is most commonly associated with vaginal discomfort, a green frothy discharge, odor, pain, itching, and irritation. Enterobius vermicularis typically causes perianal itching due to the female worms laying their eggs in this region Plasmodium falciparum, a causative agent of malaria, is characterized by fever and chills. Endolimax nana is not considered to be a pathogen, according to the CDC.

All of the following colony and microscopic descriptions are paired correctly with the mold indicated, EXCEPT: A. Macroscopic description: rose red or purple red pigmentation Microscopic description: multi-celled, sickle shaped macroconidia Fusarium sp. B. Macroscopic description: grayish-brown colonies Microscopic description: Dark, elliptical conidia each supported by a conidiophore ("lollipops") Scedosporium apiospermum C. Macroscopic description: green, granular, rugose colonies Microscopic description: chains of spherical conidia produced from branching phialides Aspergillus sp. D. Macroscopic description: green lawn that extends from edge to edge Microscopic description: tight clusters of spherical conidia held by finger-like phialides Gliocladium sp.

C. The correct answer is macroscopic description: green, granular, rugose colonies; microscopic description: chains of spherical conidia produced from branching phialides; Aspergillus sp. "Chains of spherical conidia produced from branching phialides" is descriptive of Penicillium species, which typically produces green, granular, rugose colonies. "Multi-celled, sickle shaped macroconidia" is the description for Fusarium species, which produces colonies with a distinctive rose red or purple red pigmentation. "Dark, elliptical conidia each supported by a conidiophore ('lollipops')" is characteristic of Scedosporium apiospermum. The grayish-brown appearance is typical macroscopic morphology. The description of "tight clusters of spherical conidia held by finger-like phialides" is the microscopic description for Gliocladium, the colony of which typically appears as a "green lawn" that extends from border to border across the Petri dish.

A 4-year-old female from South Carolina who was suffering from malaise, bloody diarrhea and abdominal pain, was rushed to the emergency room. Examination revealed rectal prolapse. Stool was submitted for parasitic examination and this suspicious form was seen. It measures 45 µm by 20 µm. Which of the following is the correct identity of this suspicious form? (Picture not included, capped egg on both ends) a. Taenia solium b. Ascaris lumbricoides c. Trichuris trichiura d. Enterobius vermicularis

C. The nematode Trichuris trichiura (whipworm) is one of the more common nematodes known to infect children. Trichuris eggs possess characteristic polar plugs at each end of the organism, as depicted in this picture. Infected children may experience a range of complications including hemorrhage, weight loss, abdominal pain, blood-tinged stools, and diarrhea. Taenia solium is the pork tapeworm and has worldwide distribution. It is found in the United States, typically among Latin American immigrants and Mexican agricultural workers. The tapeworm is more prevalent in underdeveloped communities when pork is ingested undercooked or raw. Tapeworm eggs can be found in the stool 2 to 3 months after the tapeworm infection is established. The eggs are round or slightly oval (31-43 µm in diameter) and yellow-brown with a thick striated shell containing a six-hooked oncosphere. Ascaris lumbricoides is the most common and the largest roundworm and many infections are asymptomatic. The large, broadly oval, mammillated ova are typically stained brown from bile. Infertile eggs may be oval or irregularly shaped with a thin shell and containing internal granules. Enterobius vermicularis (pinworm) is distributed worldwide and is commonly identified in group settings of children 5 to 10 years. During the night, the mature female worm migrates out of the anus of the infected host and lays eggs in the perianal region. The embryonated eggs mature and a third-stage larva develops. Infections are typically asymptomatic, with the most common complaint being perianal itching and restless sleep.

Patients with which of the following conditions would benefit most from washed red cells: a. Warm autoimmune hemolytic anemia b. Cold autoimmune hemolytic anemia c. IgA deficiency with anti-IgA d. Multiple red cell alloantibodies

C. Washing not only reduces the number of leukocytes and platelets that are often responsible for febrile reactions, but also eliminates plasma proteins, such as IgA. Warm autoimmune hemolytic anemia, cold autoimmune hemolytic anemia, and alloantibodies all involve patient antibody binding to antigens on the donor red blood cells. Washing would not remove the antigens from the red blood cells so washing has no advantage in these cases.

An analytical method with a low detection limit would: a. Measure only what is being assayed b. Measure zero concentrations of analyte c. Perform accurately in the presence of interfering substances d. Measure low concentrations of analyte

D. An analytical method able to detect low concentrations of an analyte would be considered to have a low detection limit. Specificity refers to measuring only what is being assayed. Analytical specificity refers to the ability of an assay to measure on particular organism or substance, rather than others, in a sample. Limit of blank measures a sample that lacks the presence of the analyte.

In the process of thrombopoiesis, which is arranged from least mature to most mature? a. Thrombocyte, metamegakarocyte, megakarocyte, promegakarocyte, megakaryoblast, stem cell b. Stem cell, megakaryoblast, megakarocyte, promegakarocyte, thrombocyte, metamegakarocyte c. Megakarocyte, promegakarocyte, megakaryoblast, thrombocyte d. Stem cell, megakaryoblast, promegakarocyte, megakarocyte, thrombocyte

D. Bone Marrow megakaryocytes are derived from pluripotential stem cells. The sequence of development is as follows; stem cell, megakaryoblast, promegakaryocyte, megakaryocyte, thrombocyte (platelet). Platelets are produced from the cytoplasm of megakaryocytes. Note: Some older sources include a metamegakaryocyte stage (megakaryocyte with platelets being shed from cytoplasm). If this is included it would follow the megakaryocyte stage.

Serous fluid effusions may result from all of the following EXCEPT: a. Congestive heart failure b. Infection and inflammation c. Malnutrition d. Dehydration

D. A serous effusion is the result of fluid build-up between the membranes where serous fluid is produced. Dehydration would not result in a serous fluid effusion. Dehydration occurs when an individual loses or uses more fluid than they take in and there is not enough water or other fluids to carry out normal functions. Congestive heart failure, infection, and inflammation, and malnutrition are all possible causes of a serous fluid effusion.

Which of the following tests is the BEST screening test for neural tube defects? a. Placental lactogen b. Bilirubin c. Estriol d. Alpha-fetoprotein

D. Alpha-fetoprotein is produced by the fetal liver. It is measured in pregnant women, using maternal blood or amniotic fluid, as a screening test for a subset developmental abnormalities. Abnormally high levels of this substance is associated with defects in the fetal neural tube, while abnormally low levels are associated with the presence of Down' syndrome.

Suppose a patient with stiffness in her fingers has a positive antinuclear antibody (ANA) test with a centromere pattern at a 1:1280 titer. What is the most likely diagnosis? a. Systemic lupus erythematosus (SLE) b. Rheumatoid arthritis c. Sjogren's syndrome d. CREST syndrome

D. CREST syndrome Centromere antibodies are present in over half of patients with the CREST syndrome, an acronym named after the major features of this autoimmune disorder: calcinocis, Raynaud's phenomenon, esophageal dysmotility, sclerodactyly, and telangiesctasia. While the ANA result alone is not diagnostic, this patient is having clinical symptoms of stiff fingers that fits nicely with the centromere ANA pattern. In Systemic lupus erythematosus (SLE), the ANA pattern is usually homogeneous or speckled; in rheumatoid arthritis and Sjogren's syndrome, it is usually specked.

Daily activities by microbiology personnel should be performed to ensure patients receive the best care. All of the following activities should be performed daily EXCEPT? a. Compare Gram stain reports with what grows in culture to ensure all organisms are recovered b. Check antimicrobial susceptibility reports to ensure correct drug/organism profiles c. Check susceptibility reports for multiple-drug-resistant organisms d. Check to ensure autoverification of results is working correctly

D. Check to ensure autoverification of results is working correctly is the correct answer because this does not have to be performed daily. Autoverification must be validated once a year to ensure results are being reported correctly. Compare Gram stain reports with what grows in culture to ensure all organisms are recovered, Check antimicrobial susceptibility reports to ensure correct drug/bug profiles, and Check susceptibility reports for multiple-drug-resistant organisms are incorrect answers because these processes must be performed daily to ensure patients are receiving the best care. By monitoring these processes, errors can be identified and corrected before results reach the physician to ensure quality care is provided.

A laboratory test flags as a critical result. The technologist repeats the test and it again flags as a critical result. Which of these scenarios correctly describes all the actions that need to be taken after this occurs? a. The technologist immediately releases the result. b. The technologist phones the clinical person handling the patient's care (caregiver) and reports the critical test result. c. The technologist phones the patient's caregiver, reports the critical result, documents his placement of the call, who was notified, time and date of notification. d. The technologist phones the patient's caregiver, reports the result and asks the caregiver to read back the information; documents his placement of the call, who was notified, time and date of notification.

D. Critical laboratory test results that are given over the phone must only be given to a clinical person (person in charge of the patient's care). The person who receives the result should be asked to read back the information that is given to verify that it was heard correctly. The person who reported and called the result, the person who was notified, and the date and time of notification should be documented along with the test result.

Which of the following blood components will provide the best source of fibrinogen for a patient with hypofibrinogenemia? The correct answer is highlighted below a. Whole blood b. Fresh frozen plasma c. Platelets d. Cryoprecipitate

D. Cryoprecipitate is the only concentrated fibrinogen product available and is used to treat patients with congenital or acquired fibrinogen defects. Whole blood is used to increase red cells mass and plasma volume. Fresh frozen plasma is used to replace stable and labile coagulation factors. Platelets are used to treat bleeding caused by thrombocytopenia or thrombocytopathy.

Which of the following drugs is used to treat congestive heart failure? a. Disopyramide b. Quinidine c. Procainamide d. Digoxin

D. Digoxin is a cardioactive therapeutic drug used to treat congestive heart failure. Disopyramide, Quinidine and Procainamide are all anti-arrhythmic medications used to treat various cardiac arrythmias (irregular heartbeats).

Illustrated in the photograph is a 15 X 25 µm trophozoite recovered from the stool specimen of a 30-year-old man with intermittent chronic diarrhea and malabsorption with onset 4 months after a camping trip to the nearby mountains. From the multiple choices listed, select the most probable presumptive identification. a. Chilomastix mesnili b. Dientamoeba fragilis c. Trichomonas hominis d. Giardia duodenalis

D. Giardia duodenalis (lamblia) is the correct selection. Easy to recognize, this trophozoite presents with two nuclei, one on either side of a central axostyle, in the center of which are two median parabasal bodies, simulating a "mustache." This presentation is often referred to as a "monkey face"; or, "little old man of the river". Chilomastix mesnili is an incorrect response. Chilomastix trophozoites possess only a single large nucleus with a karyosome that is placed immediately beneath the outer membrane. One to three flagella are observed immediately adjacent to the nucleus. Dientamoeba fragilis is an incorrect response. D. fragilis produces only small trophozoites measuring only 5 - 12 µm, with a pair of nuclei, each with a prominent karyosome that may fragment into multiple granules. An outer chromatin ring is absent. Trichomonas hominis is an incorrect response. The trophozoites of T. homonis are small (5 - 15 µm) and teardrop in shape with a single anterior nucleus that is displaced adjacent to the outer membrane. An undulating membrane extends the full length of this trophozoite.

Hemoglobin D (HbD) is elevated in all of the following EXCEPT? a. Hemoglobin D disease b. Hemoglobin SD disease c. Hb D/beta-thalassemia d. Alpha thalassemia

D. HbD is not present in alpha thalassemia. HbD is an inherited autosomal recessive variation of HbA that occurs in the ß-globin protein chain of HbA. The formation of HbD occurs by substitution of glutamic acid for glutamine at codon 121 of the ß-chain. HbD disease, also known as HbDD, occurs when two copies of the Hb D variant gene are inherited. It is also possible to inherit a condition known as Hb D/beta-thalassemia (Hb D/ß-thal). Although rare, HbSD does exist.

What is a LIS in a hospital setting? a. Laboratory Immunology System b. Laboratory Intellectual System c. Laboratory Informative Subsets d. Laboratory Information System

D. Laboratory Information System

Which testing method is appropriate for identifying specific allergens? Please select the single best answer a. Complement fixation b. C-reactive proteins c. Radioimmunoassay (RIA) d. Radioallergosorbent tests (RAST)

D. Radioallergosorbent tests (RAST) is a procedure that detects the presence of IgE (and IgG) antibodies to allergens; a method used to measure antigen-specific IgE by means of a noncompetitive solid-phase immunoassay. Complement fixation is a traditional procedure that detects the presence of a specific antigen-antibody reaction by causing in vitro activation of complement. If complement is not fixed, lysis of the preantibody-coated reagent erythrocytes (RBCs) occurs. The acute phase protein, CRP, is used clinically for monitoring infection, autoimmune disorders, and more recently, healing after a myocardial infarction. Radioimmunoassay (RIA) is an older and less frequently used laboratory technique that uses radioactive substances to evaluate immunoglobulins. Traditional RIA is done with specific antibodies in liquid solution. Solid-phase RIA uses antibody-bound to a solid support, e.g. glass beads.

Vaccines can be divided into the following non-experimental types with the exception of: a. Live, attenuated b. Inactivated c. Toxoid d. Recombinant vector

D. Recombinant vector vaccines are experimental vaccines similar to DNA vaccines, but they use an attenuated virus or bacterium to introduce microbial DNA to cells of the human body. Vector refers to the virus or bacterium used as the carrier. Researchers are working on recombinant vector vaccines for HIV, rabies and measles. Live, attenuated vaccines contain a version of the living microorganisms that has been weakened in the laboratory to prevent the organism from causing disease. A live attenuated vaccine is the closest thing to exposure to a natural infection. Inactivated vaccines are manufactured by killing an infectious microbe with chemicals, heat or radiation. This type of vaccine is more stable and safer than live vaccines. The disadvantage of inactivated vaccines is that they stimulate a weaker immune system response than live vaccines. Usually, a weaker response necessitates additional doses or booster shots to maintain immunity, e.g. influenza vaccination. Toxoid vaccines are used when a bacterial toxin is the main cause of illness. Toxins can be inactivated. Such "detoxified" toxins called toxoids are safe for use as vaccines. A toxoid vaccine containing a harmless toxoid stimulates the immune system to produce antibodies that react to and block the toxin. Vaccines against diphtheria and tetanus are examples of toxoid vaccines.

A tech is using the H (flagellar) antigen to serotype a suspected Salmonella species. The results of the serotyping are agglutination with one well that indicates a group of possible serotypes, not one particular serotype. Quality control passed. Why did this occur? a. The tech contaminated the suspension used for serotyping b. The organism is not a Salmonella species c. The Vi antigen is covering the somatic antigen so serotyping is inaccurate d. Some Salmonella H antigens can occur in multiple strains and serotypes

D. Salmonella species have two different types of flagellar (H) antigens, phase 1 and phase 2. Phase 1 antigens occur in only a few serotypes and typically will provide the identity of one Salmonella serotype. Phase 2 antigens are non-specific and can occur in many different strains of Salmonella. Thus, serotyping that does not point to a specific serotype, indicates a phase 2 antigen, and can include several strains or serotypes. The sample would not be contaminated since the testing worked correctly and one well had agglutination. Contamination of multiple organisms is questioned if more than one well of testing is positive. Since there was agglutination in the testing, the organism should be a Salmonella species. If the organism was a different species, no agglutination would be the expected result. Salmonella typhi can contain a Vi antigen (capsular antigen) that can be removed by heating. However, the H (flagellar antigen) is on the flagella of the organism, which is not "hidden" by the Vi antigen like the somatic (O) antigen can be. Thus, heating the organism would not improve the agglutination of serotyping of the H antigen.

Upon reviewing a fecal preparation for Ova and Parasites, a technologist thought they had just identified the egg of Taenia saginata. After a quick look, the microbiology supervisor reminded the technologist that T. saginata eggs are indistinguishable from which of the following? a. Dipyllidium caninum b. Echinococcus granulosus c. Hymenolepis diminuta d. Taenia solium

D. The correct answer is Taenia solium. According to the CDC, speciation of Taenia is impossible if solely based on microscopic examination of eggs, because all Taenia species produce eggs that are morphologically identical. Microscopic identification of gravid proglottids (or, more rarely, examination of the scolex) allows species determination. Dipyllidium caninum, also known as the dog tapeworm, accidentally infects humans by ingestion of fleas containing the larvae. The eggs are typically seen in packets of 15 to 25 eggs. Individual eggs do resemble Taenia sp. but they are distinguishable. Echinococcus granulosus typically infect canines with sheep as the usual host for the larval stage. The eggs can be isolated from the definitive host and also resemble Taenia sp. eggs. Hymenolepis diminuta is also known as the rat tapeworm. The eggs most commonly need to be distinguished from H.nana, but are similar in size and shape to Taenia sp. eggs.

Adult cestodes utilize which body part to attach to the human intestine? a. Strobila b. Tegument c. Proglottids d. Suckers

D. The correct answer is suckers. The suckers serve as the mechanism by which adult cestodes obtain nourishment. Attachment to the human intestine allows feeding to occur. The strobila is the body of a tapeworm, excluding the head and neck region. The tegument is the outer body covering of helminths. The proglottids are the major portion of the body of the tapeworm that contains the reproductive organs and often eggs.

Which of the following stains is used to differentiate a neutrophilic leukemoid reaction from chronic myelogenous leukemia (CML)? a. Sudan black B b. Periodic acid-Schiff reagent (PAS) c. Acid phosphatase with tartrate d. Leukocyte alkaline phosphatase (LAP)

D. The leukocyte alkaline phosphatase stain, also known as LAP is the correct stain to differentiate between chronic myelogenous leukemia and a leukemoid reaction. LAP activity is scored in the segs and bands only with an activity score range from 0 to 4+. The scores of 100 cells are added to obtain the LAP score. LAP scores are characteristically low (usually less than 15) or absent in CML. LAP scores in leukemoid reactions are characteristically high, or over 100. The normal range for LAP scores is 20-100. Sudan Black B is used to differentiate myelogenous and myelomonocytic leukemia. Periodic acid-Schiff reagent (PAS) is used to differentiate erythroleukemia and acute lymphoblastic leukemia. Acid phosphatase with tartrate distinguishes lymphocytes from other mononuclear cells. Specifically, this stain is a significant marker in hairy cell leukemia, which is tartrate resistant.

The image is a stained smear of cerebrospinal fluid. The cell in this slide could be caused by what condition? (Picture not included-Very large cell and it's dividing) a. Viral meningitis b. Allergic reaction c. Cerebral hemorrhage d. Metastasis

D. The nucleus in this cell is abnormal and appears to have divided. This finding correlates with the presence of metastasis in the CSF. A broad spectrum of lymphocytes and monocytes would be present in a case of viral meningitis. An allergic reaction will show predominantly eosinophils. Cerebral hemorrhage would show predominantly RBCs.

The type of hypersensitivity reaction associated with macrophage activation, cytokine-mediated inflammation is: a. Type I Anaphylactic (Immediate hypersensitivity) b. Type II Cytotoxic (Antibody mediated and antibody dependent, complement mediated hypersensitivity) c. Type III Immune complex mediated hypersensitivity d. Type IV Cell mediated hypersensitivity (T-cell dependent)

D. Type IV Cell mediated hypersensitivity is associated with macrophage activation. Type IV is characterized by direct target cell lysis and cytokine-mediate inflammation. There are three defining characteristics of type IV hypersensitivity reactions: (1)Type IV delayed-type hypersensitivity involving antigen-sensitized T cells or particles that remain phagocytized in a macrophage and are encountered by previously activated T cells for a second or subsequent time. Delayed hypersensitivity is a major defense mechanism again various intracellular pathogens, including mycobacteria, fungi, and certain parasites. (2) Rejection of foreign tissue grafts, elimination of tumor cells bearing neoantigens. (3) Formation of chronic granulomas. -Type I Immediate hypersensitivity is mast cell-derived mediators (vasoactive amines, lipid mediators and cytokines). Cytokine-mediated inflammation involves eosinophils, neutrophils and lymphocytes. Type I reactions can range from life-threatening anaphylactic reactions to milder manifestations associated with food allergies. -Type II Antibody mediated hypersensitivity is associated with complement and Fc receptor-mediated recruitment and activation of leukocytes (neutrophils and macrophages). Opsonization and phagocytosis of cells. Abnormalities in cellular function, e.g. hormone or neutrotransmitter receptor signaling. Types II and III are initiated by the interaction between antibody, except IgE, and antigen. Three different mechanisms of antibody-mediated injury exist in type II reactions: (1) Antibody-dependent, complement-mediated cytotoxic reactions characterized by the interaction of IgG or IgM antibody with cell-bound antigen. (2) Antibody-dependent, cell-mediated cytotoxicity that depends on the initial binding of specific antibodies to target cell surface antigens. (3) Antireceptor antibodies that disturb the functioning of receptors. Transfusion reactions are an example of an antibody-dependent, complement-mediated cytotoxic reaction. Hyperacute graft rejection is also an example of a Type II hypersensitivity reaction. -Type III Immune complex mediated hypersensitivity is associated with complement and Fc receptor-mediated recruitment and activation of leukocytes and tissue damage secondary to impaired blood flow. Type III reactions are caused by IgG, IgM and possibly other antibody types. Immune complexes can cover a spectrum of biological activities, including suppression or augmentation of the immune response by interacting with T and B cells; inhibition of tumor cell destruction; and deposition in blood vessel walls, glomerular membranes, and other sites. These deposits interrupt normal physiologic processes because of tissue damage secondary to the activation of complement and resulting activities.

Which one of the following factors, if deficient, would cause a prolongation of the PT assay without prolonging the aPTT? a. Factor XI b. Factor VII c. Factor VIII d. Factor IX

In this case, only factor VII of the factors listed above would affect the PT assay while resulting in a normal aPTT assay. Factor VII is an extrinsic factor and is assessed by the PT assay only. The remaining factors are included in the intrinsic pathway and would each prolong the aPTT if deficient. Factor VIII, Factor IX, and Factor XI are all part of the Intrinsic Factors, which also includes Factor XII. The intrinsic factors are assessed by aPTT only, and would not affect the PT assay. Common Pathway Factors- X, V, II, I (assessed by PT and aPTT).

Which symptom of HDFN does phototherapy help prevent? a. Anemia b. Kernicterus c. Enlarged spleen d. Hydrops

Phototherapy helps prevent kernicterus by chemically altering the bilirubin in the surface capillaries to harmless substances that can be excreted. Kernicterus is associated with severe cases of jaundice. Phototherapy cannot reverse the red cell destruction, anemia, or edema in the newborn.

Referring to the image on the right, what is the correct identification for each egg? (Picture not included, first egg has plugs on both ends of the egg) a. A: Hookworm egg B: Trichuris trichiura egg C: Enterobius vermicularis egg b. A: Ascaris lumbricoides egg B: Enterobius vermicularis egg C: Hookworm egg c. A: Diphyllobothrium latum egg B: Hookworm egg C: Trichuris trichuria egg d. A: Trichuris trichiura egg B: Hookworm egg C: Enterobius vermicularis egg

The correct answer is D- A: Trichuris trichiura egg B: Hookworm egg C: Enterobius vermicularis egg. Trichuris trichuris eggs have a thick shell with hyaline polar plugs. Hookworm eggs have a thin shell that is broadly oval and between four to eight cells inside when passed in stool. Enterobius vermicularis eggs have colorless shell that is flattened on one side and c-shaped larva inside. Ascaris lumbricoides have a bumpy (mammillated) outer shell. Diphyllobothrium latum eggs have an inconspicuous operculum with a small knob at the opposite end.

A modified acid-fast stain was used to produce the image on the right. What is the appropriate interpretation? Picture needed but couldn't copy over. Please select the single best answer a. Acid-fast positive b. Acid-fast negative c. Indeterminate d. Gram negative cocci

The correct answer is acid-fast positive (a.). Explanation: The modified acid-fast stain uses carbolfuchsin as the primary stain and a surface-active detergent to help the stain penetrate the waxy material of acid-fast organisms. Acid-alcohol is the decolorizing agent and methylene blue is used as the counterstain. The organism observed in this image is Cryptosporidium sp. which is in fact modified acid-fast positive. Acid-fast negative organisms will appear the same color as the background material in this image, which is based on the counterstain, thus would appear blue. This image represents an appropriately stained smear; thus the results are not indeterminate. Modified acid-fast staining does not allow for Gram reaction interpretation.

All of the statements below regarding amylase and lipase in pancreatitis are TRUE EXCEPT: Please select the single best answer a. Amylase and lipase are as predictive in chronic as in acute pancreatitis. b. Diagnosis sensitivity is increased by assaying both amylase and lipase. c. Urinary amylase: creatinine ratio is the most sensitive test for acute pancreatitis. d. Serum lipase peaks at 24 hours after an episode of acute pancreatitis and remains high for 7-8 days.

a. Amylase and lipase are as predictive in chronic as in acute pancreatitis. Serum amylase and lipase levels may be slightly elevated in chronic pancreatitis, but not diagnostic enough to predict chronic pancreatitis; wheras high levels are found only during acute pancreatitis episodes. In the later stages of chronic pancreatitis, normal to decreased levels of amylase and lipase are caused by the gradual inability of the pancreas to secrete the enzymes.

MacConkey agar contains all of the following except: a. Penicillin b. Bile salts c. Lactose d. Crystal violet

a. MacConkey Agar is a selective and differential medium. It selects for Enterobacteriaceae and other gram negative bacilli using bile salts and crystal violet present in the medium, which inhibit gram positive organisms. The medium is differential because it contains lactose, which will differentiate lactose fermenting gram negative bacilli from non-lactose fermenting gram negative bacilli. Lactose fermenters will appear pink on the agar, and non-lactose fermenters will appear clear. Penicillin is not used as an inhibitory factor in MacConkey agar or any other routine media. Oxacillin can be used in agars to select for organisms that are resistant, such as Methicillin Resistant Staphylococcus aureus (MRSA).

You are analyzing a PCR run and see that one of your samples is a high positive but the internal control failed. What should you do? a. Result the sample as positive b. Result the sample as negative c. Result the sample as indeterminate d. Repeat the sample

a. Result the sample as positive This sample can be resulted as positive. The purpose of an internal control is to show amplification occurred. Internal controls add extra assurance to negative samples that they are true negatives. However, because this sample is positive, we know amplification occurred even without the internal control. The sample is positive, even though the internal control didn't show up, therefore you would not result this sample as negative. Indeterminate results often occur from a sample being a very low level positive, where you can not accurately differentiate between positive and negative. In this case the sample is a clear positive. This sample would not need to be repeated and can be resulted as is.

Based on the morphologic features of this 40 µm (in diameter) ovum, as seen in the upper photomicrograph with the accompanying scolex of the adult worm illustrated in the lower image, select the presumptive identification of this cestode from the multiple choice answers listed below. Please select the single best answer a. Taenia saginata b. Taenia solium c. Hymenolepis nana d. Diphyllobothrium latum

a. Taenia solium is the correct response. Although the spherical ova with their thick, striate shell and internal hooklets does not rule out Taenia saginata, the scolex of T. solium, with its distinctive rostellum armed with a ring of hooklets, serves to exclude T. saginata, the scolex of which is flat and rounded and devoid of an armed rostellum. Taenia saginata ova are similar in appearance to those of T. solium, with a thick striated shell and three pairs of hooklet observed interiorly. Distinctive is the scolex of the adult worm with a round, smooth anterior end devoid of an armed rostellum. Hymenolepis nana ova have a thin outer non-striated shell and an inner membrane within which three pairs of hooklets are contained. The scolex of the adult worm also has an armed rostellum, but in contrast to T. solium, is small and projects outward. Diphyllobothrium latum ova are large (up to 70 µm), and have a thin smooth shell with a distinctive, inconspicuous non-shouldered operculum at one end. The scolex of the adult worm is long and narrow with a dorso-ventral groove surrounded on either side by lateral lip-like folds.

A pleural fluid specimen is received in the laboratory for aerobic culture. The specimen measures about 0.5 mL. How should the tech process this specimen for culture? Please select the single best answer a. Use a sterile pipette and add a drop of fluid per agar plate b. Insert a swab into the specimen container and inoculate all agar plates c. Add 0.5 mL of saline to the 0.5 mL of fluid, vortex, and inoculate plates d. Reject the specimen due to insufficient quantity for culture

a. Use a sterile pipette and add a drop of fluid per agar plate Pleural fluid is a sterile fluid and should be inoculated directly to the agar plates. The best way to recover the most organisms is to use a drop of fluid on each plate, obtained via sterile pipette to not contaminate the culture. Swabs are typically not used to inoculate fluids. If there is a sterile body fluid, the number of organisms present in the sample can be low, and the best way to promote organism recovery is to use a drop of the fluid instead of a swab for inoculation. Adding saline to the fluid would dilute out the fluid and decrease the ability to recover organisms present in the fluid. This sample is a sterile body fluid and may be an irretrievable specimen (difficult to recollect) and should be cultured. Even though the volume is low, there is still enough to inoculate agar plates and culture the specimen.

The absence of which clotting factor causes NO bleeding tendency? Please select the single best answer a. Antihemophiliac factor (VIII) b. Plasma thromboplastin component (IX) c. Hageman factor (XII) d. Plasma thromboplastin antecedent (XI)

c. Hageman factor (XII) Factor XII, or Hageman factor, may cause prolonged clotting in the specimen tubes, but does not cause abnormal coagulation symptoms or complications in vivo. A Factor VIII, IX, or XI deficiency are all types of hemophilia, which are inherited hemorrhagic disorders caused by a genetic mutation of the gene coding for the coagulation factor.

Which of the following is a TRUE statement when performing maintenance or repair tasks on electrical equipment? a. The Occupational Health and Safety Administration (OSHA) does NOT have any standards that are aimed at protecting workers from injury when performing maintenance or repair tasks on electrical equipment. b. It is unnecessary to take any additional measures when performing repairs on a piece of equipment, except to turn it off. c. Tagout is the placement of a tag on the energy-isolating device notifying staff: "Do Not Operate this Equipment." d. Two-prong electrical plugs are acceptable for all equipment.

c. Tagout is the placement of a tag on the energy-isolating device notifying staff: "Do Not Operate this Equipment." Explanation: Tagout is the placement of a tag on the energy-isolating device notifying staff: "Do Not Operate this Equipment." Lockout is the physical placement of a device on the breaker or the placement of the plug into a canister to prevent its use. Extra measures are necessary to reduce the risk of shock or injury due to the unexpected startup of equipment or the release of hazardous energy while a machine or instrument is out of service. Simply turning it off is NOT sufficient. The OSHA Control of Hazardous Energy standard, 29 CFR 1910.147, requires lockout/tagout (LOTO) procedures to protect workers whenever maintenance or repair tasks are being performed on electrical equipment. Three-prong electrical plugs should be used for all equipment.


संबंधित स्टडी सेट्स

Pathophysiology- Genetic and Cogenital Disorders

View Set

Total and net ionic reactions readings

View Set

CP Ch. 9: Legal and Ethical Issues

View Set

Sesh 20 Seeing well, + 21 signalling your intention

View Set

Chapter 5: Sensation and Perception

View Set

VHL Leccion 14 Prueba de Practíca

View Set

4510: Concept Synthesis, Exam 1 - Oxygenation, Perfusion, Elimination, Fluid and Electrolyte, Infection

View Set

Personal Lines (PA) Practice Exam Study Set

View Set